Download as pdf or txt
Download as pdf or txt
You are on page 1of 80

MATEMATIČKO–FIZIČKI LIST (MFL) SADRŽAJ

za učenike i nastavnike.
Izlazi u četiri broja tokom školske godine.
Matematika
Izdaju:
Željko Hanjš,
HRVATSKO MATEMATIČKO DRUŠTVO i
Soroban – japanski abak . . . . . . . . . 67
HRVATSKO FIZIKALNO DRUŠTVO
Pretplata za 2008./ 2009. je 100 kn, pojedini broj Mirjana Čižmešija, Maja Kozulić, Boško Šego
25 kn. Primjena osnovne analize vremenskog
Za inozemstvo pretplata je 28 EUR, a pojedini broj niza u poslovanju fondova . . . . . . . . 69
7 EUR. Marko Juraić,
(Uplata se može obaviti u kunama ili devizama po Povijest matrica i determinanti . . . . . . . 76
tečaju u trenutku plaćanja.) Roko Pešić,
Adresa lista je: S razredbenog ispita u Japanu . . . . . . . 80
“Matematičko–fizički list, Bijenička 32, 10001 Petar Vranjković,
Zagreb, tel. (01) 4833-891, fax (01) 4683-535. Jedno svojstvo logaritamske funkcije
Uplate na žiro račun: i njegova primjena . . . . . . . . . . . . 82
Hrvatsko fizikalno društvo, Zagreb, Fizika
br. 2360000-1101301202 (kn), Tome Antičić i Vuko Brigljević,
ZBZ d.d. SWIFT ZABA HRXX 70313-978-3239853 LHC: zašto mu se divimo? . . . . . . . . 86
(EUR).
Ljiljana Sudar,
Na uplatnici kao svrhu uplate molimo naznačite
Geometrija i problemi kretanja . . . . . . . 95
“za MFL”!
Molimo Vas da kod svake uplate pošaljete (foto-) Astronomija
kopiju uplatnice ili da nas obavijestite telefonom Dario Hrupec,
ili elektronskom poštom o uplati. Teleskop MAGIC – čarobni instrument
URL: http:/ / web.math.hr/ mfl, e-mail: mfl@hfd.hr astročestične fizike . . . . . . . . . . . . 102
Zabavna matematika . . . . . . . . . . 105
-
Uredivački odbor: Zadaci i rješenja
ŽELJKO HANJŠ (Zagreb), glavni i odgovorni urednik, A) Zadaci iz matematike . . . . . . . . . 106
e-mail: hanjs@math.hr B) Zadaci iz fizike . . . . . . . . . . . . 106
MATKO MILIN (Zagreb), urednik za fiziku,
e-mail: matkom@phy.hr C) Rješenja iz matematike . . . . . . . . . 107
ANTE BILUŠIĆ (Split), IGOR GAŠPARIĆ, ZDRAVKO D) Rješenja iz fizike . . . . . . . . . . . 112
KURNIK, VLADIMIR PAAR, MAJA PLANINIĆ, MIRKO Zanimljivosti
POLONIJO, DUBRAVKA SALOPEK WEBER, SAŠA
SINGER, ANA SMONTARA, BOŠKO ŠEGO, VLADIMIR
Sanja Varošanec, Matematička konferencija
VOLENEC, MLADEN VUKOVIĆ, tajnica SANDRA u čast akademiku Josipu Pečariću . . . . . 117
POŽAR (Zagreb), e-mail: sandra@phy.hr -
49. medunarodna matematička olimpijada . . 120
Izdavački savjet: Druga srednjoeuropska matematička olimpijada,
ALEKSA BJELIŠ (Zagreb), LIDIJA COLOMBO (Zagreb),
Olomouc, Češka, 4. – 10. rujna 2008. . . . 123
BRANIMIR DAKIĆ (Zagreb), VLADIMIR DEVIDÉ (Za-
-
Medunarodno matematičko natjecanje
greb), MARIJAN HUSAK (Varaždin), MARGITA PA- “Klokan bez granica” 2008. g. . . . . . . 126
VLEKOVIĆ (Osijek), ERNA ŠUŠTAR (Zagreb), PETAR Nove knjige
VRANJKOVIĆ (Zadar), VLADIS VUJNOVIĆ (Zagreb),
PAŠKO ŽUPANOVIĆ (Split)
Žarko Dadić,
Egzaktne znanosti u Hrvata u
List financijski pomaže Ministarstvo znanosti, obrazovanja poslijeprosvjetiteljskom razdoblju
i športa Republike Hrvatske. (1789.–1835.) . . . . . . . . . . . . . . 136
Kvalifikacijski ispiti
Slog i prijelom: Razredbeni ispit na FOI-u iz
Element, Zagreb, Menčetićeva 2 matematike 2008. g. . . . . . . . . . . . 138
Tisak: Nagradni natječaj br. 185 . . . . . . . . 144
Tiskara Zelina d.d., Sv. Ivan Zelina, Ul. K. Krizmanić 1
Naklada ovog broja 3000 primjeraka

Slika na naslovnici prikazuje detektor CMS na CERN-u u


fazi sklapanja.
Dragi čitatelji!

Početkom jeseni ove godine po cijelom je svijetu odjeknula vijest iz CERN-a o


puštanju u rad Velikog hadronskog sudarivača, LHC (engl. Large Hadron Collider)
za ispitivanje materije. Očekuju se sudari s najvišim energijama čestica koje se kreću
skoro brzinama svjetlosti. Protoni i druge čestice kreću se u tunelu opsega 27 km 100
metara ispod površine Zemlje. Eksperimenti uključuju više tisuća znanstvenika iz cijelog
svijeta, a trajat će sljedećih 10 do 20 godina, u nastojanju da se odgonetne što više tajni
svemira. U taj pothvat uključeno je i desetak hrvatskih fizičara. O ovoj zanimljivoj temi
upoznaju nas Tome Antičić i Vuko Brigljević, viši znanstveni suradnici u Zavodu za
eksperimentalnu fiziku Instituta “Ruder - Bošković” u Zagrebu.

U prilogu profesorice Ljiljane Sudar iz Leskovca, Geometrija i problemi kretanja,


pokazuje kako se mnogi problemi kretanja efikasno i elegantno rješavaju pomoću
nestandarne i veoma moćne metode, koja se bazira na primjeni grafova i geometrije.
U prošlosti, ne tako davnoj, koristila su se razna računala u svakodnevnom životu.
Ovdje se, u prilogu Soroban – japanski abak, samo prisjećamo ruskog, kineskog i
japanskog abaka. Učenicima i nastavnicima ekonomskih škola namijenjen je članak
Primjena osnovne analize vremenskog niza u poslovanju fondova, koji su priredili
Mirjana Čižmešija, Maja Kozulić i Boško Šego s Ekonomskog fakulteta u Zagrebu.
U prilogu Povijest matrica i determinanti Marka Juraića iz Zagreba dan je ukratko
povijesni prikaz ove grane matematike. Roko Pešić navodi jedan zanimljiv zadatak
koji se pojavio na razredbeom ispitu na jednom sveučilištu u Japanu. Profesor Petar
Vranjković s gimnazije u Zadru opisuje jedno zanimljivo svojstvo logaritamske funkcije
i njegovu primjenu u rješavanju raznih nejednakosti.
Dario Hrupec iz Koprivnice u prilogu Teleskop MAGIC – čarobni instrument
astročestične fizike opisuje novo, interdisciplinarno, znanstvenoistraživačko područje
koje uključuje fiziku čestica, astronomiju, astrofiziku te kozmologiju.
U Trogiru je održana matematička konferencija u čast akademiku Josipu Pečariću,
svjetski poznatom matematičaru iz područja matematičkih nejednakosti, povodom
-
njegovog 60-tog rodendana. -
Tu su još izvješća za zadacima s 49. medunarodne
-
matematičke olimpijade, Druge srednjoeuropske matematičke olimpijade i Medunarodnog
matematičkog natjecanja “Klokan bez granica” 2008. g.
Akademik Žarko Dadić, autor mnogih knjiga iz fizike i matematike objavio je još
jednu, Egzaktne znanosti u Hrvata u poslijeprosvjetiteljskom razdoblju (1789.–1835.).
Na zadnjoj strani omota prisjetili smo se redovitog profesora PMF-a Matematičkog
odjela, znanstvenika-geometričara, metodičara, autora udžbenika i popularizatora
matematike.

Uredništvo lista

66 Matematičko-fizički list, LIX 2 (2008. – 2009.)


Soroban – japanski abak

Željko Hanjš, Zagreb

Abak je naziv za napravu koja je služila za računanje s


većim brojevima, a tokom povijesti konstruirane su različite
njegove vrste. Spomenut ćemo tri najpoznatija abaka: ruski,
kineski i japanski. Pravokutnog su oblika, napravljeni od
drveta, sa žicama ili prutićima za kuglice. Kineski i japanski
abak imaju neparan broj prutića, a može ih biti i preko 30.
Ruski abak ima deset žica s po deset kuglica, kao na slici 1.
Engleska riječ abacus etimološki je izvedena iz grčke
riječi αβακ ι oν , koja označava daščice za računanje
pokrivenu pijeskom, a ova potječe od semitske riječi koja
ima značenje daščicu za računanje pokrivenoj prašinom ili
pijeskom. U grčko i rimsko doba koristili su kuglice ili
zrnca od kamena ili metala. Slika 1. Ruski abak
Kineski abak ima svoje porijeklo
u rimskom abaku. Datira iz sredine
16. stoljeća, imao je sedam kuglica na
svakom prutiću, dvije u gornjem i pet
u donjem dijelu, kao što je prikazano
na slici 2. Prenesen u Japan negdje u
17. stoljeću postao je vrlo popularnom
pomoćnom napravom za računanje u
trgovinama. U Japanu su racionalizirali
ovaj abak odbacujući suvišne kuglice.
Slika 2. Kineski abak
Oko godine 1920. japanski abak, koji se tamo nazviva soroban, poprimio je novu
konfiguraciju. Napravljen je od drveta, najčešće hrasta, i na svakom prutiću, obično od
bambusa, ima po pet kuglica, jednu u gornjem i četiri u donjem dijelu ploče. Pokazalo
se da je to najracionalnije, jer se kod obavljanja operacija na kineskom abaku nikad ne
mora koristiti krajnja donja niti krajnja gornja kuglica. Blaise Pascal, veliki francuski
matematičar, u 17. stoljeću medu- prvima je konstruirao nekoliko modernijih računskih
strojeva.

Slika 3. Japanski abak


Japanski abak je jednostavna naprava koja omogućuje brzo obavljanje aritmetičkih
računskih operacija, a bazira se na decimalnom sistemu. Svako od 4 zrnca na donjem

Matematičko-fizički list, LIX 2 (2008. – 2009.) 67


dijelu prutića abaka ima vrijednost 1, a zrnce na njegovom gornjem dijelu ima vrijednost
5. U neutralnom položaju, kada su donja zrnca pomaknuta do kraja dolje, a gornje do
kraja gore, njegova vrijednost je jednaka nuli. Tek pomakom zrnca prema sredini ono
poprima odredenu - vrijednost, 1 ili 5.
Prije pojave elektroničkih računala/ kalkulatora soroban je u Japanu bio u širokoj
upotrebi. Bio je vrlo koristan u školama za učenje osnovnih računskih operacija.
Štoviše, korištenjem sorobana razvijale su se i sposobnosti mentalnog računanja. Tada
- da će abak u budućnosti imati vrlo važnu ulogu, ne samo u svakodnevnom
se predvidalo
životu.
Još prije tridesetak godina soroban je bio u velikoj upotrebi, ali su ga potiskivali
kalkulatori i danas se više gotovo i ne koristi. U Japanu se više ne proizvode klasični
sorobani i zadržali su se još samo kao neizbježni turistički suveniri sa samo do desetak
prutića, najčešće od plastike. Ipak uz dosta truda može se tek tu i tamo u trgovini
naći i klasični soroban s 23 prutića. Po veličini sorobani se dijele na male (s 13
prutića), srednje (s 15 prutića) i velike (s 23 prutića). Inače ih ima i s po 27, pa čak
i s 31 prutićem, ali su najčešći bili oni s 21 prutićem jer su uglavnom zadovoljavali
svakodnevne potrebe. U pravilu broj prutića mora biti neparan.
Godine 1954. Takashi Kojima je izdao knjižicu The Japanese Abacus, Its Use
and Theory da bi 1996. izašlo već njezino 36. izdanje. Tamo je na mnogo primjera
ilustrirano kako se vrše osnovne računske operacije: zbrajanje, oduzimanje, množenje,
dijeljenje. Takoder - su se održavala natjecanja oko 40-tih godina prošlog stoljeća izmedu
-
tadašnjih elektroničkih računala i natjecatelja koji su koristili soroban. Godine 1946.
održano je u japanu jedno takvo natjecanje, a pobjeda japanskog abaka bila je tada
još uvijek neupitna. Tada su stručnjaci za soroban kod zbrajanja i oduzimanja bili brži
od tadašnjih elektroničkih računala. U Japanu je nekad postojao Institut za istraživanje
abaka.
S vremenom je abak/ soroban postigao nevjerojatnu popularnost u Japanu. Godine
1965. oko milijun zainteresiranih ljudi je pristupilo ispitivanju vlastitih sposobnosti
računanja na sorobanu, od kojih je njih četvrtina položila ispit (5000 ih je postiglo
odobrenje prvog, 25 000 drugog i 230 000 trećeg stupnja).
Abak je bio uključen u nastavni program za osnovne škole kod učenja elemenata
aritmetike, a izučavao se i za potrebe u trgovini. Ukratko, abak je postao vrlo popularnim
predmetom kojeg je imalo skoro svako domaćinstvo.
Kako objasniti iznenadan pad popularnosti nekad neizbježnog i vrlo popularnog
abaka, u vrijeme modernizacije tokom dvadesetog stoljeća? Bez ikakve dvojbe glavno
objašnjenje je u činjenici da su operacijske metode kod sve modernijih kalkulatora
postale znatno jednostavnije, savršenije i brže.
Iako su se u poslovima u Japanu sve više koristila elektronička računala, nije bilo
-
iznenadujuće što se abak u velikoj mjeri još dugo koristio, kako u malim, tako i u
velikim trgovinama.
Još sredinom prošlog stoljeća stručnjaci za abak/ soroban bili su mišljenja da će
vrijeme utrošeno na učenje i vježbavanje operacija na tom računalu, biti korisno za
učenje aritmetike, a ujedno će biti ogromna pomoć u poslovima i u svakodnevnom
životu.
Abak je izuzetno važan i kao pomoć slijepima. U japanskim školama za slijepe, sve
vrste pomagala za računanje baziraju se na abaku. Čak su se od 1964. g. provodili ispiti
računanja za slijepe osobe.
Obavljanje operacija na sorobanu je potpuno identično rukovanju s kineskim abakom.
U časopisu Matka, 1994. br. 10, opisano je ukratko kako se na kineskom abaku vrši
zbrajanje, oduzimanje i množenje, a u Matematičko-fizičkom listu, 1994./ 95. br. 2/ 178,
dani su algoritni za računanje drugog i trećeg korijena.
Ipak i ovo je tek prisjećanje na nekad zlatno doba abaka/ sorobana.

68 Matematičko-fizički list, LIX 2 (2008. – 2009.)


Primjena osnovne analize vremenskog niza u poslovanju fondova

Mirjana Čižmešija ∗ , Maja Kozulić ∗∗ , Boško Šego ∗∗∗ , Zagreb

Uvod

Analiza vremenskih nizova ima široko područje primjene kako u stručnim tako i u
znanstvenim istraživanjima. Analiza dinamike promatrane pojave važna je za donošenje
poslovnih odluka, a može poslužiti i kao podloga za daljnje složenije statističke analize.
Vremenski niz je niz kronološki uredenih - vrijednosti promatrane pojave, a notira se
na sljedeći način:
Y : y1 , y2 , ..., yt , ..., yn t = 1, 2, 3, ..., n, (1)
pri čemu je Y promatrana pojava u vremenu, yt je vrijednost pojave Y u vremenu t
(frekvencija vremenskog niza), a n je duljina vremenskog niza.
Prema načinu promatranja pojave u vremenu razlikuju se intervalni i trenutačni
vremenski nizovi. Intervalni niz nastaje promatranjem odredene - -
pojave u odredenim
vremenskim intervalima (godina, tromjesečje, mjesec, dan i sl.). Često se u ekonomiji
promatraju sljedeće pojave: godišnja proizvodnja, mjesečni broj prevezenih putnika,
dnevni ostvareni promet, i dr. Intervalni vremenski niz ima svojstvo kumulativnosti
što znači da zbroj frekvencija intervalnog niza ima smislenu interpretaciju. Trenutačni
vremenski niz nastaje promatranjem pojave u odredenom - trenutku (npr. stanje na tekućem
računu, broj zaposlenih i dr.). Trenutačni niz nema svojstvo kumulativnosti.
Vremenski nizovi mogu biti izvorni i izvedeni. U izvornom nizu frekvencije
nastaju izravnim mjerenjem pojave po odabranim intervalima vremena ili u odabranim
vremenskim točkama. Izvedeni vremenski niz čine frekvencije koje su nastale primjenom
brojčanih operacija nad izvornim vrijednostima jednog ili više vremenskih nizova.

Grafička analiza vremenskog niza

Veoma korisno sredstvo u analizi vremenskog niza su grafički prikazi. U tabličnim


kalkulatorima kao što su Excel, Lotus i drugi moguće je konstruirati različite vrste
grafičkih prikaza. Najčešće se vremenski nizovi prikazuju površinskim i linijskim
grafikonima. Za prikaz vremenskog niza koji pokazuje sezonske oscilacije pojave s
vremenom koristi se polarni dijagram. Grafikon mora imati naslov, izvor podataka
(ako se koriste sekundarni podaci) i po potrebi napomene i legendu. Ako se frekvencije
vremenskog niza nalaze na visokoj brojčanoj razini (daleko od nule) i s malim rasponom
varijacije, prikladno je primijeniti horizontalni prekid mjerila (prekida se mjerilo na osi
ordinata). Uobičajeno se na osi apscisa nalazi aritmetičko mjerilo za vrijeme, a na osi
ordinata je aritmetičko mjerilo za frekvencije razmatrane pojave

∗ Docentica na Ekonomskom fakultetu Sveučilišta u Zagrebu, Katedra za statistiku.


∗∗ Studentica Ekonomskog fakulteta Sveučilišta u Zagrebu.
∗∗∗ Redoviti profesor na Ekonomskom fakultetu Sveučilišta u Zagrebu, Katedra za matematiku.

Matematičko-fizički list, LIX 2 (2008. – 2009.) 69


Intervalni niz prikazuje se površinskim ili linijskim grafikonom tako da se grafikon
konstruira u prvom kvadrantu koordinatnog sustava. 1 Trenutačni vremenski niz prikazuje
se samo linijskim grafikonom.

Osnovni pokazatelji dinamike vremenskog niza

Iz grafičkog prikaza mogu se uočiti osnovne karakteristike stanja i razvoja promatrane


-
pojave u odredenom vremenu. Te se promjene mogu kvantificirati izračunom osnovnih
pokazatelja dinamike vremenskog niza. Pokazateljima se mjere promjene koje je moguće
razlikovati prema više kriterija:
— prema razdoblju za koje se pokazatelj izračunava:
• pojedinačne promjene,
• prosječne promjene,
— prema mjernoj jedinici u kojoj su promjene izražene:
• apsolutne promjene,
• relativne promjene,
— prema razdoblju s kojim se usporeduje: -
• promjene u odnosu na razinu pojave u odabranom baznom vremenskom
razdoblju,
• promjene razine pojave u uzastopnim vremenskim razdobljima.
Slijedi kratki pregled osnovnih pokazatelja dinamike po navedenim kriterijima.
Pojedinačne, apsolutno izražene, promjene pojave u uzastopnim razdobljima (prve
diferencije)
Ako su Y : y1 , y2 , ..., yt , ..., yn , t = 1, 2, 3, ..., n , frekvencije vremenskog niza
s jednakim intervalima promatranja, pojedinačne promjene u apsolutnom iznosu u
uzastopnim razdobljima dane su izrazom:
Δyt = yt − yt−1 , t = 2, 3, 4, ..., n. (2)
Navedene promjene tumače se kao povećanje (ako je vrijednost pojave definirane
formulom (2) pozitivan broj) ili smanjenje (ako je riječ o negativnom broju) promatrane
pojave u tekućem razdoblju u odnosu na prethodno razdoblje. Naravno, ako je Δyt = 0
za dano t , onda u trenutku t nije došlo do promjene vrijednosti pojave Y u odnosu
na trenutak t − 1 . Pri tome su promjene izražene u istim mjernim jedinicama kao i
promatrana pojava (u apsolutnim mjernim jedinicama).
Pojedinačne, apsolutno izražene promjene pojave u odnosu na (fiksno) bazno
razdoblje
Δy∗t = yt − yt−1 , t = 1, 2, 3, 4, ..., n. (3)
Ove se promjene tumače kao porast ili smanjenje promatrane pojave u tekućem
- su izražene u istim
razdoblju u odnosu na neko fiksno, bazno razdoblje. Takoder
mjernim jedinicama kao i pojava za koju se izračunavaju.

1 Ako se prikazuje intervalni vremenski niz s nejednakim razdobljima promatranja, potrebno je korigirati
frekvencije niza. Korigiranje frekvencija ne provodi se kod trenutačnog niza jer frekvencije toga niza pokazuju
-
stanje pojave u odredenom -
trenutku, a ne nad odredenim razdobljem.

70 Matematičko-fizički list, LIX 2 (2008. – 2009.)


Prosječna promjena pojave (prosječna prva diferencija)
n
Δyt
t=2 yn − y1
Δy = ili Δy = . (4)
(n − 1) n−1
Budući da se prosječna prva diferencija izračunava iz razlike posljednje i prve
frekvencije, a ostale frekvencije vremenskog niza nemaju utjecaj na veličinu prosjeka,
ovu je prosječnu mjeru promjene pojave uputno koristiti ako pojava ne pokazuje značajne
varijacije u vremenu (izmedu- prvog i posljednjeg razdoblja promatranja).
Pojedinačne stope promjene pojave u uzastopnim razdobljima
yt − yt−1
st = · 100, t = 2, 3, 4, ..., n. (5)
yt−1
Stopa promjene pojave pokazuje za koliko se (izraženo relativno) promijenila razina
pojave u vremenu t u odnosu na vrijeme t − 1 .
Pojedinačne stope promjene pojave u odnosu na (fiksno) bazno razdoblje
yt − yb
s∗t = · 100, t = 1, 2, 3, ..., n. (6)
yb
Stopa promjene pojave u odnosu na bazno razdoblje pokazuje za koliko se (izraženo
relativno) promijenila razina pojave u vremenu t u odnosu na bazno razdoblje b .
-
Ako je potrebno izmjeriti prosječnu (relativno izraženu) promjenu pojave u odredenom
razdoblju, izračunava se prosječna stopa promjene.
Prosječna stopa promjene pojave
  
yn
s = (G − 1) · 100, G= n−1 −1 , (7)
y1
pri čemu je G geometrijska sredina.
- nije uputno koristiti ako vremenski niz pokazuje
Prosječnu stopu promjene takoder
izrazito velike oscilacije s vremenom. Time prva i posljednja frekvencija niza nisu
pouzdan pokazatelj dinamike razvoja pojave unutar cijelog niza.
Prognoza pojave pomoću prosječne stope promjene
Ako pojava nema izražene varijacije u vremenu i ako se pretpostavi da će se i u
-
narednom (kratkoročnom) razdoblju mijenjati prema utvrdenoj prosječnoj stopi, može
se predvidjeti buduća razina pojave u vremenu n + τ koristeći se formulom
ŷn+τ = yn · Gτ , (8)
pri čemu je:
τ – broj razdoblja nakon posljednjeg za koje se provodi prognoza,
ŷn+τ – prognozirana vrijednost pojave u razdoblju n + τ .
Individualni indeksi
U osnovnoj numeričkoj analizi vremenskog niza nezaobilazno je korištenje
individualnih indeksa. Ako se pomoću indeksa mjeri dinamika razvoja jedne pojave tada
su to individualni indeksi. Skupni indeksi mjere dinamiku skupine pojava. Individualni
indeksi mogu biti verižni (lančani) i bazni. Indeksi su pozitivni brojevi, a mogu biti
manji od sto, veći od sto ili jednaki sto.
Verižnim indeksom se mjeri promjena pojave u tekućem razdoblju u odnosu na
prethodno razdoblje (izražena relativno). To su stope promjene pojave u uzastopnim

Matematičko-fizički list, LIX 2 (2008. – 2009.) 71


vremenskim razdobljima povećane za 100. Verižni indeks pokazuje koliko jedinica
pojave u vremenu t dolazi na sto jedinica pojave u vremenu t − 1 .
yt
Vt = · 100, t = 2, 3, 4, ..., n, (9)
yt−1
Vt = st + 100. (10)
Verižni indeksi se mogu grafički prikazati grafikonom jednostavnih stupaca 2 ili linijskim
grafikonom.
Bazni indeksi (indeksi na stalnoj bazi) izražavaju promjenu pojave u tekućem
razdoblju u odnosu na bazno razdoblje (izražena relativno). To su stope promjene pojave
u odnosu na bazno razdoblje povećane za 100. Bazni indeks pokazuje koliko jedinica
pojave u vremenu t dolazi na sto jedinica pojave u baznom vremenu b :
yt
It = · 100, t = 1, 2, 3, 4, ..., n, (11)
yb

It = st + 100. (12)
Budući da se pri izračunu baznih indeksa frekvencije niza dijele istim brojem, indeksi
na stalnoj bazi proporcionalni su veličinama iz kojih su izračunani (frekvencijama
vremenskog niza). Kao i verižni indeksi, oni su jednaki sto, manji od sto ili veći od sto.

Primjena osnovne analize vremenskog niza u poslovanju fondova

cijena udjela
datum
fonda u kn
25.10.2006. 154.86
25.11.2006. 154.47
25.12.2006. 157.11
25.01.2007. 164.75
25.02.6007. 177.48
25.03.2007. 184.44
25.04.2007. 200.71
25.05.2007. 206.12
25.06.2007. 204.09
25.07.2007. 210.74
25.08.2007. 206.67
25.09.2007. 211.12
25.10.2007. 220.63
26.11.2007. 198.92
24.12.2007. 208.43
25.01.2008. 190.11
25.02.2008. 185.13
25.03.2008. 169.72
24.04.2008. 164.85
Tablica 1. Cijene udjela PBZ Equity fonda. Izvor: www.pbzinvest.hr
Na temelju podataka o cijenama udjela PBZ Equity fonda 3 provedena je empirijska
analiza vremenskog niza za razdoblje od 25. listopada 2006. do 24. travnja 2008. godine.
Duljina vremenskog niza je 19. Cijene su bilježene gotovo istog dana u mjesecu, stoga
2 Pri tome se na osi apscisa nanosi aritmetičko mjerilo za vrijeme koje upućuju na promjenu u tekućem prema
prethodnom razdoblju.

72 Matematičko-fizički list, LIX 2 (2008. – 2009.)


je vremenski niz cijena udjela fonda trenutačni vremenski niz (tablica 1).
Cijena udjela u fondu formira se ovisno o kretanjima cijena dionica u koje fond ulaže,
ovisi o tržištu efektive, te o kretanju cijena državnih obveznica. Najvažniji cilj Fonda je
sigurno plasirati prikupljena sredstva, te osigurati stalnu likvidnost uloga imatelja, uz što
veću profitabilnost ulaganja. Osnovna analiza vremenskog niza je od posebne važnosti
u donošenju poslovnih odluka svih onih koji su direktno ili indirektno vezani uz rad
fonda.
240
220
200
cijena, kn

180
160
140
120
100
25.10.2006.

25.12.2006.

25.02.6007.

25.04.2007.

25.06.2007.

25.08.2007.

25.10.2007.

24.12.2007.

25.02.2008.

24.04.2008.
Grafikon 1. Cijene udjela PBZ equity fonda. Izvor: www.pbzinvest.hr
stope
cijena
prve pojedinačne promjene
udjela verižni bazni indeksi
datum diferencije stope cijene u odnosu
fonda indeksi 25.04.06 = 100
cijene udjela na bazu
u kn
udjela b = 25.04.2007.
t yt Vt It Δyt st s∗t
1 2 3 4 5 6 7
25.10.2006. 154.86 − 77.16 − − − 22.84
25.11.2006. 154.47 99.75 76.96 −0.39 −0.25 −23.04
25.12.2006. 157.11 101.71 78.28 2.64 1.71 −21.72
25.01.2007. 164.75 104.86 82.08 7.64 4.86 −17.92
25.02.6007. 177.48 107.73 88.43 12.73 7.73 −11.57
25.03.2007. 184.44 103.92 91.89 6.96 3.92 −8.11
25.04.2007. 200.71 108.82 100.00 16.27 8.82 0.00
25.05.2007. 206.12 102.70 102.70 5.41 2.69 2.70
25.06.2007. 204.09 99.02 101.68 −2.03 −0.98 1.68
25.07.2007. 210.74 103.26 105.00 6.65 3.26 5.00
25.08.2007. 206.67 98.07 102.97 −4.07 −1.93 2.97
25.09.2007. 211.12 102.15 105.19 4.45 2.15 5.19
25.10.2007. 220.63 104.50 109.92 9.51 4.50 9.92
25.11.2007. 198.92 90.16 99.11 −21.71 −9.84 −0.89
24.12.2007. 208.43 104.78 103.85 9.51 4.78 3.85
25.01.2008. 190.11 91.21 94.72 −18.32 −8.79 −5.28
25.02.2008. 185.13 97.38 92.24 −4.98 −2.62 −7.76
25.03.2008. 169.72 91.68 84.56 −15.41 −8.32 −15.44
24.04.2008. 164.85 97.13 82.13 −4.87 −2.87 −17.87
Tablica 2. Kretanje cijene udjela PBZ Equity fonda kroz godinu dana
Iz navedenog grafičkog prikaza (grafikon 1) može se uočiti da su cijene udjela do
listopada prošle godine pokazale dinamiku ujednačenog rasta, a od tada se, s iznimkom
u siječnju 2008. godine kada su se povećale, kontinuirano smanjuju. Uočenu dinamiku
kretanja moguće je kvantitativno izraziti osnovnim pokazateljima dinamike kako je
izloženo u prethodnom dijelu rada (tablica 2). Pri tome je nužno ukazati na ograničenja
primjene nekih od pokazatelja korištenima u analizi navedenog vremenskog niza cijena
udjela.
Matematičko-fizički list, LIX 2 (2008. – 2009.) 73
Diferencija uz datum 25.04.2007. prikazuje da je na taj dan cijena udjela u Equity
fondu bila veća za 16.25 kn u odnosu na mjesec prije tj. u odnosu na 25.03.2007. No,
diferencija uz datum 25.08.2007. prikazuje da je cijena udjela toga dana bila manja za
4.07 kn u odnosu na 25.07.2007. Diferencije ostalih dana u mjesecu interpretiraju se na
isti način.
Pojedinačna stopa promjene cijene udjela Equity fonda na dan 25.05.2007. godine
iznosi 2.69. To znači da je prinos toga dana bio veći za 2.69% u odnosu na 25.04.2007.
Dok je 25.06.2007. godine stopa promjene cijene iznosi −0.98 . To znači da je prinos
tog dana bio manji za 0.98% u odnosu na prethodno razdoblje.
Stopa promjene u odnosu na bazno razdoblje pokazuje za koliko se relativno
promijenila razina pojave u vremenu t u odnosu na bazno razdoblje. Stopa promjene
25.11.2006. u odnosu na bazno razdoblje 25.04.2007. iznosila je −23.04 , što znači
da je prinos cijena udjela u Fondu tog datuma u odnosu na bazni datum bio niži za
23.04%, dok je 25.10.2007. stopa promjene iznosila 9.92 u odnosu na bazno razdoblje,
što je za 9.92% viša cijena udjela u Fondu u odnosu na bazno razdoblje. Tumačenje
stopa promjene u uzastopnim razdobljima jednako je tumačenju verižnih indeksa (tabela
2, stupac 3), a stope promjene u odnosu na bazno razdoblje tumače se isto kao i bazni
indeksi (tabela 2, stupac 4), odnosno bazni indeks cijene udjela 25.10.2006. iznosi 77.16
što pokazuje da je na 100 kuna cijene udjela u fondu 25.04.2007. dolazilo 77.16 kuna
cijene udjela 25.10.2006., odnosno da je cijena udjela u fondu bila manja za 22.84%.
U odnosu na bazni datum 25.04.2007. najveći pad cijene udjela u fondu bio
je 25.11.2006., dok je najveći porast cijene u odnosu na bazni datum zabilježen
25.10.2007.
-
Ranije je već konstatirano, a grafikon 3 to i potvrduje, da su se cijene udjela do
listopada 2007. godine uglavnom povećavale u odnosu na prethodni mjesec. Stupci na
grafikonu verižnih indeksa koji su iznad osi 100 pokazuju povećanje. Smanjenje u tom
razdoblju zabilježeno je u studenome 2006., lipnju i kolovozu 2007. godine. Nakon toga
je uslijedilo razdoblje kada su verižni indeksi bili manji od 100, stupci na grafikonu
su stoga ispod osi 100, što znači da su se cijene uzastopno smanjivale u odnosu na
prethodni mjesec osim u prosincu kad su se povećale za nešto manje od 5%.
115
indeksni bodovi

105

95

85
11.06./10.06.

12.06./11.06.

01.07./12.06.

02.07./01.07.

03.07./02.07.

04.07./03.07.

05.07./04.07.

06.07./05.07.

07.07./06.07.

08.07./07.07.

09.07./08.07.

10.07./09.07.

11.07./10.07.

12.07./11.07.

01.08./12.07.

02.07./01.08.

03.08./02.08.

04.08./03.08.

Grafikon 3. Verižni indeksi cijena udjela PBZ Equity fonda. Izvor: izračun autora

Budući da je iz grafičkog prikaza i iz stopa promjene u uzastopnim razdobljima,


odnosno iz verižnih indeksa očito da se cijena udjela uzastopno povećavala do listopada
2007., a od tada se smanjivala, nije uputno računanje prosječne prve diferencije niti
prosječne stope promjene cijene za cijelo razdoblje. Kao što je ranije navedeno, ove
se prosječne mjere temelje na prvoj i posljednjoj frekvenciji niza koje u ovom slučaju
nisu dobar reprezentant dinamike cijena u cjelokupnom razdoblju. Stoga je vremenski
niz podijeljen u dva dijela: od listopada 2006. do listopada 2007. i od studenoga 2007.
do travnja 2008. godine i izračunate su prosječne mjere promjene za svaki dio niza
posebno.

74 Matematičko-fizički list, LIX 2 (2008. – 2009.)


Tako je za prvo razdoblje (do listopada 2007. godine) prosječna prva diferencija:
yn − y1 220.63 − 154.86
ΔyI = = = 5.48,
n−1 13 − 1
a za razdoblje od studenoga 2007. do travnja 2008. to je:
164.85 − 198.92
ΔyII = = −6.82
6−1
Prosječna stopa promjene cijene udjela PBZ Equity fonda za oba razdoblja je:
     
y n 12 220.63
sI = (n−1) − 1 · 100 = − 1 · 100 = 2.99
y1 154.86
 
5 164.85
sII = − 1 · 100 = −3.69
198.92
Cijena udjela u fondu u razdoblju od 25. listopada 2006. do 25. listopada 2007.
povećavala se prosječno mjesečno za 5.48 kuna ili gotovo 3%, a nakon toga u razdoblju
od studenoga 2007. do travnja 2008. se smanjivala prosječno mjesečno za 6.82 kn ili za
3.7%.
Na temelju uočene dinamike cijena udjela u fondu, nije moguće pretpostaviti da
-
će se cijena mijenjati u narednom razdoblju po posljednjoj utvrdenoj prosječnoj stopi,
stoga nije moguće niti prognoziranje njene razine na temelju posljednje cijene iz travnja
i prosječne stope promjene. Prognoziranje je nužno provesti složenijim statističkim
prognostičkim metodama i modelima koji nisu tema ovoga rada.

Zaključak

Strategija ulaganja i izbora instrumenata od kojih će se graditi portfelj Fonda sadrži
nešto veći rizik ulaganja, ali s tim nosi i veće prinose u fondu. Fond je namijenjen
ulagateljima koji žele plasirati dio svoje imovine u domaće i strane dionice, ulagati na
dulji rok te ostvariti veće stope prinosa. Cilj fonda je ostvarivanje relativno velikog
prinosa kroz dulje razdoblje, uz postojanje kratkoročnog rizika. Kontinuirano praćenje i
analiziranje promjena na tržištu kapitala postaje imperativ.
Analiza cijene udjela PBZ Equity fonda u devetnaest mjeseci dala je niz korisnih
informacija. Tendencija rasta cijene udjela u Fondu je sve do listopada 2007. godine bila
rastuća, ali se nakon toga uočava poremećaj i kontinuirano smanjenje cijena koje može
biti jednim dijelom posljedica zasićenosti tržišta fondovima čiji se broj s vremenom
povećava.
Grafička i numerička analiza vremenskog niza osnovnim pokazateljima dinamike ima
široke, ali uvijek treba imati na umu i ograničenje, mogućnosti primjene. Numerički
postupci nisu zahtjevni, a daju korisne informacije o dinamici promatrane pojave, što
čini izvor informacija za odlučivanje.

Literatura

[1] K. BLACK , Business statistics for contemporary decision making, Willey, New York (2006).
[2] S. PIVAC , B. ŠEGO , Statistika – udžbenik i zbirka zadataka, Alka script, Zagreb (2005).
[3] I. ŠO ŠIĆ , Primijenjena statistika, Školska knjiga, Zagreb (2006).
[4] I. ŠO ŠIĆ , Statistika, Školska knjiga, Zagreb (2006).
[5] I. ŠO ŠIĆ , V. SERDAR , Uvod u statistiku, XII. izdanje, Školska knjiga, Zagreb (2002).
[6] www.pbzinvest.hr

Matematičko-fizički list, LIX 2 (2008. – 2009.) 75


Povijest matrica i determinanti

Marko Juraić, Zagreb

Počeci korištenja matrica i determinanti sežu u drugo stoljeće pr. Kr., no neki tragovi
govore da su se koristile čak i u četvrtom stoljeću pr. Kr. Ipak pravi razvoj i korištenje
je započelo tek u 17. st.
Matrice i determinante su nastale proučavanjem sustava linearnih jednadžbi. Babilonci
su proučavali probleme koji su se mogli riješiti korištenjem takvih jednadžbi, a neki od
njih su sačuvani na glinenim pločicama. Jedna takva pločica potječe iz oko 300. godina
pr. Kr. i sadrži sljedeći problem:
Imamo dva polja čija je ukupna površina 1800 m2 . Oba polja su za proizvodnju žita,
2 2 1 2
s tim da jedno daje kg/m , a drugo daje kg/m . Ukoliko je ukupan rod 1100 kg,
3 2
kolika je površina svakog polja?
Kinezi su se, izmedu - 200. i 100. godine pr. Kr., više približili matricama nego
Babilonci. Za vrijeme dinastije Han izraden - je tekst pod nazivom Nine Chapters on
the Mathematical Art (Devet poglavlja matematičkog umijeća) koji je prvi ikad naden -
primjer korištenja matričnih metoda. Prvi problem je vrlo sličan Babilonskom, a glasi
ovako:
Imamo tri vrste žita, od čega tri mjere prvog, dvije drugog i jedna trećeg zajedno
čine 39 jedinica. Dvije mjere prvog, tri drugog i jedan trećeg čine 34 jedinica, a jedna
mjera prvoga, dvije drugoga i tri trećega čine 26 jedinica. Koliko jedinica žita sadrži
jedna mjera za svaki tip žita?
Ovaj problem su rješavali tako da su postavili jednadžbe, čije koeficijente su posložili
u stupce pridružene nepoznanicama:
1 2 3
2 3 2
3 1 1
26 34 39
Današnja metoda rješavanja kaže nam da se koeficijenti pišu u redove, ali to sada nije
bitno. U uputama za rješavanje ovog zadatka navedeno je: pomnoži srednji stupac s 3
i oduzmi desni stupac koliko god puta možeš. Zatim se isto napravi i s prvim stupcem
(pomnoži s 3 i oduzmi se treći stupac). To daje,
0 0 3
4 5 2
8 1 1
39 24 39
Zatim se prvi stupac pomnoži s 5 i od njega se oduzme srednji stupac onoliko puta
koliko je to moguće napraviti. Dobivamo,
0 0 3
0 5 2
36 1 1
99 24 39

76 Matematičko-fizički list, LIX 2 (2008. – 2009.)


Odavde se može izračunati koliko je jedinica za treću vrstu žita, zatim za drugu, a onda
i za prvu (uvrštavanjem). Ova metoda se naziva Gaussova metoda eliminacije i nije bila
poznata sve do početka 19. st.
Godine 1545. Jerome Cardan, u djelu Ars Magna, iznosi pravilo za rješavanje
sustava dviju linearnih jednadžbi koje se naziva majka svih pravila. Ovo pravilo je
istovjetno Cramerovom pravilu za rješavanje (2, 2) sistema, no Cardan nije napravio
završni korak. Nije došao do definicije determinante, ali je bio na pravom putu.
Mnogi rezultati elementarne matrične teorije su se pojavili mnogo prije nego su
matrice postale predmet matematičke analize, tako da su mnoge započete teorije bile
zapravo put ka otkrivanju mnogo kompliciranijih izvoda i rezultata za koje se u to
vrijeme kada su stvorene nije ni pomišljalo.
Ideja o determinanti se gotovo istovremeno pojavila u Japanu i u Europi, ali prvi
ju je objavio Seki Takakazu (poznat kao Seki Kowa). On je 1683. g. napisao djelo
Method of solving the dissimulated problems (Metoda rješavanja skrivenih problema)
koje sadrži metode rješavanja pomoću matrica ispisanih u tablicu i to na isti način kako
su to radili u staroj Kini. Bez da je spomenuo i jednu riječ koja bi mogla predstavljati
“determinantu”, dao je općenite načine izračunavanja determinanti što se baziralo na
primjerima (determinante matrica reda (2, 2), (3, 3), (4, 4) i (5, 5)).
Iste godine se determinanta pojavila i u Europi. Godine 1683. Gottfried Wilhelm
Leibniz je pisao l’Hospitalu i objasnio mu da sustav jednadžbi
10 + 11x + 12y = 0
20 + 21x + 22y = 0
30 + 31x + 32y = 0
ima rješenje jer je 10·21·32+11·22·30+12·20·31 = 10·22·31+11·20·32+12·21·30
što je zapravo uvjet da je determinanta koeficijenata matrice jednaka 0. Leibniz je u
ovome primjeru koristio brojeve kao koeficijente, a oni zapravo predstavljaju znakove
koje mi danas pišemo kao a11 , a12 itd., odnosno prvi od dva indeksa predstavlja u kojoj
se jednadžbi nalazi koeficijent, a drugi označava nepoznanicu.
Leibniz je bio uvjeren da je ključ uspjeha u matematici pravilno označavanje
koeficijenata, pa je on eksperimentirao upravo s nasumičnim odabirom pisanja. U
njegovim neobjavljenim radovima nadeno - je preko 50 različitih načina pisanja sistema
koeficijenata koje je pisao kroz pedesetak godina počevši od 1678. Samo na dva mjesta
(1700. i 1710. godine) dobiven je rezultat sistema i tada je korišten zapis koji je napisao
u svom pismu l’Hospitalu.
Leibniz je za današnju “determinantu” koristio naziv “rezultanta”, i dokazao je
različite rezultate u ovisnosti o determinanti što poznajemo kao Cramerovo pravilo.
Isto tako je znao da se determinanta može izračunati razvojem po bilo kojem stupcu, što
se danas zove Laplaceov razvoj.
Tridesetih godina 18. st. Maclaurin je napisao Treatise of algebra (objavljeno
1748. g.) u kojemu je dokazao Cramerovo pravilo za (2, 2) i (3, 3) sisteme i pokazao
kako bi se mogao rijesiti (4, 4) sustav. Cramer je odredio općenito pravilo za (n, n)
sisteme u djelu Introduction to the analysis of algebraic curves (Uvod u analizu
algebarskih krivulja) (1750. g.), te je u tom djelu izrazio želju za pronalaženjem
-
jednadžbe ravnine koja je zadana odredenim brojem točaka. Pravilo je napisao na kraju
djela, ali ono nije bilo dokazano.
Proučavanje determinanata se sada počelo razvijati.
Godine 1764. Bezout je otkrio metode za računanja determinanti, a isto je učinio
i Vandermonde 1771. g. Laplace je 1772. g. tvrdio da su Cramerove i Bezoutove

Matematičko-fizički list, LIX 2 (2008. – 2009.) 77


metode nepraktične te je napisao diskusije rješenja linearnih jednadžbi bez računanja
sustava, već je samo računao determinante. Iznenadujuće - je da je determinante nazivao
“rezultante”, jednako kao i Leibniz, a zapravo nije bio upoznat s njegovim radom.
Lagrange je 1773. g. proučavao identitete determinanti (3, 3) matrica, ali njegovi svi
-
komentari su bili nagadanja i on nije mogao naći vezu izmedu - svog rada i rada Laplacea
i Vandermondea. No ipak njegovo djelo sadrži izraz za volumen tetraedra zadanog s
točkama O(0, 0, 0), M(x, y, z), M  (x , y , z ), M  (x , y , z ):
1
[z(x y − y x ) + z (yx − xy ) + z (xy − yx )].
6
Izraz “determinanta” je prvi upotrijebio Gauss u djelu Disquisitiones arithmeticae
(1801. g.) dok je opisivao kvadratične forme, a to ime je izabrao zato što determinanta
-
odreduje (eng. determines) svojstva kvadratničkog oblika. No njegov koncept nije
jednak onome koji mi poznajemo. U istom djelu Gauss je ispisao koeficijente svoje
kvadratične forme u oblik pravokutnika. Opisao je množenje matrica (koje je smatrao
kao kompoziciju pa stoga nije dosegao koncept matrične algebre) i izračunavanje inverza
matrice.
“Gaussovu eliminaciju”, koja se prvi put pojavila u djelu Nine Chapters on
Mathematical Art (Devet poglavlja matematičkog umijeća) 200. g. pr. Kr., Gauss je
koristio pri proučavanju putanje asteroida Pallas. Korištenjem tih podataka uzetih izmedu -
1803. i 1809. g., sastavio je sistem od šest linearnih jednadžbi sa šest nepoznanica.
Našao je sistematsku metodu za rješavanje takvih jednadžbi što se upravo i naziva
Gaussova metoda eliminacije.
Augustin Louis Cauchy je 1812. g. prvi koristio determinantu u modernom smislu.
Cauchyjev rad je najkompletniji rad o determinantama ikad uraden. - On je dokazao
prethodne rezultate i dao nove na temelju minora i adjunkta. Godine 1812. dokazao je
teorem o množenju determinanti po prvi put.
Godine 1826. Cauchy je napisao dijagonalizaciju matrice. Isto tako je uveo ideju o
simetričnim matricama i pokazao da one imaju realne svojstvene vrijednosti. Poslije je
dokazao i da se svaka realna simetrična kvadratna matrica može dijagonalizirati.
Zanimljivo je da niti Cauchy niti Sturm nisu našli općenite relacije za ideje koje
su iznijeli, već su ih koristili i vidjeli samo u kontekstu onoga čime su se oni bavili.
Jacobi (1830. g.), a zatim i Kronecker i Weierstrass (1850. i 1860. g.), bavili su
se matričnim rezultatima ali takoder - ne općenito već u smislu linearne transformacije.
Jacobi je objavio tri rada o determinantama 1841. g. koji su bitni zato što je po prvi
puta definicija determinante bila napisana preko algoritma, pa su rezultati bili točni bilo
da se radilo samo o brojevima ili funkcijama. Nakon ovih radova ideja determinanti
postala je poznatom cijelom svijetu.
Cayley je takoder- 1841. g. objavio prvi engleski članak o teoriji determinanti. Na
svom papiru je koristio dvije vertikalne linije na lijevoj i desnoj strani redova čime je
označavao determinante, a takvo označavanje je ostalo i do danas.
Ime “matrica” prvi je koristio Sylvester 1850. g. On je definirao matricu kao pravilnu
skupinu vrijednosti i gledao ju kao nešto što ima ulogu odrediti determinantu vrijednosti
koju sadržava. Nakon povratka iz Amerike u Englesku 1851. g., Sylvester je postao
odvjetnik i upoznao Cayleya, odvjetnika kojeg je matematika takoder - zanimala. Kada je
Cayley vidio važnost matrica, 1853. g. objavio je članak u kojem se po prvi put pojavio
inverz matrice.
Cayley je objavio Memoir on the theory of matrices koja po prvi puta sadrži apstraktnu
definiciju matrice 1858. g. Pokazao je da su sustavi koeficijenata za kvadratične forme i

78 Matematičko-fizički list, LIX 2 (2008. – 2009.)


linearne transformacije zapravo specijalni slučajevi njegovog općenitog koncepta. Cayley
je pokazao definicije zbrajanja, množenja i inverza matrica. Takoder - je dokazao da, u
slučaju (2, 2) matrice, matrica poništava svoj svojstveni polinom. Hamilton je nakon
toga dokazao specijalni slučaj za (4, 4) matrice i stoga se to otkriće danas naziva
Cayley-Hamiltonov teorem.
Godine 1870. Jordan je u djelu Treatise on substitutions and algebraic equations
objavio nešto što se danas naziva “Jordanova forma matrice”. To je dijagonalna
blok-matrica na čijoj se dijagonali nalaze tzv. “Jordanovi blokovi” (kvadratna matrica
specijalnog oblika).
Frobenius je 1878. g. napisao vrlo važan rad On linear substitutions and bilinear
forms (O linearnim supstitucijama i bilinearnim formama) s tim da nije bio upoznat
s Cayleyvim radom. Nakon toga je, pročitavši Cayleya, dokazao Cayley-Hamiltonov
teorem za općeniti slučaj (1896), i izveo definiciju za rang matrice.
Pozitivnost kvadratne matrice je 1884. g. dao Sylvester. Došao je do zaključka da
je kvadratna forma pozitivna ako i samo ako su glavne minore odgovarajuće matrice
pozitivne.
Glavni ljudi koji su počeli ozbiljno promovirati teoriju matrica i determinanti bili
su Weierstrass (objavio je rad On determinant theory (O teoriji determinante)) koji je
objavljen 1903. nakon njegove smrti) i Kronecker (njegovi radovi su takoder - objavljeni
nakon njegove smrti). Teoriju determinanti su matematičari odmah prihvatili, dok je za
teoriju matrica trebalo nešto više vremena. Rad koji je pridonio potpunom shvaćanju
teorije matrica bio je Introduction to higher algebra (Uvod u višu algebru) od Bochera
1907. g. Turnball i Aitken su napisali vrlo utjecajne radove 1930. godine, a nakon rada
Mirskya 1955. (An introduction to linear algebra (Uvod u linearnu algebru)), linearna
algebra je zauzela mjesto jedne od najvažnijih grana matematike.



Kalendar natjecanja u matematici


za učenike osnovnih i srednjih škola 2009. g.

— Školska natjecanja — 29. siječnja


— Županijska natjecanja — 23. veljače
— “Klokan bez granica" — 19. ožujka
— Državno natjecanje — od 29. ožujka do 1. travnja
— Mediteransko matematičko natjecanje — travanj ili svibanj
— Regionalna natjecanja — 8. svibnja
— -
Medunarodna matematička olimpijada — od 10. do 22. srpnja
— Srednjoeuropska matematička olimpijada — od 24. do 29. rujna

Matematičko-fizički list, LIX 2 (2008. – 2009.) 79


S razredbenog ispita u Japanu

Roko Pešić, Zagreb


U jednom američkom časopisu objavljen je sljedeći zadatak koji se pojavio na
razredbenom ispitu na jednom sveučilištu u Tokiju, ali bez rješenja. Zadatak mi se
- a nadam se će i vama biti interesantan.
svidao,

Zadatak. Zadana je pravilna četverostrana


piramida i kugla sa središtem koje leži u bazi
piramide. Ona dira sve njezine bridove, a brid
-
osnovice je duljine a . Nadite:
a) visinu v piramide;
b) obujam V dijela koji je zajednički kugli i
piramidi.
Rješenje. a) Iz uvjeta zadatka se vidi da
središte S kugle mora biti u središtu baze
piramide (baza je kvadrat sa stranicom duljine
a), a to znači da je polumjer kugle jednak
polovici duljine stranice a ,
a
R= . (0)
2 Slika 1.

2 2 2 a 2
Promotrimo pravokutni trokut ASV : |AS| + |SV| = |AV| , gdje je |AS| = ,
2
|SV| = v (visina piramide), |AV| = b (bočni brid piramide), pa je
a2
v2 = b 2 − . (1)
2
- iz uvjeta zadatka slijedi da je visina |SE| trokuta ASV jednaka polumjeru R
Takoder
kugle, |SE| = R. Označimo li |AE| = x , tada je |EV| = b − x .
S obzirom da su trokuti ASV , ASE i ESV pravokutni, vrijede ove jednakosti:
R2 = x(b − x), (2)

 a 2 2 2
a
= bx =⇒ x = . (3)
2 2b
Uvrstivši jednakosti (0) i (3) u (2) dobivamo a = b , i to je piramida kojoj su svi
bridovi jednake duljine (polovica oktaedra). Iz jednakosti (1) tada dobivamo visinu
piramide √
a 2
v= .
2
b) Zajedničkom dijelu obujma pripada gornja polukugla umanjena za četiri kuglina
odsječka koji ne pripadaju piramidi – po jedan nad svakom pobočkom piramide. Ti
-
odsječci su odredeni četirima ravninama u kojima leže pobočke piramide jer njih,
(odsječke) te ravnine odsijecaju od gornje polukugle.
Obujam kuglinog odsječka je
1
Vo = π h(3r2 + h2 ), (4)
6

80 Matematičko-fizički list, LIX 2 (2008. – 2009.)


gdje je r polumjer baze, a h visina kuglinog odsječka. Baza kuglinog odsječka je krug
- kružnicom polumjera r . Ta kružnica dira bočne bridove piramide, a kako je
omeden
ova jednakostranični trokut stranice duljine a , ta kružnica je upisana u jednakostranični
α
trokut pa za nju vrijedi jednakost a = 2r tg , gdje je α središnji kut jednakostraničnog
2
trokuta. Kako je α = 360◦ : 3 tj. α = 120◦ , uvrštavanjem
√ te vrijednosti u prethodnu
jednakost dobivamo da je a = 2r tg 60◦ , tj. a = 2r 3 odnosno
a
r= √ . (5)
2 3

Slika 2. Slika 3. Slika 4.


Za odsječak kugle polumjera R, visine h i polumjera baze r (vidi sliku 3) vrijedi
jednakost:
r2 = h(2R − h), (6)
-
što lako slijedi iz Pitagorinog poučka. Uvrstivši (0) i (5) u (6), nakon sredivanja
a2
dobivamo kvadratnu jednadžbu u varijabli h , h2 − ah + = 0. Njezina rješenja su:
  12
1 1
h=a +√ ≈ 0.9a, (7a)
2 6
1 1
h=a −√ ≈ 0.1a. (7b)
2 6
Rješenje (7a) nije ispravno, jer iz (0) bi slijedilo h > R, što ne može biti. Rješenje
(7b) zadovoljava uvjet h < R, pa je to rješenje ispravno.
Uvrstivši izraze (5) i (7b) u formulu (4) za obujam kuglinog odsječka konačno
dobivamo:  √ 
a3 π 1 7 6
Vo = − . (8)
6 2 36
Zajednički obujam ćemo dobiti ako od obujma polukugle oduzmemo ukupni obujam
četiriju kuglinih odsječaka:
 √ 
V1 2 3 a3 π 1 7 6
V= − 4Vo = R π − 4 · − .
2 3 6 2 36
-
Nakon sredivanja dobivamo zajednički obujam kugle i piramide
 √ 
a3 π 7 6 1
V= − .
2 27 2

Matematičko-fizički list, LIX 2 (2008. – 2009.) 81


Jedno svojstvo logaritamske funkcije i njegova primjena

Petar Vranjković, Zadar

U ovom će članku biti govora o jednom manje poznatom svojstvu logaritamske
funkcije i njegvoj učinkovitoj primjeni na rješavanje nekih logaritamskih nejednadžbi.
To su one u kojima su baza logaritamske funkcije i njezin argument funkcije iste
varijable. Znamo da se takve nejednadžbe rješavaju metodom razlikovanja slučajeva.
Pokazat ćemo da se primjenom tog svojstva spomenute nejednadžbe rješavaju znatno
lakše i jednostavnije.
-
Podimo od definicije logaritamske funkcije. Funkcija f : R+ −→ R odredena -
formulom
f (x) = logb x
je definirana za b > 0 i b = 1 .
Primjer 1. Riješimo nejednadžbu
log1+x (2x − 6) ≥ 0 x ∈ R.
Rješenje. Ovu nejednadžbu ćemo riješiti metodom razlikovanja slučajeva.
1◦ 0<1+x <1 2◦ 1+x >1
2x − 6 > 0 2x − 6 > 0
1 + x = 1 1 + x = 1
2x − 6 ≤ 1 2x − 6 ≥ 1
7
x∈∅ x≥
2
7
Prema tome, rješenje zadane nejednadžbe je x ≥ .
2
No, uočimo sljedeće nejednadžbe (ostale su u 1◦ i 2◦ ):
1+x<1 1+x>1
2x − 6 > 0 2x − 6 > 0
1+x−1<0 1+x−1>0
2x − 6 − 1 ≤ 0 2x − 6 − 1 ≥ 0
(1 + x − 1)(2x − 6 − 1) ≥ 0 (1 + x − 1)(2x − 6 − 1) ≥ 0.
Dakle, u oba slučaja su umnošci istog predznaka. Na tom tragu se pojavila ideja o
“novom” svojstvu logaritamske funkcije.
Poučak. Neka je a > 0 , a = 1 i b > 0 . (1)
Tada vrijedi
loga b ≤ 0 ⇐⇒ (a − 1)(b − 1) ≤ 0 ili loga b  0 ⇐⇒ (a − 1)(b − 1) ≥ 0 (2)
Dokaz. Provedimo dokaz za slučaj loga b ≤ 0.
Ako a i b zadovoljavaju (1) onda imamo:
loga b ≤ 0 ⇐⇒ ((0 < a < 1 ∧ b ≥ 1) ∨ (a > 1 ∧ 0 < b ≤ 1))

82 Matematičko-fizički list, LIX 2 (2008. – 2009.)


⇐⇒ ((a − 1 < 0 ∧ b − 1 ≥ 0) ∨ (a − 1 > 0 ∧ b − 1 ≤ 0))
⇐⇒ (a − 1)(b − 1) ≤ 0.
Analogno se dokazuje ekvivalentnost nejedankosti
loga b ≥ 0 ⇐⇒ (a − 1)(b − 1) ≥ 0.
Vratimo se sada primjeru 1 i riješimo ga pomoću ovog poučka. Prema (1) imamo
1 + x > 0, 2x − 6 > 0, 1 + x = 1.
Rješenje ovog sustava je x > 3 . (3)
Prema (2) imamo
(1 + x − 1)(2x − 6 − 1) ≥ 0,
odnosno x(2x − 7) ≥ 0 tj. 2x2 − 7x ≥ 0 .
7
Rješenje ove kvadratne nejednadžbe je x ≤ 0 ili x ≥ , a zbog (3), konačno rješenje
2
7
je x ≥ .
2
U slučaju kada je ili baza ili argument logaritamske funkcije konstanta, onda je
postupak brži i lakši. Evo jednog primjera.
Primjer 2. Riješimo nejednadžbu
log 1 (4x + 1) < 0, x ∈ R.
3

1
Rješenje. Prema (1) je 4x + 1 > 0 , pa je x > − , a prema (2) imamo
  4
1
− 1 (4x + 1 − 1) < 0,
3
odnosno x > 0 . Konačno rješenje je x > 0 .
Zadatak 1. Ako je a > 0 , b > 0 , c > 0 i a = 1 , onda vrijedi
loga b ≥ loga c ⇐⇒ (a − 1)(b − c) ≥ 0 (4)
loga b ≤ loga c ⇐⇒ (a − 1)(b − c) ≤ 0 (4*)
Dokaz. Dovoljno je dokazati prvu tvrdnju.
loga b ≥ loga c ⇐⇒ loga b − loga c ≥ 0  
b b
⇐⇒ loga ≥ 0 ⇐⇒ (a − 1) − 1 ≥ 0 ⇐⇒ (a − 1)(b − c) ≥ 0.
c c
Pri čemu je korišteno (2).
Evo jednog primjera.
-
Primjer 3. Nadimo rješenje nejednadžbe
log2x (4x − 1) ≥ 2, x ∈ R.
Rješenje. Domenu rješenja ćemo dobiti rješavanjem sustava
2x > 0, 2x = 1, 4x − 1 > 0.
Dobijemo

1 1
x∈ , +∞ , x = . (5)
4 2

Matematičko-fizički list, LIX 2 (2008. – 2009.) 83


Zadanu nejednadžbu ćemo transformirati tako da možemo primijeniti nejednakost
(4). Dobijemo
(2x − 1)(4x − 1 − (2x)2 ) ≥ 0,
-
odnosno, nakon sredivanja (2x − 1)3 ≤ 0 .
1
Odavde imamo 2x − 1 ≤ 0 , tj. x ≤ . Uzimajući u obzir domenu rješenja, konačno

2
1 1
rješenje je x ∈ , .
4 2

Primjer 4. Riješimo nejednadžbu



log|x| x2 − 9 ≥ 1. (6)

Rješenje. Odredimo domenu rješenja:


x = −1, x = 0, x = 1, x2 − 9 > 0.
Ovi uvjeti su zadovoljeni za
x ∈  −∞, −3 ∪  3, +∞ . (7)
Nadalje, (6) možemo zapisati

log|x| x2 − 9 ≥ log|x| |x|.
Prema (4) imamo

(|x| − 1)( x2 − 9 − |x|) ≥ 0. (8)
Dalje, možemo rješavati na standardni način, ali ovdje ćemo koristiti drugu metodu.
U domeni rješenja sigurno vrijedi

x2 − 9 + |x| ≥ 0. (9)
Pomnožimo li (8) s (9), dobivamo
(|x| − 1)(x2 − 9 − x2 ) ≥ 0 tj. |x| − 1 ≤ 0.
Rješenje ove nejednadžbe je −1 ≤ x ≤ 1 , a iz (7) imamo konačno rješenje x ∈ ∅ , tj.
nijedan realan broj x ne zadovoljava danu nejednadžbu.

Zadatak 2. Neka je
a > 0, b > 0, c > 0, d > 0, a = 1, c = 1. (10)
Tada je
1◦ loga b · logc d ≤ 0 ⇐⇒ (a − 1)(b − 1)(c − 1)(d − 1) ≤ 0 ili (11)
loga b · logc d ≥ 0 ⇐⇒ (a − 1)(b − 1)(c − 1)(d − 1) ≥ 0 (11∗)
◦ loga b
2 ≤ 0 ⇐⇒ (a − 1)(b − 1)(c − 1)(d − 1) ≤ 0, d = 1 ili (12)
logc d
loga b
≥ 0 ⇐⇒ (a − 1)(b − 1)(c − 1)(d − 1) ≥ 0, d = 1 (12∗)
logc d

84 Matematičko-fizički list, LIX 2 (2008. – 2009.)


Primjer 5. Riješimo nejednadžbu
logx (1 − x) · log2−x (2x) ≥ 0, x ∈ R. (13)
Rješenje. Prema (10) i (13) dobivamo
x > 0, x = 1, 1 − x > 0, 2 − x > 0, 2 − x = 1, 2x > 0.
Ovi uvjeti su zadovoljeni za
x ∈  0, 1 . (14)
Primjenom (11) nejednadžba (13) je ekvivalentna sljedećoj
(x − 1)(1 − x − 1)(2 − x − 1)(2x − 1) ≥ 0, (15)
tj.
x(1 − x)2 (2x − 1) ≥ 0. (16)
2
Kako je (1 − x) > 0 za sve x ∈  0, 1 , (16) je ekvivalentno s nejednadžbom
x(2x − 1) ≥ 0,


1
čije je rješenje x ∈  −∞, 0] ∪ , +∞ , pa iz (14) dobivamo konačno rješenje
2
1
x∈ ,1 .
2

Zadaci za vježbu

1. log2 (4x) > 1 , Rj. x > 0 .


fi fl
1 4
2. log3x (4x − 1) ≥ log3x (9x2 − 1) , Rj. x ∈ , .
3 9
3. logx 2 · log2 (4x) > 1 , Rj. x > 1 .
fi fl
3 4
4. log2x+4 x2 < 2 , Rj. x ∈ − , − .
2 3
|x| − 2
5. logx+1 < 0, Rj. x > 2 .
|x| + 3
x x
6. log 1 + logx > 0 , Rj. x ∈  0, 1 .
x 2 4
„ « fi fl
1+x 1
7. log 3+x > 0, Rj. x ∈ ,1 .
1−x 2−x 2
8. logcos x sin x · logsin x cos x ≤ 0 , Rj. nema rješenja.
π
9. logsin x 4 · logsin2 x 2 > 0 , Rj. kπ < x < π + kπ , x = + 2kπ , k ∈ Z .
2
log2+x (1 + x)
10. ≥ 0, Rj. x > 1 .
log1+x x

Matematičko-fizički list, LIX 2 (2008. – 2009.) 85


LHC: zašto mu se divimo?

Tome Antičić i Vuko Brigljević 1 , Zagreb

Brzina je važna

Fizika čestica istražuje najmanje dijelove prirode i stanje svemira duboko u prošlosti,
samo nekoliko trenutaka nakon Velikog praska. Jedan od glavnih načina za dobivanje
laboratorijskih uvjeta za stvaranje novih, težih čestica i za ispitivanje ponašanja prirode na
visokim temperaturama je sudaranjem visokoenergetskih čestica. Einsteinova jednadžba
koja povezuje masu i energiju objašnjava nam kako je to moguće:
1
E=  · m · c2 ,
v2
1− 2
c
gdje su m, v i E masa, brzina i energija čestice, a c brzina svjetlosti. S malo
razmišljanja može se doći do zaključka da približavanjem brzine v brzini svjetlosti c,
energija čestice raste prema beskonačnosti. Dakle, s većim brzinama stvaraju se veće
energije, a time i uvjeti za proizvodnju čestica s većom masom i proučavanje fizike na
višim temperaturama.

Slika 1. Shematski prikaz LHC-a

1 Dr. Tome Antičić i dr. Vuko Brigljević su viši znanstveni suradnici u Zavodu za eksperimentalnu fiziku Instituta
- Bošković“ u Zagrebu.
“Ruder

86 Matematičko-fizički list, LIX 2 (2008. – 2009.)


Puno komaraca u Velikom hadronskom sudarivaču

Najmoćnije orude- današnjice za takvo ispitivanje materije je Veliki hadronski sudarivač


(engl. Large Hadron Collider, skraćeno LHC), smješten oko 100 m ispod površine
na švicarsko-francuskoj granici. Izgradnja LHC-a dovršena je upravo ove godine, a
u proljeće 2009. očekuju se sudari na najvišim energijama za koje je konstruiran.
Pridjev “veliki” je posve na mjestu: LHC ubrzava protone (i druge čestice) u kružnom
tunelu s opsegom od čak 27 km, i kao takav je daleko najveći sudarivač na svijetu.
Veliki je i zato što, dijelom i zbog svojih impresivnih dimenzija, stvara najenergetskije
laboratorijske sudare. Naime, kroz LHC mogu kružiti dva snopa protona kroz odvojene
cijevi u suprotnim smjerovima s energijama od čak 7 TeV ( 1 TeV = 1012 eV, a
1 eV = 1.6 · 10−19 J, energija koju jedan elektron dobije kada je na potencijalu od
jednog volta). Izraženo brzinama, svaki proton će se kretati brzinom jednakom čak
0.999999991 brzine svjetlosti. Dakle, svake sekunde će protoni u snopu čak 11 000 puta
obići cijeli prsten LHC-a. Izraženo svakodnevnim primjerom, ta energija je otprilike
jednaka kinetičkoj energiji letećeg komarca. Na prvi pogled to se i ne doima jako
impresivno, ali treba imati na umu da je na LHC-u ta energija koncentrirana u prostor
bilijun puta manji od dimenzije tog istog komarca.

Slika 2. Slikoviti prikaz stvaranja novih čestica (razno voće) iz energije

Još impresivnija je energija po snopu. U svakom od njih je oko 3000 skupina čestica,
koji se sastoje od čak 100 milijardi protona. To znači da će svaki snop (od dva) na
LHC-u imati ukupnu energiju od oko 3.5 · 108 J, ekvivalentno 400 tonskom vlaku koji
putuje brzinom od 150 km/ sat!

Puno energije u Velikom hadronskom sudarivaču

LHC je dizajniran kao sudarivač, što znači da se ubrzane čestice sudaraju jedna u
drugu, za razliku od inžinjerski puno jednostavnijeg i jeftinijeg rješenja usmjeravanja
snopa na nepokretnu metu. Razlog tome je različit iznos količine energije u sustavu
centra mase, to jest korisne količine energije za stvaranje novih čestica. U frontalnom

Matematičko-fizički list, LIX 2 (2008. – 2009.) 87


sudaru čestica iz suprotnih pravaca energija se zbraja, tako da se dobiva 14 TeV-a
na LHC-u za protonske snopove (snopovi teških jezgara, poput olova, imat će i veću
energiju), dok se kod nepokretne mete korisna energija
mijenja kao korijen od energije
udarajuće čestice, to jest za velike energije E ≈ 2 · mproton · Esnop , pa se umjesto
14 TeV-a dobiva tričavih 0.114 TeV-a.

Slika 3. Sličnosti u operaciji televizora i sudarivača

Moćni magneti

Kretanje čestice kroz tunel kontrolira se pomoću dvije vrste magneta: dipola i
kvadrupola. Dipolni magneti imaju, kao što im ime i kaže, dva magnetska pola, i
zaduženi su za držanje čestica na kružnoj putanji. Naime, magnetsko polje ne povećava
brzinu čestice, već je skreće, tako da čestica koja ide pod pravim kutom u odnosu na
polje, putuje kružnom putanjom. Što je veća energija koja se želi postići, to dipolni
magneti (a ima ih 1232 na LHC-u) moraju proizvoditi jače magnetsko polje da bi mogli
čestice zadržati unutar cijevi sudarivača. Za energije koje se žele postići na LHC-u,
magnetsko dipolno polje iznosi čak 8.3 T (tesla), što zahtijeva supravodljive magnete i
-
hladenje pomoću helija u superfluidnom stanju (koji ima iznimno dobru vodljivost za
toplinu, ali je strahovito težak za manipuliranje) na temperaturi od samo 1.9 K (kelvina),
što je hladnije čak i od dalekog Svemira (2.7 K). Ukupno će u svakom trenutku magneti
- s čak 120 tona helija!
biti hladeni
Da je LHC manji, magnetsko polje bi trebalo biti još veće za uspostavu iste energije
snopa, što je i razlog potrebe tunela što većeg promjera:
p ≈ 0.3 · B · R,
gdje je p impuls čestice u GeV/ c2 (1 GeV = 109 eV), B je polje u teslama, a R
promjer u metrima.
Kvadrupolni magneti, s četiri naizmjenična pola, su zaduženi za fokusiranje snopa.
Zbog sprečavanja nepoželjnih sudara, snop se kreće u cijevima u kojima je uspostavljen
vakuum deset puta bolji od vakuuma na Mjesecu (Mjesec nema atmosferu).

88 Matematičko-fizički list, LIX 2 (2008. – 2009.)


Slika 4. Pogled na dipolne magnete unutar LHC-tunela

Gubici energije u Velikom hadronskom sudarivaču

Uz magnete, LHC ima i elektromagnetske komore koje služe najprije za ubrzavanje


čestica i onda za držanje istih na konstatnoj energiji kompenzirajući gubitke energije.
Operacija sudarivača je ustvari na puno načina slična televizoru (onom starinskog tipa s
katodnim cijevima), kao što je objašnjeno na slici 3. Samo nekoliko čestica je prikladno
za ubrzavanje u sudarivačima. One moraju biti i stabilne i nabijene, što izbor svodi
na protone, jezgre težih elemenata ili elektrone. Spomenuti gubitak energije eliminira
elektrone kao čestice prikladne za LHC. Riječ je o takozvanom sinkrotonskom zračenju,
kada nabijena čestica uslijed kretanja u kružnoj putanji gubi energiju emitirajući fotone:
1  Esnop 4
Esinkroton ∞ .
R m
Ovaj gubitak energije je očito iznimno osjetljiv na masu čestice, i za teže čestice kao
što su protoni je puno manji (po jednom krugu rotacije) nego za oko 2000 puta lakše
elektrone. Ako protoni energije 7 TeV na LHC-u gube po krugu 7 · 103 eV, izračunajte
koliko bi elektron izgubio sinkrotonskim zračenjem! Odgovor će vam nedvosmisleno
objasniti zašto se elektroni ne rabe na LHC-u. Izbor dakle ostaje samo na protonima (i
težim ionima) kao jedinim prikladnim česticama.

Slika 5. Prošireni prikaz impresivnog CMS (od engl. Compact Muon Solenoid) detektora.
Sićušna mrlja ispred CMS-a je ustvari prikaz čovjeka u pravoj skali. CMS je napravljen,
izmedu- ostalog, od čak 10 000 tona željeza, što je više nego u Eiffelovom tornju. Detektor je
15 m visok, 21 m dugačak i ima masu od 12 500 tona.

Matematičko-fizički list, LIX 2 (2008. – 2009.) 89


Protoni se klasificiraju kao hadroni, vrsta čestica karakterizirana kvarkovskom strukturom
(vidi prvi okvir). Tako je konačno objašnjen i pridjev “hadronski” u LHC-u. Mana
protonskih sudara je upravo postojanje njihove strukture, zbog čega je samo dio energije
koristan za stvaranje novih čestica. U principu bi se na LHC-u mogli sudarati protoni
-
i antiprotoni, što bi imalo još veću korisnu energiju. Medutim, protone je puno lakše
stvoriti i time je stvoreni broj sudara u sekundi (veličina proporcionalna tzv. luminozitetu)
iznimno veći.

Okvir 1. Standardni model elementarnih čestica


Elementarne čestice (fermioni)
Postoje tri generacije elementarnih čestica. Čestice druge i treće generacije su u svemu slične
onima iz prve generacije osim po tome što su teže. Takoder - su nestabilne i nakon kratkog
vremena se raspadaju u čestice prve generacije. Materijalni svijet oko nas se sastoji isključivo
od čestica prve generacije, dok se one ostalih generacija stvaraju u prirodnim ili umjetnim
visokoenergetskim procesima i brzo raspadaju.
leptoni kvarkovi
2 1
električni −1 0 + − generacija
3 3
naboj
elektron elektronski neutrino gornji (Up) donji (Down) 1
gradi atom -
jako slabo medudjeluje sastavni dio sastavni dio
zajedno s i može proći kroz protona i protona i
jezgrom cijelu zemlju a da neutrona neutrona
se ne zaustavi, zajedno sa
milijarde njih prolaze down kvarkom
kroz nas svake
sekunde; ima jako
malu masu
mion mionski neutrino čarobni strani (Strange) 2
-
teži rodak sličan elektronskom (Charm) -
teži rodak
elektrona neutrinu, stvara se -
teži rodak down
zajedno s mionima up kvarka kvarka
u nekim raspadima
tau tau-neutrino vršni (Top) dubinski (Bottom) 3
još teži, otkriven tek 2000. najteža od - down
još teži rodak
jako godine kao svih poznatih kvarka, važan za
nestabilan zadnji od svih čestica, težak proučavanje asimetrije
fermiona kao atom materija-antimaterija
zlata
Kvarkovi se nikada ne nalaze sami. Uvijek su vezani u hadrone. Jedini stabilni hadroni su
proton i neutron (i neutron zapravo samo ukoliko se nalazi u atomskoj jezgri, izvan nje se
raspada nakon nekoliko minuta). Proton se sastoji od dva up-kvarka i jednog down-kvarka, a
neutron od dva down-kvarka i jednog up-kvarka.
Svi leptoni i kvarkovi su fermioni: njihov spin, to jest intrinsična kutna količina gibanja, je
višekratnik Planckove konstante podijeljene s 2. Fermioni zadovoljavaju Paulijev princip: dva
identična fermiona ne mogu se nalaziti istodobno u istom stanju. Taj zahtjev je neophodan
uvjet za bogatstvo struktura koje vidimo u svijetu, od atomske jezgre do kompleksnih DNK
molekula života.
Za svaki fermion, kvark ili lepton, postoji i odgovarajuća antičestica, jednake mase, jednakog
trajanja života i spina, ali sa suprotnim električnim nabojem. Tijekom velikog praska nastajale
-
su jednake količine čestica i antičestica. Medutim, antičestice su potpuno nestale iz našeg
-
svemira, a razlog za to još nije utvrden.

90 Matematičko-fizički list, LIX 2 (2008. – 2009.)


Divovski detektori za otkrivanje mikrosvijeta

Na više mjesta na LHC-tunelu protoni iz suprotnih pravaca se fokusiraju unutar


desetak mikrona i sudaraju. Oko tih lokacija izgradeni - su masivni detektorski sustavi
koji detaljno analiziraju stvorene čestice i iz toga pokušavaju razumjeti fiziku koja
se pri sudaru dogodila. Detektori su koncipirani slojevito: čestice koje proizlaze iz
točke sudara prolaze kroz razne slojeve. Svaki od njih mjeri specifičnu karakteristiku
prolazećih čestica: količinu gibanja, električni naboj, energiju ili brzinu. Razne vrste
čestica će ostaviti različite “potpise” u detektoru. Dok će neke kao elektroni ili fotoni biti
zaustavljene već u prvim slojevima detektora, druge će, poput miona, proći kroz gotovo
cijeli detektor i jedini ostaviti trag u golemim mionskim komorama u vanjskom sloju
detektora (vidi sliku 6). Svaki od tih divovskih eksperimenta je rezultat kolaboracije
više tisuća znanstvenika i tehničara.

Slika 6. Presjek CMS detektora s raznim slojevima detektora.


Prikazan je i odziv detektora na razne vrste čestica.

Čarobne riječi: Higgs, dodatne dimenzije, supersimetrija, tamna tvar,. . .

LHC je stvoren jer naše znanje Svemira nije konačno. Tzv. Standardni model čestica
i sila ujedinjuje naše trenutno znanje fizike čestica, vidi okvir 1 i 2. Standardni model
je testiran na brojnim eksperimentima i bio je iznimno uspješan u predvidanju - novih
-
čestica i njihovih svojstava. Medutim, puno neriješenih problema ostaje, i LHC je
upravo dizajniran da postigne energiju gdje se neki odgovori moraju pojaviti. Prvo
veliko otvoreno pitanje je pitanje porijekla mase, koje je od ključne važnosti za
samu konzistentnost standardnog modela. U verziji u kojoj postoje samo već poznate
čestice, masa svih čestica bi treba biti jednaka nuli, što očito ne odgovara činjenicama.
Standardni model se može nadograditi uvodenjem- Higgsovog polja, s čijom interakcijom
čestice dobivaju masu (jeste li se kada upitali: zašto tijela imaju masu?). To polje
podrazumijeva i najmanje jednu još neotkrivenu česticu, tzv. Higgsov bozon. Teorija
predvida- sve njegove karakteristike osim jedne, njegove mase. Medutim, - iz ranijih
mjerenja i teorijskih razloga znamo da mu masa mora biti veća od otprilike 120 i manja
od 200 vodikovih atoma. Ako postoji i ako mu je masa u tom području, Higgsov bozon
će nastati u proton proton sudarima na LHC-u i eksperimenti će ga otkriti. Otkriće
Higgosvog bozona je jedan od prvenstvenih ciljeva dva najveća LHC-eksperimenta,
CMS-a i ATLAS-a.

Matematičko-fizički list, LIX 2 (2008. – 2009.) 91


-
Medutim, otkriće Higgsovog bozona ne može biti kraj priče. Naime, standardni
model čak i s Higgsovim poljem ne daje kompletnu sliku svih fundamentalnih sila, te
sadrži i puno proizvoljnih parametara, i ostavlja niz otvorenih pitanja: zašto se priroda
ponavlja po tri puta (vidi okvir 1)? Krije li se dublji princip reda iza tri generacija
elementarnih čestica? Zašto su mase elementarnih čestica toliko različite jedna od druge
(top kvark je više od 300 tisuća puta teži od elektrona)? Kako uključiti gravitaciju u
sliku? Jedna moguća nadogradnja tog modela je tzv. teorija supersimetrije, koja postulira
temeljnu simetriju izmedu - bozona i fermiona, i koja predvida - da svaka postojeća čestica
ima i svog “superpartnera”. Ako je ta teorija točna, LHC će najvjerojatnije naći najlakšu
supersimetričnu česticu, koja je ujedno i glavni kandidat za objašnjenje tamne tvari, tj.
23% gustoće energije svemira koja ne potječe od nama poznatih čestica (odgovornih za
tek 4%). Postoje još egzotičnije stvari koje će možda biti otkrivene, kao što su dodatne
dimenzije prostora, ili odstupanje zakona gravitacije od 1/r2 ovisnosti na jako malim
skalama, što bi moglo dovesti do jako zanimljivog fenomena produkcije mini crnih rupa
na LHC-u.
Okvir 2. Standardni Model elementarnih čestica
Elementarne sile (bozoni)
-
Sile su odgovorne za medudjelovanje - elementarnih čestica. Bez njih ne bi bilo
izmedu
strukturirane tvari, reakcija, raspada. Danas poznajemo četiri osnovne sile. Sile se prenose
putem izmjene čestica nosioca sila.
elektromagnetska slaba jaka gravitacija
osjetljive čestice kvarkovi i nabijeni svi kvarkovi kvarkovi sve čestice
leptoni i leptoni s masom
vezane pojave drži elektrone oko uzrok veže kvarkove u prva poznata
atomskih jezgri, radioaktivnosti protonu, neutronu i sila, ali na
spaja atome u drugim hadronima, mikroskali
molekule, i drži atomsku jezgru još uvijek
zaslužna je na okupu i pri tome najlošije
za svojstva je jača od ogromne shvaćena;
plinova, električne odbojne ne uklapa se
tekućina, i sile izmedu- u teorijski
krutih tvari protona opis standardnog
modela
nositelj sile foton W + , W − , i Z 8 gluona graviton? (postulat)
Elektromagnetska i slaba sila su prepoznate kao različite manifestacije jednog istog principa
i sjedinjene u elektroslabu silu. U okviru standardnog modela opisane su elektroslaba i jaka
sila. Uključenje gravitacije u tu sliku ostaje jedan od velikih izazova fizike na početku 21.
stoljeća.
Nosioci sila su bozoni, što znači da je njihov spin (vidi okvir 1) cjelobrojan (0,1,2,. . . )
višekratnik Planckove konstante.

Potraga za novom fizikom: traženje igle u puno plastova sijena

Kako će fizičari naći Higgsov bozon, supersimetrične čestice ili neki drugi znak nove
fizike u LHC sudarima? Kao prvo treba napomenuti da je riječ o vrlo rijetkim procesima
koje će fizičari trebati izdvojiti iz ogromne količine podataka. U srcu CMS i ATLAS
detektora će se svakih 25 nanosekundi križati snopovi protona i u svakom križanju će
doći do nekoliko proton-proton sudara koji će ostaviti tragove u detektoru, dakle 40

92 Matematičko-fizički list, LIX 2 (2008. – 2009.)


milijuna puta svake sekunde. Očekuje se da će stopa proizvodnje Higgsovog bozona,
ovisno o teorijskom modelu, biti reda veličine jedan na dan (po svakom eksperimentu),
-
što znači da se traži jedan od 10 13 dogadaja! Traženje igle u plastu sijena u usporedbi
izgleda dječjom igrom: ako je volumen tipične iglice 5 mm 3 , a volumen plasta sijena 50
m 3 , traženje nove fizike u LHC sudarima odgovara traženju jedne igle u tisuću plastova
sijena! Pri tome je prvi izazov uopće sačuvati podatke zanimljivih sudara. Svaki sudar
će u detektoru generirati otprilike 1 MB informacije, a 40 milijuna njih 40 TB svake
sekunde! Takvu količinu podataka nije moguće pohraniti i od njih će se samo 100 do 200
najzanimljivijih sačuvati za daljnju analizu. Odbačeni dogadaji - su zauvijek izgubljeni i
stoga fizičari moraju biti sasvim sigurni da smo sačuvali dogadaje - koji sadrže tragove
Higgsovog bozona ili drugih oznaka još nepoznate fizike. Za izbor najzanimljivijih
- je odgovoran veliki računalni grozd sastavljen od nekoliko tisuća računala koji
dogadaja
u nekoliko milisekundi mora analizirati svaki sudar tražeći oznake zanimljivih dogadaja. -
Tu fizičarima zakoni fizike ipak dolaze u pomoć i bitno im olakšavaju posao. U većini
proton-proton sudara će se dva protona tek okrznuti i samo lagano raspršiti, te će
produkti tih sudara uglavnom letjeti jako blizu osi samog snopa. S druge strane, ako se
u sudaru stvori teži objekt poput Higgsovog bozona ili supersimetrične čestice, od kojih
znamo iz rezultata ranijih eksperimenata da ukoliko postoje moraju biti teži od otprilike
100 atoma vodika, raspasti će se u puno lakše vidljive ili nevidljive čestice. Iz već
spomenute relacije izmedu - mase i energije slijedi da će se masa teškog objekta u raspadu
pretvoriti u veliku količinu gibanja laganih produkata raspada. A zato jer je u principu
svaki pravac leta produkata raspada moguć, iz takvih dogadaja - se očekuju često čestice
s velikim kutom raspršenja prema pravcu snopa, ili velikom transverzalnom količinom
gibanja u odnosu na pravac snopa. Taj jednostavan argument je ključ izbora zanimljivih
-
dogadaja: traže se sudari s česticama velike transverzalne energije ili količine gibanja.

Slika 7. Unutrašnja struktura protona: proton je sastavljen od dva donja i jednog gornjeg
kvarka. Ali unutar protona možemo naći i gluone koji vežu kvarkove, a i dodatne parove
kvark-antikvark koji se mogu na trenutak stvoriti i ponovo nestati.

Kad su podaci sudara jednom snimljeni, postaju objekt duge i često komplicirane
analize fizičara koji će krenuti u potragu za oznakama nove fizike. Tražene nove čestice,
npr. Higgsov bozon, se obično neće mjeriti direktno u detektoru, jer su jako nestabilne.
Raspasti će se vrlo brzo nakon stvaranja i produkti njihovog raspada, obično već poznate
čestice, su oni koji će ostaviti vidljiv trag u raznim slojevima detektora. Ako je npr.
masa Higgsovog bozona jednaka masi 150 do 200 atoma vodika, često će se raspasti
u 2 elektrona i 2 miona. Elektroni i mioni (mion je teži rodak - elektrona,vidi okvir 1)
ostavljaju karakterističan trag u detektoru (vidi sliku 6). Iz mjerenja njihovih impulsa
može se jednostavno odrediti masa Higgsovog bozona. Pri tome se koristi samo zakon
očuvanja energije i impulsa i relativistička relacija izmedu- mase, energije i impulsa
2 2 2 2 4
jedne čestice: E = p c + m c (ovo je jednadžba s početka članka u drugačijem
obliku). Mala komplikacija je takozvana pozadina: bit će puno drugih dogadaja - s 2
elektrona i 2 miona u konačnom stanju, zapravo će ih obično biti za nekoliko redova
veličine više od dogadaja- s Higgsovim bozonom (signal). Tu se otvara prostor za
maštu i kreativnost fizičara koji će kroz lukavi odabir zahtjeva uspjeti odbaciti skoro
sve pozadinske dogadaje - i sačuvati one s Higgs-signalom. Katkad će se za tu proceduru

Matematičko-fizički list, LIX 2 (2008. – 2009.) 93


koristiti složenim alatima poput neuronskih mreža. Nova fizika se može otkriti iako se
stvaraju nove čestice koje nevidljivo pobjegnu iz detektora bez da ostave ikakav trag,
ali mogu sa sobom nositi značajnu količinu energije i količinu gibanja. To bi npr. bio
slučaj za česticu tamne tvari, koja samo vrlo slabo djeluje s normalnom materijom.
Koristeći se ponovo zakonima očuvanja količine gibanja i energije i činjenicom da
detektor potpuno okružuje točku sudara može se izmjeriti sveukupni impuls i energiju i
primijetiti da nešto nedostaje. Iz ostatka detektora može se odrediti energija, a možda i
masa i druge osobine “bjegunca”. Možemo dakle otkriti i nešto što uopće nismo vidjeli,
zakoni očuvanja su stvarno važni u fizici!
Konačno je vrijedno spomenuti još jednu karakteristiku sudara proton-proton koja
dodatno otežava mjerenja na LHC-u: okruženje tih sudara je jako prljavo! Da bismo
razumjeli o čemu je riječ moramo se sjetiti da proton nije jednostavan i elementaran
objekt. Sastoji se od tri kvarka, ali zapravo je i složeniji od toga (vidi okvir 1). U
protonu možemo naći kvarkove, ali i gluone, pa čak i antikvarkove. Kad se u sudaru
proton-proton stvara Higgsov bozon, ili par top-kvarkova (za njih znamo sigurno da
postoje!), to je produkt elementarne reakcije izmedu - dvije elementarne čestice (kvark,
antikvark ili gluon) iz dva protona. Ali tu su i ostali sastavni dijelovi protona, ostali
kvarkovi i gluoni, koji nisu sudjelovali u “glavnom” procesu, u kojem igraju na jedan
način samo ulogu gledatelja. Nakon tog glavnog procesa, i kvarkovi-gledatelji će letjeti
dalje i zakomplicirati konačno stanje s dodatnim česticama. Nalaženje tragova iz raspada
neke nove čestice medu - svim tim česticama (njih često više od 100) predstavlja dodatni
izazov u fascinantnom intelektualnom pothvatu potrage za novom fizikom
Osim sudara proton-proton, sudarima iona olova LHC će takoder - proučavati i stanje
materije koja je postojala nekoliko mikrosekundi nakon velikog praska, tzv. kvarkovsko-
gluonske plazme. U tom stanju kvarkovi i gluoni su se slobodno kretali u iznimno vrućoj
i gustoj juhi, za razliku od danas kada ih se samo može naći u česticama kao što su
protoni ili neutroni. U sudarima olovo-olovo stvorena temperatura bit će više od 100 000
puta veća nego ona u središtu Sunca. Eksperiment optimiziran za ova istraživanja je
ALICE. Pomoću četvrtog eksperimenta na LHC-u koji se zove LHC-b, pokušat će se
odgonetnuti tajne asimetrije tvari u svemiru, to jest naći odgovor na pitanje zašto nema
jednake količine materije i antimaterije u svemiru.
Eksperimenti na CERN-u uključuju i po više tisuća znanstvenika iz cijelog svijeta
(većinom fizičara) koji će imati pune ruke posla sljedećih 10 do 20 godina da pokušaju
iz dobivenih podataka odgonetnuti što više tajni Svemira. I desetak hrvatskih fizičara
je uključeno u taj pothvat. Možete ih kontaktirati ako možda i vi želite biti dio ovog
najvećeg znanstvenog projekta u povijesti. . .



Nadopuna. U članku J. Šiftara Abelova nagrada 2008. g. – John G. Thompson i


Jacques Tits u br. 1/ 233 ispuštene su slike dobitnika nagrade, pa ih sada donosimo.

John G. Thompson Jacques Tits

94 Matematičko-fizički list, LIX 2 (2008. – 2009.)


Geometrija i problemi kretanja

Ljiljana Sudar, Leskovac


Neki fizički problemi mogu se riješiti na više načina, primjenom različitih područja
matematike. Problemi kretanja se, npr. najčešće rješavaju algebarski. Taj način rješavanja
zahtijeva za dano kretanje poznavanje eksplicitne ovisnosti brzine i prijedenog - puta od
vremena.
-
Medutim, i bez poznavanja te eksplicitne ovisnosti moguće je i efikasnije i elegantnije
riješiti mnoge probleme kretanja pomoću nestandardne, veoma moćne metode, koja se
bazira na primjeni grafova i geometrije.
Ukoliko se pri kretanju intenzitet brzine za isti vremenski interval uvijek mijenja
za istu vrijednost (tj. ubrzanje je konstantno), samo na osnovu grafova i elementarnog
poznavanja geometrije mogu se postaviti relacije iz kojih se dobiva rješenje problema.

Slika 1. Grafovi ovisnosti brzine o vremenu

Ako se intenzitet brzine za svaki mali vremenski interval Δt uvijek povećava za istu
vrijednost Δv (tj. ubrzanje je konstantno), graf brzine je pravac koji zaklapa oštar kut
Δv v2 − v1
α s pozitivnim dijelom t -osi (sl. 1a). S grafa se vidi: tg α = = = a,
Δt t2 − t1
gdje je a intenzitet ubrzanja. Ako se kroz točku K koja je središte dužine DC
povuče pravac paralelan t -osi, pravokutni trokuti DD1 K i KC1 C su identični, pa
imaju jednake površine. Zato će osjenčana površina trapeza ABCD (u oznaci P) biti
jednaka površini pravokutnika ABC1 D1 (PABC1 D1 ) osnovice Δt = t2 − t1 i visine v tj.
-
P = PABC1D1 = v Δt , pa kako je v Δt elementarni prijedeni put ΔS za vrijeme Δt , bit
će P = ΔS tj. površina ispod odgovarajućeg dijela grafa brzine predstavlja elementarni
- put.
prijedeni
Ako se intenzitet brzine za svaki mali vremenski interval Δt uvijek smanjuje za
istu vrijednost Δv (tj. ubrzanje je konstantno), graf brzine je pravac koji zatvara
tupi kut α s pozitivnim dijelom t -osi (sl. 1b). S grafa je očigledno α = π − β i
Δv
tg α = tg(π − β ) = − tg β = − = −a , gdje je a intenzitet ubrzanja, pa slijedi
Δt
tg β = a , gdje je β kut koji pravac zatvara s negativnim dijelom t -osi. Analogno
-
se, kao u prethodnom slučaju, može pokazati da je elementarni prijedeni put tijela za
vrijeme Δt (ΔS) osjenčana površina trapeza ABCD (P) s osnovicama AD i BC i
|AD| + |BC|
visinom Δt = t2 − t1 , tj. ΔS = P = Δt .
2

Matematičko-fizički list, LIX 2 (2008. – 2009.) 95


Ako se intenzitet brzine tijekom vremena ne mijenja, pravac je paralelan s t -osi
-
(sl. 1c). Prijedeni put tijela za vrijeme Δt (u oznaci ΔS) je, očigledno, osjenčana
površina pravokutnika ABCD (P) osnovice Δt i visine v, tj. ΔS = P = Δt · v.
-
Zbroj elementarnih prijedenih -
putova je ukupan prijedeni put tijela i predstavlja
površinu ispod odgovarajućeg dijela grafa brzine.
Metoda rješavanja problema kretanja pomoću grafa, primjenom geometrije zahtijeva,
osim dobrog razumijevanja grafičkog prikaza problema kretanja, i to imajući u vidu:
- put tijela i
1) da je površina ispod grafa brzine prijedeni
2) da je nagib pravca, kojom se prikazuje ovisnost brzine tijela o vremenu, definiran
ubrzanjem tijela koje se kreće.
Slijede primjeri koji ilustriraju ovu metodu.

Primjer 1. Prvu polovinu puta tijelo prijede- za dvostruko dulje vrijeme nego drugu
polovinu. Srednja brzina duž cijelog puta je 6 m/ s. Kolika je srednja brzina na prvoj, a
kolika na drugoj polovini puta? [2]
Rješenje.
Dan je graf brzine tijela u ovisnosti o vreme-
s
nu. Drugu polovinu puta s2 = tijelo prelazi
2
brzinom v2 za vrijeme t . Prvu polovinu puta
s
s1 = tijelo prelazi brzinom v1 za vrijeme 2t
2
(tj. dva puta dulje vrijeme nego drugu polovinu
puta), pa je ukupno vrijeme kretanja tijela 3t i
čitav put
s = s1 + s2 . (1)
Put s1 jednak je površini pravokutnika OABC ,
osnovice |OA| = 2t i visine |OC| = v1 , (u oznaci POABC ), tj.
s
s1 = = POABC = v1 · 2t. (2)
2
Put s2 jednak je površini pravokutnika ADEF , osnovice |AD| = t i visine |AF| = v2 ,
(u oznaci PADEF ), tj.
s
s2 = = PADEF = v2 · t. (3)
2
Čitav put s tijelo bi prešlo i da se sve vrijeme (3t) kretalo srednjom brzinom vsr = 6
- put s u tom slučaju bio jednak površini
(u m/ s), pa je s grafa očigledno da bi prijedeni
pravokutnika ODGH , osnovice |OD| = 3t i visine OH = vsr , (u oznaci PODGH ), tj.
s = PODGH = vsr · 3t = 18t. (4)
Iz relacija (2) i (3) dobivamo
2v1 = v2 . (5)
Zamjena (2), (3) i (4) u (1), uz uvjet (5) daje v1 = 4.5 i v2 = 9 (u m/ s).

Primjer 2. Ivan i Luka hodaju s istog mjesta u suprotnim smjerovima oko nogometnog
igrališta koje je dugačko 110 m, a široko 70 m. Luka hoda brzinom 5 m/ s, a Ivan
brzinom 4 m/ s. Koliko metara će svaki od njih prevaliti kad se prvi put sretnu? Nakon
koliko vremena će to biti? [3]

96 Matematičko-fizički list, LIX 2 (2008. – 2009.)


Rješenje.

Na sl. a) su dani grafovi brzina Luke (L) i Ivana (I) u ovisnosti o vremenu.
Do trenutka (t) kad se prvi put sretnu Luka je prešao put (sL ) jednak površini
pravokutnika OACD, osnovice |OA| = t i visine |OD| = 5 (u m/ s), (u oznaci POACD ),
pa je
sL = POACD = 5t. (1)
Ivan je do prvog susreta prešao put jednak površini pravokutnika OABE , osnovice
|OA| = t i visine |OE| = 4 (u m/ s), (u oznaci POABE ), pa je
sI = POABE = 4t. (2)
-
U trenutku susreta (t) je zbroj prijedenih putova Luke i Ivana jednak opsegu igrališta
(O) dužine a = 110 m i širine b = 70 m (vidi sl. b), tj.
sL + sI = O. (3)
Kako je opseg igrališta O = 2a + 2b = 360 (u m), kao i relacija (1), (2) i (3) dobiva
se: 9t = 360 , tj. t = 40 (u s) što znači da je vrijeme prvog susreta Luke i Ivana 40 s
nakon istovremenog polaska s istog mjesta (na sl. b) to mjesto je vrh M pravokutnika;
Luka iz njega kreće oko nogometnog igrališta u smjeru kretanja kazaljki na satu, a Ivan
u suprotnom smjeru, na što ukazuju strelice). Na osnovu relacije (1), Luka je do prvog
susreta prešao put sL = 5t = 200 (u m), a na osnovu relacije (2) Ivan je prešao put
sI = 4t = 160 (u m).
Primjer 3. Padobranac skače iz aviona te nakon 3 s otvara padobran. Nakon otvaranja
padobrana on naglo (trenutno) usporava te nastavlja padati brzinom od 5.4 m/ s. Deset
sekundi nakon njegovog skakanja iz aviona skače drugi padobranac. Nakon koliko
vremena on mora otvoriti padobran da bi zajedno stigli do tla? Pretpostavite da je otpor
zraka prije otvaranja padobrana zanemariv. [3]
Rješenje.

Grafovi I i II daju ovisnost brzina od vremena oba padobranca. Nulti trenutak


vremena je onaj kada skače prvi padobranac. Prve tri sekunde on slobodno pada tj.
kreće se s ubrzanjem a = g , a zatim konstantnom brzinom 5.4 (u m/ s). Nakon deset
sekundi iskače i drugi. On slobodno pada tj. kreće se s ubrzanjem a = g sve do trenutka

Matematičko-fizički list, LIX 2 (2008. – 2009.) 97


t kada sustiže prvog padobranca, otvara padobran pa njegova brzina trenutno postaje 5.4
(u m/ s) (vidi graf ). Od trenutka t oba padobranca se kreću brzinom 5.4 (u m/ s) sve
dok istovremeno ne stignu do tla.
Zbog istog ubrzanja padobranaca dok slobodno padaju (a1 = a2 = g), nagib dužina
OB i CE prema pozitivnom dijelu t -osi je isti, pa iz pravokutnih trokuta ΔOAB i
ΔCDE imamo
v1 v2
= = g. (1)
3 t − 10
- put prvog padobranca, s1 , je u trenutku sustizanja (t) jednak zbroju površina
Prijedeni
trokuta ΔOAB, osnovice |OA| = 3 (u s) i visine |AB| = v1 , i pravokutnika ADFG,
osnovice |AD| = t − 3 i visine |DF| = 5.4 (u m/ s), tj.
3v1
s1 = + 5.4(t − 3). (2)
2
- put drugog padobranca, s2 , je u trenutku sustizanja (t) jednak površini trokuta
Prijedeni
ΔCDE , osnovice |CD| = t − 10 i visine |DE| = v2 , tj.
v2 (t − 10)
s2 = . (3)
2
- putovi padobranaca su jednaki u trenutku sustizanja (t) tj.
Prijedeni
s1 = s2 (4)
pa je prema relacijama (2) i (3):
3v1 v2 (t − 10)
+ 5.4(t − 3) = . (5)
2 2
Zamjenom relacije (1) u (5) dolazi se do kvadratne jednadžbe:
 10.8 30.24
t2 − t 20 + + 91 + = 0.
g g
Ako se uzme g = 9.81 (u m/ s2 ), kvadratna jednadžbe glasi:
t2 − 21.1t + 94.08 = 0.
Njena rešenja su: t1 = 14.7 i t2 = 6.4 (u s). Drugi padobranac otvara padobran nakon
t − 10 sekundi (vidi sliku), pa je očigledno da fizički smisao ima samo drugo rješenje
za koje je t − 10 = 4.7 (u s).
Primjer 4. Kuriri iz mjesta A i B kreću jedan drugome u susret, pri čemu se svaki
kreće jednoliko, ali različitom brzinom u odnosu na onog drugog. Pošto su se sreli, da
bi stigli u nasuprotna mjesta, jednome je potrebno još 16, a drugome 9 sati. Koliko
je potrebno vremena jednom, a koliko drugom, da prijedu - čitav put izmedu
- A i B?
(Zadatak Lewisa Carrolla, engleskog matematičara i pisca knjige za djecu “Alisa u
zemlji čudesa”.) [4]
Rješenje.

98 Matematičko-fizički list, LIX 2 (2008. – 2009.)


Dan je graf brzina oba kurira u ovisnosti o vremenu. Ne smanjujući općenitost
rješavanja možemo pretpostaviti da je brzina vA kurira iz mjesta A veća od brzine vB
kurira iz mjesta B tj. vA > vB . Nulti trenutak vremena je onaj kada se kuriri počinju
kretati jedan drugome ususret, a t je trenutak njihovog susreta. U trenutku susreta (t)
-
zbroj prijedenih putova kurira jednak je udaljenosti d mjesta A i B (vidi sl. b)) tj.
sA + sB = d. (1)
Kurir iz A je do susreta prešao put sA , jednak površini pravokutnika OKMN ,
osnovice |OK| = t i visine |ON| = vA , (u oznaci POKMN ), pa je
sA = POKMN = vA t. (2)
Kurir iz B je do susreta prešao put sB , jednak površini pravokutnika OKLP , osnovice
|OK| = t i visine |OP| = vB , (u oznaci POKLP ), pa je
sB = POKLP = vB t. (3)
Zamjenom relacija (2) i (3) u (1) dobiva se
vA t + vB t = d. (4)
Kako je vA > vB kurir iz A je manje udaljen od mjesta B nego kurir iz B od mjesta A,
pa će po uvjetu zadatka njemu trebati 9 sati da stigne u B, dok će kuriru iz B trebati
16 sati da stigne u A. S grafa je očigledno da kurir iz A stiže u B u trenutku t + 9 ,
prešavši put jednak udaljenosti d mjesta A od mjesta B, što na grafu odgovara površini
pravokutnika OQRN, osnovice |OQ| = t + 9 i visine |ON| = vA , (u oznaci POQRN ), tj.
d = POQRN = vA (t + 9). (5)
Kurir iz B stiže u A u trenutku t + 16 prešavši put jednak udaljenosti d mjesta B od
mjesta A, što na grafu odgovara površini pravokutnika OSTP , osnovice |OS| = t + 16
i visine |OP| = vB , (u oznaci POSTP ) tj.
d = POSTP = vB (t + 16). (6)
d d
Iz relacija (5) i (6) možemo izraziti brzine: vA = i vB = .
t+9 t + 16
Njihovom zamjenom u relaciju (4) dolazi se do kvadratne jednadžbe:
d d
t+ t = d tj. t2 = 144 odnosno t = 12 (u h).
t+9 t + 16
Kurir iz A krećući se brzinom vA preći će čitav put od A do B za vrijeme:
t + 9 = 21 (u h)
a kurir iz B krećući se brzinom vB preći će čitav put od B do A za vrijeme:
t + 16 = 28 (u h).

Primjer 5. Tijelo je bačeno brzinom 20 m/ s vertikalno uvis. Na visini 15 m nalazi


se horizontalna ploča od koje se tijelo savršeno elastično odbije. Nakon koliko vremena
(od trenutka izbačaja) se tijelo vrati natrag? (g = 9.81 ms−2 ). [5]
Rješenje: Graf predstavlja ovisnost brzine
tijela od vremena. Nulti trenutak vremena je
trenutak izbačaja tijela vertikalno uvis. Brzina
tijela se tokom kretanja naviše smanjuje i u
trenutku t1 , kada tijelo ima brzinu v1 , udara u
horizontalnu ploču. Prema uvjetu zadatka udar
tijela o ploču, koja je na udaljenosti d = 15 m
od mjesta izbačaja tijela, savršeno je elastičan

Matematičko-fizički list, LIX 2 (2008. – 2009.) 99


pa brzina v1 mijenja samo smjer dok njezin intenzitet ostaje isti. Tijelo se nakon udara
o ploču kreće naniže i u trenutku t2 vraća na mjesto izbačaja. Prijedeni- put tijela
vertikalno uvis do udara u ploču (s1 ) jednak je udaljenosti ploče od mjesta izbačaja (d)
tj. s1 = d = 15 (u m).
Na grafu tom putu odgovara površina pravokutnog trapeza OABC , s osnovicama
|OC| = 20 (u m/ s) i |AB| = v1 i visinom |OA| = t1 , (u oznaci POABC ), pa je
20 + v1
s1 = POABC = t1 = 15 (u m). (1)
2
-
Na grafu prijedenog puta tijela vertikalno dolje do mjesta izbačaja (s2 ) odgovara
površina pravokutnog trapeza ADEB, s osnovicama |AB| = v1 i |DE| = v2 i visinom
|AD| = t2 − t1 = Δt , (u oznaci PADEB ), pa je
v2 + v1
s2 = PADEB = Δt = 15 (u m). (2)
2
2
Ubrzanje tijela prema gore i prema dolje je gravitacijsko (g = 9.81 m/ s ) i odreduje -
nagib dužine BC prema negativnom dijelu t -osi i nagib dužine BE prema t -osi (vidi
sliku) pa je: FBC = KBE i vrijedi
20 − v1 v2 − v1
g= = . (3)
t1 Δt
Ako oko dužine AB rotiramo pravokutni trapez ADEB za 180◦ , on će prijeći u osjenčani
trapez AD1 E1 B s osnovicama |AB| = v1 i |D1 E1 | = v2 i visinom |AD1 | = |AD| = Δt
i imat će, naravno, istu površinu kao i trapez ADEB. Sa slike je očigledno razlika
površina pravokutnog trapeza OABC i osjenčanog trapeza AD1 E1 B takoder - površina
pravokutnog trapeza OD1 E1 C s osnovicama |OC| = 20 (u m/ s) i |D1 E1 | = |DE| = v2
i visinom |OD1 | = t1 − Δt tj.
20 + v2
POABC − PADEB = (t1 − Δt). (4)
2
Zamjenom relacija (1) i (2) u (4) dobiva se:
20 + v2
0= (t1 − Δt) odakle je t1 = Δt (5)
2
pa iz (3) dobivamo v2 = 20 (u m/ s).
Zato graf brzine tijela u ovisnosti od vremena, zapravo, izgleda ovako.
S grafa i iz relacije (5) se dobiva
t2 = t1 + Δt = 2t1 . (6)
S grafa su očigledne i relacije:
20 − v1
g= i (7)
t1
20 + v1
POABC = t1 = 15. (8)
2
Iz relacija (7) i (8) se dobiva kvadratna jednadžba: gt12 − 40t1 + 30 = 0 čija su
rješenja: t1 = 0.99 (u s) i t1 = 3.09 (u s). Rješenje t1 = 3.09 nema fizički smisao
jer se iz (7) za njega dobiva negativna vrijednost za veličinu brzine, što je nemoguće
(v1 = 20 − gt1 = −10.3). Na osnovu rješenja t1 = 0.99 (u s) i relacije (6) dobiva se
proteklo vrijeme od trenutka izbačaja do povratka tijela
t2 = 2t1 = 2 · 0.99 = 1.98 (u s)
Ova nestandardna metoda rješavanja problema kretanja može se veoma uspješno
primijeniti i na sve vrste hitaca (vertikalan, kosi, horizontalan), kao i na slaganje
kretanja, pa se naizgled jako teški zadaci mogu lako i brzo riješiti.

100 Matematičko-fizički list, LIX 2 (2008. – 2009.)


Zadaci za vježbu

- put od 3 m.
1. Tokom šeste sekunde, krećući se jednoliko usporeno, motociklista prijede
2
Ubrzanje kojim se pri tome kretao ima intenzitet od 2 m/ s . Kolika mu je početna brzina? (Rj.
14 m/ s.) [2]
2. Balon se kreće vertikalno uvis stalnom brzinom 5 m/ s. Poslije 20 s od balona se odvoji
kuglica.
a) Kolika je najveća visina do koje će kuglica dospjeti? (Rj. 101.27 m.)
b) Poslije kojeg vremena će kuglica pasti na Zemlju ako se ono mjeri od trenutka odvajanja
od balona? (Rj. 5.05 s.)
c) Kolika je srednja brzina kuglice na čitavom putu? (Rj. 8.08 m/ s.) [2]
3. Ako se početna brzina tijela poveća 1.2 puta, pri nepromijenjenom usporenju, zaustavni
put se poveća za 22 m. Koliki je bio zaustavni put prije navedene promjene? (Rj. 50 m.) [2]
4. Čestica, koja izlijeće iz nekog izvora, najprije prelazi 200 m stalnom brzinom v , a zatim
usporava s 2 m/ s2 . Pri kojoj je vrijednosti brzine v minimalno r vrijeme kretanja čestice od
l
izlijetanja iz izvora do zaustavljanja? (Rj. 20 m/ s; tmin = 2 ). [6]
g
5. Tri kuglice se puste slobodno padati, svaka sljedeća sa zakašnjenjem od jedne sekunde.
-
Kolike su njihove medusobne udaljenosti pet sekundi nakon puštanja prve? (Rj. 44.145 m;
34.335 m; 78.48 m.) [1], [2]

Literatura

[1] LJILJANA SUDAR , Geometrija pomaže fizici, Zbornik sa XXVI republičkog seminara o
nastavi fizike, Društvo fizičara Srbije, Vrnjačka Banja 2008., str.123–130.
[2] JEVREM JANJIĆ , MIROSLAV PAVLOV , BRANKO RADIVOJEVIĆ , Fizika za I razred srednjeg
obrazovanja i vaspitanja, Naučna knjiga, Beograd 1987. str. 129–132.
[3] Zadaci i rješenja – rješenja iz fizike, zadatak OŠ–259., zadatak 1357., MFL 1/ 229, god.
LVIII, Zagreb 2007.–2008., str. 45, 46.
[4] MILAN ŠARIĆ , BOGOLJUB MARINKOVIĆ , Različiti načini rešavanja zadataka II, problemi
kretanja, Materijali za mlade matematičare sv. 83 (10), Arhimedes–R. Kašanin, Beograd–
Beli Manastir 1995., str. 16.
[5] Kvalifikacijski ispiti, zadatak F–25., MFL 2/ 218, god. LV, Zagreb 2004.–2005., str. 126.
[6] NATA ŠA ČALUKOVIĆ , MILAN RASPOPOVIĆ , Fizika 1M, Krug, Beograd 2001., str. 9.



Bangov teorem

Sve strane tetraedra imaju isti opseg ako i samo ako su


-
one medusobno sukladne.
Sve strane tetraedra imaju jednake površine ako i samo
-
ako su one medusobno sukladne.

Matematičko-fizički list, LIX 2 (2008. – 2009.) 101


Teleskop MAGIC – čarobni instrument astročestične fizike

Dario Hrupec 1 , Koprivnica

Astročestična fizika

Astročestična je fizika novo, interdisciplinarno, znanstvenoistraživačko područje koje


uključuje fiziku čestica, astronomiju i astrofiziku te kozmologiju (dijelom takoder, -
-
nuklearnu fiziku i gravitaciju). Ona istražuje, izmedu ostalog, daleke i egzotične objekte
poput pulsara 2 i supermasivnih crnih rupa. U blizini takvih objekata postoje polja
(električno, magnetsko i gravitacijsko) ekstremne jakosti koja uzrokuju ubrzavanje
čestica do vrlo visokih energija te emisiju različitih valova (elektromagnetskih i
gravitacijskih). Čestice i valove, koji dodu - do Zemlje i ondje budu opaženi, nazivamo
prenositeljima (engl. messenger). Istovremeno opažanje različitih prenositelja koji dolaze
iz istog kozmičkog objekta, što je danas u astronomiji trend, nazivamo kombiniranim
pristupom (engl. multimessenger approach) [1].

Gama-astronomija

Jedna od najuspješnijih grana astročestične fizike danas je gama-astronomija. Ona


istražuje kozmičke gama-zrake, odnosno elektromagnetsko zračenje najviših energija koje
dolazi iz svemira [2]. Niže energijsko područje kozmičkih gama-zraka (do par desetaka
GeV 3 ) dostupno je, neposredno, samo detektorima na satelitima. Trenutno je u orbiti
-
nekoliko satelita koji opažaju kozmičke gama-zrake, no najveći i najperspektivniji medu
njima je nedavno lansirani satelit Fermi (bivši GLAST) [3]. Više energijsko područje
kozmičkih gama-zraka (od par desetaka GeV do par desetaka TeV) dostupno je,
posredno, detektorima na Zemlji. Visokoenergijske gama-zrake (kao i visokoenergijske
nabijene čestice, takozvane kozmičke zrake) izazivaju u atmosferi velike pljuskove
sekundarnih čestica. Nabijene sekundarne čestice u pljusku, čija je brzina veća od
brzine svjetlosti 4 u zraku, emitiraju posebnu vrstu svjetlosti – Čerenkovljevo zračenje.
Čerenkovljevo zračenje iz pljuska sekundarnih čestica u atmosferi moguće je opaziti
1 - Bošković” u Zagrebu, e-mail: dario.hrupecirb.hr
Autor je viši asistent u Institutu “Ruder
2 Pulsar je brzorotirajuća neuronska zvijezda, kozmički objekt ogromne gustoće, središnji ostatak eksplozije
supernove.
3 GeV je gigaelektronvolt, 109 eV, a TeV teraelektronvolt, 1012 eV.
-
4 Brzina svjetlosti u vakuumu, c , granična je brzina u prirodi. Medutim, kroz prozirnu tvar (npr. staklo, vodu
c
ili zrak) svjetlost putuje sporije, brzinom v = , gdje je n indeks loma tvari. Čestica se kroz tvar može gibati
n
brzinom koja je veća od v , ali još uvijek manja od c .

102 Matematičko-fizički list, LIX 2 (2008. – 2009.)


posebnom vrstom teleskopa – Čerenkovljevim teleskopom. Danas postoji nekoliko
sustava Čerenkovljevih teleskopa, H.E.S.S. u Namibiji [4], MAGIC na Kanarima [5],
VERITAS u Arizoni te CANGAROO III u Australiji. Čerenkovljevi teleskopi su iznimno
uspješni znanstveni instrumenti koji gotovo svakodnevno donose nova otkrića. Vodeću
ulogu u zemaljskoj gama-astronomiji danas drže dva europska 5 sustava teleskopa,
MAGIC i H.E.S.S. Kolaboracije MAGIC i H.E.S.S. takoder - zajednički rade na budućem
velikom sustavu, od par desetaka do možda i stotinu, Čerenkovljevih teleskopa CTA
(Cherenkov Telescope Array) koji bi trebao proraditi za pet do deset godina [6].

Slika 1. Teleskop MAGIC na kanarskom otoku La Palmi. Izvor: Robert Wagner, MPI, München.

Uvod u teleskop MAGIC

Teleskop MAGIC (Major Atmospheric Gamma Imaging Cherenkov) najveći je


Čerenkovljev teleskop, promjera 17 m, i trenutno jedan od znanstveno najproduktivnijih
instrumenata iz područja astročestične fizike. Smješten je na vrhu kanarskog otoka La
Palma, na visini 2200 m, unutar opservatorija Roque de los Muchachos. Teleskop je
dovršen krajem 2003. godine i opaža punim kapacitetom od 2004. godine. Pri samom
je završetku drugi teleskop, MAGIC II, koji bi trebao proraditi početkom 2009. godine.
Oba će teleskopa moći raditi samostalno ili u paru. Kolaboracija MAGIC broji oko 150
članova, čak iz 17 institucija. Od ove godine članica kolaboracije MAGIC je i hrvatska
grupa (Croatian MAGIC Consortium) od šest fizičara s Instituta “Ruder - Bošković”,
Sveučilišta u Splitu i Sveučilišta u Rijeci. Za razliku od optičkih teleskopa, Čerenkovljev
teleskop nema cjeloviti reflektor nego segmentirani, sastavljen od mnoštva manjih zrcala,
kako bi se relativno jeftino dobila što veća sabirna površina. Teleskop MAGIC ima
ukupnu površinu reflektora od 236 m2 . Nadalje, kamera Čerenkovljevog teleskopa ne
- CCD (Charge-Coupled Device), kao kod većine optičkih teleskopa,
sastoji se od uredaja
nego od fotomultiplikatora 6 . To su, zasad, jedini prikladni fotosenzori za bilježenje vrlo
kratkotrajnih bljeskova Čerenkovljeve svjetlosti. Kamera teleskopa MAGIC II sadržavat
će, po prvi put u povijesti Čerenkovljevih teleskopa, napredne fotosenzore tipa HPD
(Hybrid Photo-Detector). U razvoju fotosenzora HPD važnu su ulogu odigrali i hrvatski
fizičari, posebice prof. dr. Daniel Ferenc sa Sveučilišta u Kaliforniji, Davis.
5 Oba su sustava zemljopisno smještena na afričkom kontinentu, zbog klime, no vode ih europske kolaboracije.
6 Fotomultiplikator je iznimno osjetljiv detektor svjetlosti koji može opaziti čak pojedinačne čestice svjetlosti,
fotone. Posebne elektrode u cijevi, takozvane dinode, djeluju kao pojačivač, ili multiplikator, signala.

Matematičko-fizički list, LIX 2 (2008. – 2009.) 103


Ciljevi i rezultati teleskopa MAGIC

Popis ciljeva teleskopa MAGIC iznimno je opsežan. U prvom redu tu su


izvangalaktički objekti i pojave: aktivne galaktičke jezge i provale gama-zračenja
(engl. gamma ray burst, GRB), zatim galaktički objekti: pulsari, dvojni sustavi
kompaktnih kozmičkih objekata 7 te ostaci supernova. Vezano uz kozmologiju, medu -
ciljevima su primjerice izvangalaktička pozadinska svjetlost te, danas jako aktualna,
tamna tvar.

Slika 2. Umjetnički prikaz aktivne galaktičke jezgre. Vidi se akrecijski disk i dva relativistička
mlaza. U samom središtu je supermasivna crna rupa. Izvor: NASA.
- važnijim
Popis dosad ostvarenih rezultata je velik i izvan opsega ovog teksta. Medu
otkrićima, koja su nedavno objavljena u uglednom časopisu Science, mogu se spomenuti:
otkriće visokoenergijskih gama-zraka iz udaljenog kvazara 3C279 8 te otkriće pulsnog
gama-zračenja iz pulsara Crab 9 .

Literatura

[1] D. HRUPEC , Kombinirani pristup u astronomiji, MFL LVI 4 (2005/ 2006)


[2] D. HRUPEC , Gama-astronomija – posljednji elektromagnetski prozor u svemir, MFL LVI
1 (2005/ 2006)
[3] http://fermi.gsfc.nasa.gov/
[4] http://www.mpi-hd.mpg.de/hfm/HESS/
[5] http://magic.mppmu.mpg.de/
[6] http://www.cta-observatory.org/

7 Kompaktni kozmički objekti su objekti vrlo velike gustoće, bijeli patuljci, neutronske zvijezde ili crne rupe,
koji nastaju nakon smrti običnih zvijezda. Naše Sunce ima relativno malu masu pa će, kad potroši sve svoje
nuklearno gorivo, završiti kao bijeli patuljak za otprilike pet milijardi godina.
8 Objavljeno u časopisu Science, 27. lipnja 2008.
9 Objavljeno u ScienceXpress, 16. listopada 2008.

104 Matematičko-fizički list, LIX 2 (2008. – 2009.)


stolićem brat Branko. — Dati za domaću
zadaću da se nadu - zadnje znamenke
ovakvih potencija 550 , 8800 , 99000 ! Zar
on misli da ću ja čitav vikend množiti
silne petice, osmice i devetke?
Profesor Strogi bio je toga dana iznimno
dobrog raspoloženja. Počeo je veselo:
— Prošlo je već sedam godina ovoga Za drugim stolićem sestra Snježana se
stoljeća, a i osma je pri kraju. Danas ćemo smješkala:
ih malo srediti. — Da, da, na velikim si mukama! Samo
— Strogi nas godinama “sreduje”. - si zaboravio uputu našeg dobrog profesora.
Bojim se da će nas i ovaj puta puno Na moraš stvarno množiti bazu po bazu
srediti! — čulo se otpozadi. da bi doznao kojom znamenkom završava
— Čuo sam vas! Ali danas nemam svaka od tih potencija.
vremena za prodiku. Treba raditi. Evo, na Što je Branko zaboravio?
ploči pišem sve brojke po veličini. Vaš
je zadatak da osam puta izmedu - nekih
brojki stavite znakove računskih radnji,
a po potrebi i zagrade, tako da dobijete Ako središta susjednih strana kocke po-
izraze kojima su redom vrijednosti 2001, vežemo dužinama, dobiva se geometrijsko
2002, 2003, 2004, 2005, 2006, 2007, 2008. tijelo koje ima 6 vrhova.
Posebno ću ocijeniti ona rješenja koja nisu
na mome popisu rješenja. Rješavanje vam
mogu olakšati ove jednakosti: 91 · 22 =
2002 , 667 · 3 = 2001 , 654 · 3 = 1962 . Na
posao!

Sela A, B i C čine vrhove pravilnog


trokuta.U selu A živi 30 školaraca, u selu Evo pitanja:
B ih je 20, dok je u selu C njihov broj 1) Koje je to tijelo?
najmanji i iznosi 10. Svi školarci išli su u
školu u prilično udaljenom mjestu i zato su 2) Koliki je obujam toga tijela ako je
žitelji tih triju sela odlučili sagraditi školu duljina brida kocke 1?
u svojoj blizini.

Na crtežu je krug razdijeljen sa 4 pravca


na 9 dijelova. Pokažite da je moguće
odabrati 4 pravca koji krug dijele na manji,
odnosno veći broj dijelova od 9. Svaki
pravac treba pritom obodnu kružnicu sjeći
Gdje treba sagraditi tu školu da zbroj u dvije točke.
udaljenosti, koje svi školarci moraju prijeći
od svojih sela do škole, bude najmanji?

Brat i sestra Veselić odlučili su odmah


riješiti domaću zadaću kako bi imali više
slobodnog vremena za odmor. Ali na Koji se najmanji, a koji najveći broj di-
početku nije sve krenulo najbolje. jelova može dobiti na taj način? Pokušajte
— Naš profesor Dobrić ovaj put je riješiti zadatak ako je broj pravaca 5 ili 6.
pretjerao — ljutio se za svojim radnim Zdravko Kurnik, Zagreb

Matematičko-fizički list, LIX 2 (2008. – 2009.) 105


izvana u točkama D , E i F . Dokaži da je
omjer površina trokuta DEF i ABC jednak
2pqr
.
(p + q)(q + r)(r + p)
Redakcija, iz tehničkih razloga, daje ovo
3140. U ravnini je zadan konveksni četvero-
upozorenje:
kut ABCD . S njegove unutarnje strane nacrtani
Krajnji rok za primanje rješenja iz ovog su jednakostranični trokuti ABQ i CDN , a s
broja je 28. veljače 2009. Rješenja (i imena njegove vanjske strane jednakostranični trokuti
rješavatelja) bit će objavljena u br. 4/ 236. BCM i ADP . Dokaži da je četverokut MQPN
Ujedno molimo da pripazite na upute rješa- paralelogram.
vateljima koje su na str. 143. 3141. Ako za kutove α , β i γ trokuta
vrijedi
A) Zadaci iz matematike sin2 α + sin2 β + sin2 γ = 2,
dokaži da je on pravokutan.
U zadatku 3131. pojavila se greška pa
navodimo ispravnu formulaciju. 3142. Za bilo koje točke A , B , C i D u
prostoru dokaži nejednakost
3131. Izračunaj sumu
|AC|2 +|BD|2 +|AD|2 +|BC|2 ≥ |AB|2 +|CD|2 .
1 2
+ 4
14 + 12 + 1 2 + 22 + 1 3143. U koordinatnoj ravnini dane su točke
2008 A(−a, 0) i B(2a, 0) , a > 0 . Što je skup svih
+... + . točaka P(x, y) takvih da je < )PBA = 2<)BAP ?
20084 + 20082 + 1
Koje točke na osi x pripadaju tom skupu?
3133. Ako su p i p2 + 8 prosti brojevi
dokaži da je i p2 + 4 prost broj. 3144. Odredi produkt
n
!
3134. Za dani prirodan broj n pokaži da je Y 2 2 π
k
1 − tg n .
64n+1 − 63n − 64 djeljivo s 3969. Općenito, 2 +1
k=1
dokaži da je za svake prirodne brojeve n i a
broj (a + 1)n+1 − an − (a + 1) djeljiv s a2 . 3145. Bridovi AB , AC i AD tetraedra
3135. Dokaži da su za svaki prirodan broj ABCD su u parovima okomiti i njihove duljine
n ≥ 5 prvih n decimala broja su redom a , b i c . Odredi polumjer sfere
p opisane tom tetraedru.
(n + n2 + 1)n
3146. U nekom gradu ima 10 000 bicikala,
jednake nuli. svaki je označen brojem od 1 do 10 000 tako
3136. Nadi - sva pozitivna rješenja jednažbe da nikoja dva od njih nemaju isti broj. Kolika
q p p je vjerojatnost da je svaka znamenka broja
x 1 − y2 + y 2 − z2 + z 3 − x2 = 3. slučajno izabranog bicikla različita od 8?
3137. Omjer duljina stranica paralelograma
je p : q , a omjer duljina njegovih dijagonala
je m : n . Odredi kutove paralelograma. B) Zadaci iz fizike
3138. Neka je P proizvoljna točka unutar
šiljastokutnog trokuta ABC i k opisana mu OŠ – 286. Kroz bakrenu zavojnicu teče
kružnica. Pravci AP , BP i CP po drugi put struja jakosti 0.4 A kad je napon na njenim
sijeku k u točkama A1 , B1 i C1 . Označimo krajevima 5.4 V. Nakon mjerenja učenici
s A2 , B2 i C2 ortogonalne projekcije točke P su odmotali žicu s kalema i izvagali je.
na pravce BC , CA i AB , tim redom. Dokaži Masa joj je iznosila 11 g. Koliki su bili
da su trokuti A1 B1 C1 i A2 B2 C2 slični. promjer i duljina bakrene žice? Gustoća bakra
3139. Tri kružnice polumjera p , q i r sa iznosi 8900 kg/ m3 , a električna otpornost
središtima A , B i C , tim redom, dodiruju se 0.017 · 10−6 Ω m.

106 Matematičko-fizički list, LIX 2 (2008. – 2009.)


OŠ – 287. Nekad su se kotači za zaprežna 1409. Elektron se kreće brzinom 0.8c .
kola pravili stavljanjem željeznih obruča na Kolika je masa elektrona i ukupna energija
drvene kotače. Na kotač promjera 80 cm (m0 = 9.1 · 10−31 kg) .
stavljao se željezni obruč kojem je promjer bio 1410. Tanki homogeni štap mase m1 = 2 kg
4 mm manji od promjera kotača. Za koliko i duljine l = 2 m učvršćen je o jedan kraj
stupnjeva je trebalo zagrijati obruč da bi se pomoću zgloba, a o drugi je privezan na
mogao staviti na kotač? Koeficijet linearnog oprugu tako da je ravnotežni položaj štapa
širenja željeza iznosi 1.2 · 10−5 K−1 . okomit. Opruga ima zanemarivu masu, a
OŠ – 288. Markov otac želi prije zime njezina konstanta elastičnosti je k = 6.1 N/ m.
napuniti cisternu s uljem za loženje. Njihova Izračunaj period malih oscilacija štapa.
cisterna je pravokutnog oblika, duljine 2 metra,
široka 1 metar i visoka 1.5 metara. Napunjena
je uljem do visine 40 cm. Koliko litara ulja
Markov otac mora naručiti da bi napunio
cisternu do vrha? Koliko će se nakon punjenja
povećati hidrostatski tlak na dnu cisterne?
Gustoća ulja za loženje je 850 kg/ m3 .
OŠ – 289. Jedan dječak vuče sanjke
na kojima sjedi drugi dječak mase 42 kg.
Sila vučenja iznosi 30 N. Masa sanjki je 1411. Izračunaj doskok skijaša skakača koji
6 kilograma. Maksimalna mišićna sila dječaka polijeće brzinom od 20 m/ s pod kutom 15◦
koji vuče sanjke iznosi 64 N. Hoće li on moći prema horizontalnoj ravnini, uz pretpostavku
vući sanjke ako na njih sjedne treći dječak da se otpor zraka zanemaruje i da se padina na
mase 50 kg? koju skijaš skače može aproksimirati kosinom
1406. Na kojoj je visini ubrzanje slobodnog koja s horizontalnom ravninom zatvara kut od
pada jednako 25% od ubrzanja slobodnog pada 45◦ . (Doskok je udaljenost od točke polijetanja
na površini Zemlje? do točke u kojoj skijaš dodirne padinu.)
1412. U valjkastu posudu polumjera
1407. Metak mase m = 6 g ispaljen je
r = 10 cm ulije se 1 litra vode. U posudu
horizontalno u blok mase M = 2.8 kg koji
spustimo staklenu kocku (ρ = 4000 kg/ m3 ) .
miruje na horizontalnoj podlozi koeficijenta
Koliku je silu potrebno upotrijebiti da se kocka
trenja μ = 0.3 . Metak se zabije u blok i ostaje
podigne s dna posude, ako je njezina masa
u njemu, a blok se pomakne za l = 0.65 m i
- početnu brzinu metka! m = 0.75 kg?
zaustavi se. Nadi
C) Rješenja iz matematike

U rješenju zadatka 3093. pojavila se greška


pa donosimo ispravno rješenje.
1408. Na horizontalnoj podlozi leži kugla 3093. Ako su a , b i c pozitivni realni
mase M = 2 kg spojena preko opruge konstante brojevi, dokaži nejednakost
elastičnosti k = 500 N/ m s čvrstim zidom. b+c c+a a+b 1 1 1
Metak mase m = 20 g i brzine v1 = 600 m/ s + 2 + ≤ + + .
a2 + bc b + ca c2 + ab a b c
zabije se u kuglu i ostaje u njoj.
Rješenje.
1 1 1 b+c c+a a+b
+ + − − − 0
a b c a2 +bc b2 +ca c2 +ab
.
· abc(a2 + bc)(b2 + ca)(c2 + ab)
a) Koliko će se stisnuti opruga?
⇐⇒ . . . ⇐⇒
b) Odredi period titranja sistema!

Matematičko-fizički list, LIX 2 (2008. – 2009.) 107


a4 b4 +b4 c4 +c4 a4 −a4 b2 c2 −b4 c2 a2 −c4 a2 b2 3106. Neka su a , b , c realni brojevi takvi
0.
abc(a2 +bc)(b2 +ca)(c2 +ab) da je abc = 0 . Ako je
( ∗) b c c a a b
Koristeći nejednakost x= + , y= + , z= + ,
c b a c b a
dokaži
x2 + y2 + z2  xy + yz + zx x2 + y2 + z2 − xyz = 4.
1
⇐⇒ [(x − y)2 + (y − z)2 + (z − x)2 ]  0
2
Rješenje.
imamo za x = a2 b2 , y = b2 c2 , z = c2 a2 :
x + y2 + z2 − xyz = 4,
2
(a2 b2 )2 + (b2 c2 )2 + (c2 a2 )2
x2 + y2 + z2 = 4 + xyz,
 a2 b2 · b2 c2 + b2 c2 · c2 a2 + c2 a2 · a2 b2 „ «2 „ «2 „ «2
b c c a a b
⇐⇒ a4 b4 + b4 c4 + c4 a4 + + + + +
c b a c b a
− a4 b2 c2 − b4 c2 a2 − c4 a2 b2  0. „ «„ «„ «
b c c a a b
=4+ + + + ,
Zato je ispravna nejednakost (∗) . c b a c b a
Jednakost vrijedi ako i samo ako je b2 c2 c2 a2 a2 b2
a = b = c. 2
+2+ 2 + 2 +2+ 2 + 2 +2+ 2
c b a c b a
„ «„ «
Edin Ajanović (3), b ab c2 a a b
=4+ + 2 + + + ,
Prva bošnjačka gimnazija, Sarajevo, BiH a c ab b b a
b2 c2 c2 a2 a2 b2
3105. Dokaži da niz brojeva an = 2
+ 2 + 2 + 2 + 2 + 2 +6
c b a c b a
3n + 4, n ∈ N ne sadrži nijedan kvadrat
prirodnog broja. b2 a2 b2 c2 c2 a2
= 4 + 1 + 2 + 2 + 2 + 2 + 2 + 2 + 1.
a c c b a b
Rješenje. Pretpostavimo suprotno, tj. da za Na kraju i lijevo i desno imamo iste članove
neki n vrijedi 3n + 4 = m2 , m ∈ N . Budući -
koji se svi medusobno eliminiraju.
da 3  | m2 , onda ni 3  | m , David Devald (1),
3n = m2 − 4, SŠ Izidora Kršnjavoga, Našice
3n = (m + 2)(m − 2). 3107. Nadi- sva cjelobrojna rješenja jed-
Jer je 3 prost broj, oba faktora moraju biti nadžbe
djeljiva s 3 ili jedan od njih mora biti jednak (x−2005)(x−2006)(x−2007)(x−2008) = 3024.
1.
Kad bi oba broja bila djeljiva s 3, tad bi
Rješenje. Zamjenom x − 2005 = a jednadž-
i njihova razlika bila djeljiva s 3, no njihova
ba postaje
je razlika jednaka 4, stoga barem jedan od tih
brojeva nije djeljiv s 3. Dakle, preostaje jedino a(a − 1)(a − 2)(a − 3) = 3024,
da je jedan od tih faktora jednak 1. (a2 − 3a)(a2 − 3a + 2) = 3024.
Za m + 2 = 1 je m = −1 što nije moguće, Supstitucijom a2 − 3a = b dobivamo
jer je m prirodan broj.
b(b + 2) = 3024,
Za m − 2 = 1 je m = 3 što nije moguće,
jer m nije djeljiv s 3. b2 + 2b − 3024 = 0.
Dakle, niz (an ) ne sadrži nijedan kvadrat Rješenja ove jednadžbe su b1 = 54 i
prirodnog broja. b2 = −56 .
Za b1 = 54 je
Kristijan Kvaternik (1),
V. gimnazija, Zagreb a2 − 3a = 54,

108 Matematičko-fizički list, LIX 2 (2008. – 2009.)


√ √ „ «
a2 − 3a − 54 = 0, z2 + zx + x2

3 1
+
1
.
a1 = 9, a2 = −6, zx 2 z x
Zbrajanjem tih triju nejednakosti dobivamo
x1 = 2014, x2 = 1999. zadanu nejednakost:
p p √
Za b2 rješenje je x2 +xy+y2 y2 +yz+z2 z2 +zx+x2
+ +
xy yz zx
a2 − 3a = −56, √ „ «
3 1 1 1 1 1 1
a2 − 3a + 56 = 0. 
2 x
+ + + + +
y y z z x
„ «
Jednadžba nema realnih rješenja. √ 1 1 1
 3 + + .
Dakle, x ∈ {1999, 2014} x y z
Kristijan Kavternik (1), Zagreb David Devald (1), Našice

√ 3109. U decimalnom zapisu broja (5 +


3108. Za pozitivne realne brojeve x , y , z
dokaži nejednakosti: 26)n odredi prvih n znamenaka poslije
q √ decimalne točke.
3 √
a) x2 + xy + y2 ≥ (x + y) ; Rješenje.
√ Neka je α = 5 + 26 , β =
2
5− 26 . Brojevi α i β su rješenja koordinatne
b) jednadžbe
p p √
x2 +xy+y2 y2 +yz+z2 z2 +zx+x2 x2 − 10x − 1 = 0.
+ +
xy yz zx Ako je
„ « √
√ 1 1 1 α n = an + bn 26,
≥ 3 + + .
x y z tada je √
β n = an − bn 26,

p 3 pa je
Rješenje. a) x2 + xy + y2  (x + y) ,
2 α n + β n = 2an = An
3 2 prirodan broj. Odatle slijedi
⇐⇒ x2 + xy + y2  (x + 2xy + y2 ),
4 An − α n = β n i
⇐⇒ x2 − 2xy + y2  0, 1 1
2 |An − αn | = |βn | = √ < .
⇐⇒ (x − y)  0. (5 + 26)n 10n
Prema tome, razlika izmedu - α n i An po
b) Iskoristit ćemo dokazanu nejednakost iz 1
a): apsolutnoj je vrijednosti manja od . Kako
q √ 10n
3 je β < 0 za parno n prvih n znamenaka poslije
x2 + xy + y2  (x + y). decimalne točke su devetke, a za neparno n
2
prvih n znamenaka poslije decimalne točke su
Podijelimo tu nejednakost s xy :
p √ nule.
x2 + xy + y2 3x+y Ur.
 ,
xy 2 xy
p √ „ « 3110. Odredi cjelobrojno rješenje jednadžbe
x2 + xy + y2 3 1 1 √ √ p
 + . 3 3 3
xy 2 x y 1 + 2 + . . . + x3 − 1 = 400.
Analogne nejednakosti dobivamo za parove y ( x je oznaka za najveći cijeli broj koji nije
i z , odnosno z i x : veći od x .)
p √ „ « √ √
y2 + yz + z2 3

3 1 1
+ , Rješenje. Kako je 3 8 = 23 = 2 , prvih
yz 2 y z 7 članova ima vrijednost 1, pa je njihov zbroj

Matematičko-fizički list, LIX 2 (2008. – 2009.) 109


√ √3
7 · 1 = 7 . Zatim, 3 27 = 33 = 3 pa svi Uz oznake kao na slici dani uvjet možemo
članovi od 8. do 26. imaju vrijednost 2. Njihov zapisati u obliku 2b − x = 2(a + x) . Iz
√ √3
zbroj je 19 · 2 = 38 . 3 64 = 43 = 4 , pa svi sličnosti ABD i BCE slijedi a : x =
članovi od 27. do 63. imaju vrijednost 3. Njihov a2
√ √
3 2b : a , odnosno x = . Uvrštavanjem
zbroj je 37·3 = 111 . Dalje 3 125 = 53 = 5 , 2b
x u početnu jednadžbu dobivamo kvadratnu
pa svi članovi od 64. do 124. imaju vrijednost
jednadžbu 4b2 − 4ab − 3a2 = 0 . Za pozitivno
4. Njihov zbroj je 61 · 4 = 244 . Zbrojimo b 3
7 + 38 + 111 + 244 = 400 . Zaključimo da je rješenje dobivamo = . Zbog cos β =
√ √
3 a 2
zadnji član jednak 3 124 = x3 − 1 iz a 1
= slijedi β = 70◦ 32 , a α = 90◦ − β =
čega slijedi x3 = 125 , odnosno x = 5 . 2b 3
19◦ 28 .
David Devald (1), Našice
Vanja Ubović (2),
3111. Ako je zbroj cijelih brojeva a , b , c Gimnazija P. Preradovića, Virovitica
jednak 0, dokaži da je 2(a4 + b4 + c4 ) kvadrat
cijelog broja.
3113. Ako su α , β , γ kutovi šiljastokutnog
Rješenje. Kako je a + b + c = 0 imamo trokuta, dokaži da vrijedi nejednakost
a2 + b2 + c2 = −2(ab + bc + ca). sin α + sin β + sin γ ≤ 2(cos α + cos β + cos γ ).
Tada je
(a2 + b2 + c2 )2 = 4(ab + bc + ca)2 ,
Rješenje.
a4 + b4 + c4 + 2(a2 b2 + b2 c2 + c2 a2 )
= 4(a2 b2 + b2 c2 + c2 a2 ) + 8abc(a + b + c),
tj. a4 + b4 + c4 = 2(a2 b2 + b2 c2 + c2 a2 ).
Sada je
2(a4 + b4 + c4 )
= a4 + b4 + c4 + 2(a2 b2 + b2 c2 + c2 a2 )
= (a2 + b2 + c2 )2 .
Po poučku o sinusima vrijedi a = 2R sin α ,
Edin Ajanović (3), Sarajevo, BIH b = 2R sin β , c = 2R sin γ . Za a = |AH|
imamo
3112. Točka D je nožište visine spuštene
iz vrha A na krak BC jednakokračnog trokuta |AC | = a · cos <
)HAC1 = a cos <
)A1 AB
ABC . Ako je |AC| + |CD| = 2(|AB| + |BD|) , „ «
π
koliki su njegovi kutovi? = a cos − β = a sin β tj.
2
Rješenje. |AC | b cos α
a = =
sin β sin β
2R sin β · cos α
= = 2R cos α .
sin β
Analogno se dobije
b = 2R cos β , c = 2R cos γ .
Iz nejednakosti o trokutu vrijedi
a + b > c,
b + c > a,

110 Matematičko-fizički list, LIX 2 (2008. – 2009.)


„ «2 „ √ «2
c + a > b, 1 2
= 1+ √ = 1+ √
odakle je sin α cos α sin 2α
√ √
 (1 + 2)2 = 3 + 2 2.
2(a + b + c ) > a + b + c, tj.
2(cos α + cos β + cos γ > sin α + sin β + sin γ . Ur.

Dakle, vrijedi stroga nejednakost. Drugo rješenje. Koristit ćemo Huygensovu


Ur. nejednakost: Ako su realni brojevi xi > 0 ,
i = 1, 2, . . . , n , tada vrijedi nejednakost
„ «n
3114. Kompleksni brojevi zadovoljavaju ove √
(1+x1 )(1+x2 ) . . . (1+xn )  1+ n x1 x2 . . . xn .
uvjete:
z1 + z2 = −i − 1, Iz dane nejednakosti slijedi
„ «„ « „ «2
z1 · z2 = −i. 1 1 1
1+ 1+  1+ √
Ne računajući z1 i z2 , odredite z1 · z2 . sin α cos α sin α cos α
„ «2
A−G 2 √ √
Rješenje. Primijetimo da je  1+  (1+ 2)2 = 3+2 2.
z sin α cos α
z1 · z2 = 1 · |z2 |2 . ( ∗)
z2 Radi poznate nejednakosti
Iz (z1 + z2 )2 = 2i = −2z1 · z2 , dobivamo da je √
2 √
0 < sin α + cos α  2 = 2,
z1 2 + 4z1 z2 + z2 2 = 0 2
odnosno
ili u ekvivalentnom obliku
„ «2 „ « 1 1
z1 z  √ ,
+ 4 1 + 1 = 0. sin α + cos α 2
z2 z2 slijedi (∗) . Jednakost vrijedi ako i samo ako
z1 √
Vidimo da je realno i negativno. Zato 2
z je sin α = cos α = , tj. ako i samo ako je
je z1 · z2 takoder- 2 realno i negativno. Kako 2
α = 45◦ .
je |z1 · z2 | = |z1 · z2 | = 1 , konačno dobijemo
z1 · z2 = −1 . Vanja Ubović (2), Virovitica

Ur. 3116. Koliko se najviše lovaca može


“ postaviti na šahovsku ploču n × n tako da
π” se nikoja dva ne napadaju.
3115. Dokaži da za α ∈ 0, vrijedi
2
nejednakost Rješenje. Promotrimo dijagonale šahovske
„ «„ «
1 1 √ ploče n · n .
1+ 1+ ≥ 3 + 2 2.
sin α cos α

Prvo rješenje.
fi fl
π
α ∈ 0, =⇒ sin α , cos α > 0 ,
2
„ «„ «
1 1
1+ 1+
sin α cos α
„ «
1 1 1
=1+ + +
sin α cos α sin α cos α (Tih dijagonala ima 2n − 1 .)
2 1 Na svakoj od tih dijagonala smije stajati
1+ √ +
sin α cos α sin α cos α najviše jedan lovac. No na dijagonali 1 i 2n−1

Matematičko-fizički list, LIX 2 (2008. – 2009.) 111


ne mogu istovremeno stajati dva lovca. (To su 2n 2n + 1
= − ,
polja na suprotnim kutovima ploče.) a2n − 1 n+1
a2 − 1
Zaključak: najviše možemo staviti (2n − pa je
1) − 1 = 2n − 2 lovaca. k
X 2n
Sk = n
No postoji razmještaj 2n − 2 lovaca uz a2 +1
n<0
zadane uvjete:
k „
X «
2n 2n + 1
= 2n − 2n+1
a −1 a −1
n=0
1 2k+1
= − 2k+1 ,
a−1 a −1
„ «
1 2k+1
S = lim Sk = lim − k+1
k→∞ k→∞ a − 1 a2 − 1
1
= .
a−1
2k + 1
jer je lim k+1 = 0.
k→∞ a2 −1
Ur.
Edin Ajanović (3), Sarajevo
3117. Nadi- vjerojatnost da slučajno izabra-
ni dvoznamenkasti broj bude djeljiv barem ili
s 3 ili sa 7. D) Rješenja iz fizike
Rješenje. Neka je A dogadaj - u kojem je
izabran broj djeljiv s 3, a B dogadaj - u
OŠ – 278. Lukina baka stanuje na četvrtom
kojem je izabran broj djeljiv sa 7. Treba naći - katova je 3 metra.
katu. Visinska razlika izmedu
vjerojatnost p(A + B) , pri čemu se dogadaji- A
Luka je izmjerio da lift putuje do četvrtog kata
i B ne isključuju. Dvoznamenkastih brojeva 12 sekundi. Snaga elektromotora koji pokreće
11, 12, . . . , 99 ima 90. Od njih je 30 djeljivo kabinu lifta je 6 kW. Kolika je korisnost motora
s 3 (povoljni slučajevi za dogadaj - A ), 13 sa
ako je masa kabine, zajedno s ljudima u njoj,
-
7 (povoljni slučajevi za dogadaj B ) i 4 su 500 kg?
djeljiva i s 3 i sa 7 (povoljni slučajevi za AB ).
Zato je Rješenje.
p(A + B) = p(A) + p(B) − p(AB) h = 4 · 3 m = 12 m
30 13 4 13 t = 12 s
= + − = = 0.43.
90 90 90 30 Pu = 6 kW
Edin Ajanović (3), Sarajevo m = 500 kg
η =?
3118. Nadi- sumu reda
X∞ G = m · g = 500 kg · 10 N/ kg
2n
2n , G = 5000 N
a +1
n=0
W = G · h = 5000 N · 12 m
gdje je a > 1 .
W = 60 000 J
Rješenje. Za a > 1 imamo W 60 000 J
Pk = =
2n 2n (a2n − 1) 2n (a2n + 1) − 2n + 1 t 12 s
= 2n+1 =
a2
n
+1 a −1 a2 −1
n+1 Pk = 5000 W = 5 kW

112 Matematičko-fizički list, LIX 2 (2008. – 2009.)


Pk 5 kW A1 = a · a = 2 cm · 2 cm
η= =
Pu 6 kW
A1 = 4 cm2
η = 0.83 = 83%
Arijan Golub (8), A = 4 · A1
OŠ Augusta Cesarca, Krapina A = 16 cm2 = 0.0016 m2

OŠ – 279. Napravi njihalo od konca i G=F =p·A


utega proizvoljne mase. Odredi period takvog G = 500 000 Pa · 0.0016 m2
njihala za duljine konca 10, 20, 30, 40
i 50 cm. Nacrtaj dijagram koji prikazuje G = 800 N
ovisnost perioda njihala o duljini konca. Je li Gs = ms · g = 4 kg · 10 N/ kg
ta ovisnost linearna?
Gs = 40 N
Rješenje. Gč = G − Gs
t Gč = 760 N
l (cm) N t (s) T= (s)
N
G 760 N
50 10 14.3 1.43 mč = č =
g 10 N/ kg
40 10 12.6 1.26
30 10 11.1 1.11 mč = 76 kg
20 10 9.53 0.95
Juraj Krsnik (8),
10 10 6.8 0.68
OŠ Augusta Cesarca, Krapina

OŠ – 281. Keopsova piramida je najstarije


od sedam starih svjetskih čuda. Sagradena -
je oko 2560 g.p.n.e. To je najveća i
-
najteža gradevina -
ikad sagradena na Zemlji.
-
Sagradena je od 2 300 000 kamenih blokova
prosječne mase 2.5 tona. Sad je visoka
135.75 metara, a duljina stranica osnovice
iznosi 229 metara. Izračunaj koliki postotak u
ukupnom volumenu piramide otpada na kamen,
a koji na prostorije i hodnike. Gustoća kamena
Ovisnost perioda o duljini niti nije linearna. krečnjaka od kojeg su isklesani blokovi iznosi
2600 kg/m3 .
Ana Paić (7),
OŠ Fausta Vrančića, Šibenik Rješenje.
m = 2.5 t · 2 300 000
OŠ – 280. Stolica ima četiri noge
kvadratnog presjeka. Njihova je debljina 2 cm. = 5 750 000 t = 5 750 000 000 kg
Masa stolice je 4 kilograma. Kolika bi trebala h = 135.75 m
biti masa čovjeka koji bi sjeo na tu stolicu da
tlak pod njenim nogama iznosi 0.5 MPa? a = 229 m
ρ = 2600 kg/ m3
Rješenje.
Volumen piramide:
n=4
h
a = 2 cm Vp = S ·
3
ms = 4 kg h
Vp = a · a ·
p = 0.5 MPa = 500 000 Pa 3
135.75
mč =? Vp = 229 · 229 ·
3

Matematičko-fizički list, LIX 2 (2008. – 2009.) 113


v
Vp = 2 372 955 m3 ω= ,
R−r
Volumen kamena: vR
ρ=
m v = Rω = .
V R−r
m
V1 = Ur.
ρ
2500 kg
V1 = 1393. Dva tijela masa m1 i m2 spojena
2600 kg/ m3 su krutom šipkom duljine l i postavljena
V1 = 0.96 m3 na horizontalnu podlogu. Koeficijenti trenja
izmedu - tijela i podloge su μ1 i μ2 . Šipka
Vk = 0.96 · 2 300 000 je zanemarive mase dok su tijela zanemarivo
Vk = 2 208 000 m3 malenih dimenzija u odnosu na duljinu šipke.
U središtu šipke djeluje sila iznosa F okomito
Vk 2 208 000 m3 na nju i paralelno s horizontalnom podlogom
= = 0.93 = 93%
Vp 2 372 955 m3 kao na slici. Koliki mora biti iznos te sile da
93% otpada na kamen, a 7% na prostorije i bi šipka ostala okomita na smjer sile?
hodnike.
Ana Paić (7),
OŠ Fausta Vrančića, Šibenik
1392. Konac namotan na kalem unutarnjeg
radijusa r i vanjskog R leži na horizontalnoj
površini i može se kotrljati bez klizanja.
Kolikom brzinom i u kojem smjeru će se kalem
gibati ako se kraj konca povlači brzinom v
kao na slici?
Rješenje. Sile trenja na mase m1 i m2
djeluju u smjeru suprotnom od sile F . Težište
sistema nalazi se u točki koja je za
lm2
l1 =
m1 + m2
udaljena od m1 ; udaljenost težišta od m2 je
lm1
l2 = .
m1 + m2
Rješenje. Kalem se kotrlja bez klizanja, pa Bez gubljenja općenitosti, možemo pretposta-
mora postojati sila trenja Ftr koja djeluje u viti da je m1 > m2 (težište se tada nalazi
suprotnom smjeru od brzine v (s hvatištem u lijevo od centra, tj. polovice šipke). Ukupan
točki dodira s podlogom). Budući da se kalem moment sile oko težišta je:
„ «
ne ubrzava, konac se povlači silom F istog l
iznosa (dakako, suprotnog smjera). Moment M = Ftr1 l1 − Ftr2 l2 + F − l1
2
sile trenja bit će veći od momenta sile F , pa
lm2 lm1
će kalem rotirati u smjeru kazaljke na satu. Za = μ1 m1 g − μ2 m2 g
vrijeme t kraj konca će se pomaknuti za v · t , m1 + m2 m1 + m2
„ «
centar kalema će se pomaknuti za v t = ω · R · t +F
l

lm2
(gdje je ω kutna brzina rotacije kalema), a 2 m1 + m2
konac će se dodatno “zamotati” za duljinu „ «
lm m l lm2
ω · r · t . Stoga vrijedi: = g 1 2 (μ1 −μ2 ) + F − .
m1 +m2 2 m1 +m2
v · t − ω · R · t = −ω · r · t,
Ako želimo da šipka ostane okomita na smjer
v = ω · R − ω · r, sile, moment sile mora biti jednak nuli (u

114 Matematičko-fizički list, LIX 2 (2008. – 2009.)


protivnom će doći do rotacije šipke): f (x + y + 2f ) = (x + f )(y + f ),
M = 0 =⇒ f x + f y + 2f 2 = xy + f x + f y + f 2 ,
„ «
lm1 m2 l lm2 f 2 = xy,
g (μ1 − μ2 ) = −F − ,
m1 + m2 2 m1 + m2
„ « što je i trebalo pokazati.
lm1 m2 l lm2
F=g (μ2 − μ1 ) : − Kristijan Kvaternik (1),
m1 + m2 2 m1 + m2 V. gimnazija, Zagreb
2lm1 m2
=g (μ2 − μ1 ). - dvije identične
l (m1 + m2 ) − 2lm2 1396. Udaljenost izmedu
metalne kuglice na kojima su različite količine
Ur.
naboja je 2 cm. Radijusi kuglica su mnogo
-
manji od njihove medusobne udaljenosti.
1394. Dijete mase m njiše se na njihaljci
-
Privlačna sila izmedu kuglica je 4 · 10−5 N.
duljine l s kutom maksimalnog otklona od
ravnotežnog položaja α . Dijete viče i time Nakon što su kuglice spojene vodljivom žicom
-
i zatim odspojene, one se medusobno odbijaju
proizvodi zvuk frekvencije f . Koji raspon
frekvencija čuje roditelj koji stoji ispred silom od 2.25·10−5 N. Odredite početne naboje
njihaljke daleko u odnosu na svoju visinu? na kuglicama.

Rješenje. Maksimalnu brzinu njihanja dijete Rješenje.


postiže u trenutku kada je u najnižem položaju F1 = −4 · 10−5 N
jer se tada sva potencijalna gravitacijska
energija pretvara u kinetičku: F2 = 2.25 · 10−5 N
mv2 r = 0.02 m
mgl − mgl cos α = ,
2 Q1 , Q2 =?
p
v = 2gl(1 − cos α ). Bez smanjenja općenitosti može se pretpostaviti
Minimalnu frekvenciju računamo pomoću Dop- da je kuglica naboja Q1 prije dodira nabijena
plerovog izraza u situaciji kada se dijete tom pozitivno, a kuglica s nabojem Q2 negativno.
brzinom udaljava od roditelja, a maksimalnu Tada je:
kada mu se približava: 1 Q1 Q2
vzvuka F1 = ,
f=f 4πε0 r2
vzvuka ± v
Q1 Q2 = 4πε0 r2 F1 .
v
=f pzvuka . Pri spajanju kuglica došlo je do prelaska
vzvuka ± 2gl(1 − cos α ) -
odredenog pozitivnog naboja Q s kuglice s
Ur. nabojem Q1 na kuglicu s nabojem Q2 , tako da
je naboj na kuglicama nakon spajanja jednak
1395. Dokaži da je za sferična zrcala (kuglice su identične; dakle, imaju jednaki
umnožak udaljenosti predmeta i slike od žarišta kapacitet).
uvijek jednak kvadratu žarišne duljine. 1 (Q1 − Q)(Q2 + Q)
F2 = ,
Rješenje. Označimo li s a i b udaljenosti 4πε0 r2
predmeta i slike od tjemena zrcala, s x i y Q1 − Q = Q2 + Q.
udaljenosti predmeta i slike od fokusa, vrijedi:
Ove tri jednadžbe čine sistem s tri nepoznanice
a = x+f, b= y+f. (Q, Q1 , Q2 ). Eliminiranjem Q2 dobivamo:
Iz jednadžbe sfernog zrcala: (Q1 − Q)2 = 4πε0 r2 F2 ,
1 1 1
= + Q1 (Q1 − 2Q) = 4πε0 r2 F1 .
f a b
imamo: Oduzimanjem ove dvije jednadžbe dobivamo:
f (a + b) = ab, Q2 = 4πε0 r2 (F2 − F1 ),

Matematičko-fizički list, LIX 2 (2008. – 2009.) 115


Q = 1.67 · 10−9 C,
p Rješenja zabavne matematike
Q1 = 4πε0 r2 F2 + Q = 2.67 · 10−9 C,
Q2 = Q1 − 2Q = −0.67 · 10−9 C.
Ur. Tanja je nabrojila 61 pravokutni trokut, a
Perica 40 paralelograma.
1397. Električna žarulja od 110 V i 60 W
spojena je na bateriju elektromotorne sile od
120 V. Unutarnji otpor baterije je 60 Ω . Hoće Prema riječima profesora Kubusa lako
li žarulja svijetliti maksimalnim intenzitetom u zaključujemo sljedeće:
ovakvom spoju?
Svaki matematičar dopisuje se na istom
Rješenje. jeziku s barem tri druga. Od ove trojice ili se
U = 110 V, dva dopisuju na tome istom jeziku, ili se sva
trojica dopisuju drugim jezikom.
P = 60 W,
Svaki matematičar dopisuje se na istom
E = 120 V, jeziku s barem šest drugih. Od ove šestorice
r = 60 Ω. ili se dva dopisuju na tom istom jeziku, ili se
sva šestorica dopisuju na ostala dva jezika, a
Otpor žarulje računamo iz njenih karakteristika: tada se zaključivanje izvodi kao u prethodnom
U2 U2 slučaju.
P = UI = =⇒ R = = 201.7 Ω.
R P Dakle, postoje najmanje tri matematičara
Struja koja će teći kroz žarulju kada je spojimo -
koji se medusobno dopisuju na istom jeziku.
na bateriju je:
E
I= = 0.46 A.
R+r Broj osvojenih bodova je 190. Ako je
Na žarulji će se razvijati snaga a broj bodova posljednjeg na tablici i r
razlika bodova svaka dva susjedna igrača,
P = I 2 R = 42.7 W,
onda su igrači osvojili redom a , a + r ,
što je manje od nominalne snage. Dakle, žarulja a + 2r ,. . . , a + 19r bodova. Zbroj tih bodova
neće svijetliti maksimalnim intenzitetom. je 20a + 190r , pa mora biti 20a + 190r = 190 .
Ur. Jedini par prirodnih brojeva koji zadovoljava
ovu jednakost je a = 0 , r = 1 . Pobjednik je
1398. Dvije identične metalne kugle zagri- osvojio 19 bodova, a posljednji na tablici 0
javaju se tako da im se polumjer poveća za bodova.
isti iznos. Jedna od njih leži na stolu, a druga
je obješena o nit. Obje kugle su toplinski izo-
lirane od stola i niti. Kojoj kugli je dovedeno
više topline i zašto?
Rješenje. Kugli koja leži na stolu pri
povećavanju polumjera raste i gravitacijska
potencijalna energija (jer joj se težište podiže)
pa je ukupna toplina koju joj treba dovesti
veća! Učenici su otkrili da se broj 666 666, nakon
Ur. nalaženja njegova rastava na proste faktore u
obliku 666 666 = 2 · 3 · 3 · 7 · 11 · 13 · 37 , može
na tri načina prikazati kao umnožak dvaju
troznamenkastih brojeva:
666 666 = 693 · 962 = 777 · 858 = 814 · 819.

116 Matematičko-fizički list, LIX 2 (2008. – 2009.)


Matematička konferencija u čast akademiku Josipu Pečariću

Sanja Varošanec 1 , Zagreb

Sudionici matematičkih natjecanja često se susreću s jednom posebnom vrstom


zadataka – s matematičkim nejednakostima. Nejednakost trokuta, Cauchy-Schwarz-
Bunjakowskijeva nejednakost, Jensenova, Holderova, Minkowskijeva nejednakost,
nejednakost izmedu - aritmetičke i geometrijske sredine, te nejednakosti izmedu
- drugih
sredina, samo su neke od osnovnih nejednakosti koje uz puno mašte i blistavih ideja
koristimo pri rješavanju takvih zadataka. No, nejednakosti se ozbiljno proučavaju i
u matematičkoj znanstvenoj zajednici. Široj je javnosti možda manje poznato da u
Zagrebu živi i djeluje jedno od najvećih svjetskih matematičkih imena iz područja
matematičkih nejednakosti – akademik Josip Pečarić. Činjenica da je u 2008. godini
-
profesor Pečarić navršio 60. godina bila je povodom za organizaciju medunarodne
konferencije Mathematical Inequalities and Application 2008 (MIA 2008). Konferencija
čija je organizacija počela dvije godine prije održana je od 8. do 14. lipnja 2008.
u Trogiru uz sudjelovanje 135 matematičara iz 27 zemalja – od sjevera Europe pa
do Australije. Malo je koja matematička konferencija u Hrvatskoj privukla toliki broj
sudionika iz cijeloga svijeta.

Mathematical Inequalities and Applications 2008


June 8 -14, 2008, Trogir, Croatia
Conference in honour of Professor Josip Pečarić on the occasion of his 60 th birthday
Plenary Speakers Scientific Committee
Peter S. Bullen Marko Matić
Lars-Erik Persson (Chairman)
Victor Burenkov Ingram Olkin
Sever S. Dragomir Zsolt Páles Shoshana Abramovich
Roland Glowinski Ivan Perić Ravi P. Agarwal
Tsuyoshi Ando
Frank Hansen Lars-Erik Persson Drumi Bainov
Alois Kufner András Prékopa Catherine Bandle
Lech Maligranda Gord Sinnamon Richard A. Brualdi
Peter S. Bullen
Roy Mathias Vladimir D. Stepanov Victor Burenkov
Wing-Sum Cheung
Yeol Je Cho
Neven Elezović
Carla Giordano
Roland Glowinski
Pankaj Jain
Anna Kaminska
Alois Kufner
Andrea Laforgia
Chi-Kwong Li
Lech Maligranda
Marko Matić
Ram N. Mohapatra
Organizing Committee Edward Neuman
Constantin Niculescu
Aleksandra Čižmešija Ludmila Y. Nikolova
Neven Elezović Ryskul Oı̆narov
Milica Klaričić Bakula Ingram Olkin
Marko Matić Bohumir Opic
Sanja Varošanec Charles E. M. Pearce
Ivan Perić
András Prékopa
Saburou Saitoh
Gord Sinnamon
Hari M. Srivastava
Vladimir D. Stepanov
George P. H. Styan
Nenad Ujević
INFORMATION: http://mia2008.ele-math.com CONTACT: mia2008@math.hr DEADLINE FOR ABSTRACTS: May 1, 2008 Sanja Varošanec
Faculty of Natural Sciences, Mathematics and Kinesiology, University of Split Faculty of Textile Technology, University of Zagreb Croatian Mathematical Society foto: Jelena Popić

Okosnicu konferencije činilo je 15 jednosatnih predavanja pozvanih znanstvenika –


P. S. Bullen (Kanada) osvrnuo se na najzanimljivija Pečarićeva otkrića, V. Burenkov
(Velika Britanija) iznio je najnovije rezultate o multiplikativnim višedimenzionalnim
nejednakostima u težinskim Lebesgueovim prostorima, S. S. Dragomir (Australija) dao je

1 Dr. sc. Sanja Varošanec je redovita profesorica PMF-Matematičkog odjela Sveučilišta u Zagrebu.

Matematičko-fizički list, LIX 2 (2008. – 2009.) 117


pregled aproksimacija Riemann-Stieltjesovog integrala, R. Glowinski (USA) diskutirao
je rješenja neglatkog problema svojstvenih vrijednosti, o operatorskim nejednakostima s
primjenama predavanje je održao F. Hansen (Danska), a o nejednakostima Hardyjevog
tipa A. Kufner iz Češke. Norme izmedu - kvazi-Banachovih Lp - Lq prostora bile
su tema predavanja L. Maligrande ( Švedska), a M. Matić (Split) pozabavio se
Eulerovim tipovima nejednakosti. Profesor I. Olkin (USA) u svom je predavanju
napravio zanimljivo povezivanje nejednakosti izmedu - momenata, sredina, matričnih i
probabilističkih nejednakosti, a o nejednakostima za konveksne funkcije višeg reda čuli
smo od A. Prekope (USA). Sredine i to generalizirane kvazi-aritmetičke bile su tema
-
predavanja Zs. Palesa (Madarska), a mješovite sredine bile su tema predavanja I. Perića
(Zagreb). Predsjednik znanstvenog odbora konferencije Lars-Erik Peršon ( Švedska)
održao je predavanje o Hardyju i njegovom uplivu na teoriju nejednakosti, dok je
G. Sinnamon prezentirao rezultate o težinskim Fourierovim nejednakostima, a V. D.
Stepanov (Rusija) o nejednakostima za integralne operatore.

Ostali sudionici konferencije svoje su rezultate izložili u vidu kratkih 20-minutnih


priopćenja. Zbog velikog broja prijavljenih priopćenja (njih 107) rad je bio organiziran
u tri paralelne sekcije, a sve se odvijalo svakodnevno od 9 sati ujutro pa do
večeri, uz pauzu za ručak. Predavanja su obuhvatila gotovo sve aspekte matematičkih
nejednakosti. Izlagalo se o analitičkim nejednakostima, o geometrijskim, integralnim,
operatorskim i matričnim, o varijacijskim, o nejednakostima u apstraktnim strukturama, o
primjenama nejednakosti u metodi konačnih elemenata, u ocjenama numeričkih formula,
pri rješavanju funkcijskih jednadžbi, nelinearnih sustava diferencijalnih jednadžbi, te
o problemima konveksnog programiranja. A naravno, održati konferenciju u Trogiru
i ne obići neke znamenitosti u okolici bilo bi nedopustivo. Sudionici konferencije
posjetili su Nacionalni park Krka, te obišli Split i Trogir. Na stranici konferencije
(http://mia2008.ele-math.com) nalaze se detalji o konferenciji, a tamo je osim
službenih informacija moguće naći i niz slika koje dočaravaju atmosferu.
Budući da je konferencija MIA 2008 organizirana u čast 60. rodendana - profesora
Pečarića, razumljivo je da su se predavači u velikoj mjeri dotakli i rezultata rada
profesora Pečarića što je potpuno i razumljivo budući da je on jedan od vodećih
stručnjaka u području matematičkih nejednakosti i jedan od najproduktivnijih svjetskih
matematičara s više od 700 objavljenih ili prihvaćenih za tisak znanstvenih radova, sa 70-
ak radova na konferencijama i u knjigama, 60-ak iz elektrotehnike, geofizike, geologije,
-
gradevinarstva, fizike i povijesti, 14 matematičkih monografija te dva visokoškolska

118 Matematičko-fizički list, LIX 2 (2008. – 2009.)


udžbenika. Bio je deseti matematičar u svijetu koji je na MathSciNetu imao preko 500
radova. Tako, na primjer, ISI Web of Knowledge u bazi SCIE prikazuje da se radovi
J. Pečarića citiraju 542 puta u 310 radova u časopisima te baze, a u monografiji P.
S. Bullena Handbook of Means and Their Inequalities, Mathematics and Applications
(Kluwer, 2003) Pečarić je najcitiraniji autor (7 knjiga i 105 članaka). Ovi brojevi
citiranosti kazuju koliki je odjek pojedinih radova u znanstvenoj zajednici, tj. koliko se
puta drugi matematičari pozivaju na te radove. Njegovim je imenom nazvano nekoliko
identiteta i nejednakosti, a o primjenama Mond-Pečarićeve metode na operatore u
Hilbertovim prostorima objavljena je monografija, dok se druga upravo priprema. Za
njegov je rad karakteristična suradnja – preko 150 suradnika iz 26 zemalja, a pod
njegovim je vodstvom izradeno - ili je u izradi 30-tak magistarskih i doktorskih radnji.
Poznato je da svega četrdesetak matematičara u svijetu ima preko sto suradnika.
- 2. rujna 1948. u Kotoru. Tamo je završio osnovnoškolsko
Josip je Pečarić roden
i srednjoškolsko obrazovanje, a 1972. je diplomirao na Elektrotehničkom fakultetu u
Beogradu s radom iz područja nuklearne fizike. Na istom je fakultetu i magistrirao, a
1982. obranio je doktorsku disertaciju iz područja matematike s naslovom Jensenove
i srodne nejednakosti. U komisiji su bili profesori Boas i Ky Fan. Dio Boasovog
izvješataja navodimo u originalu:
Let me say to begin with that I would consider Pečarić’s thesis acceptable in
any american university, and a better thesis than most of thesis I have seen.
Pečarić has, in the first place, systematized a large amount of material that
appeared in a haphazard way over the years and had become rather chaotic. This
synthesis is a valuable piece of work in itself.
In the second place, Pečarić has shown great ingenuity in finding simpler proofs
of some inequalities and appropriate generalizations of others. Consequently he
has made many inequalities more understandable. He has also been able to unify
some inequalities that originally seemed to be quite unrelated.
Finally, he has made many original contributions to the field.
Do 1987. godine radio je u Geomag-
netskom institutu u Grockoj te na Grade- -
vinskom fakultetu u Beogradu, a onda s
obitelji preseljava u Zagreb te se zapoš-
ljava na Tekstilno-tehnološkom fakultetu
Sveučilišta u Zagrebu gdje i danas radi
kao redoviti profesor. Od tada do danas
nekoliko je puta kao gostujući profesor bo-
ravio na sveučilištima u Australiji, Italiji,
Pakistanu i Švedskoj. Godine 1998. pok-
-
renuo je medunarodni znanstveni časopis
Mathematical Inequalities and Applicati-
ons (izdavač Element, Zagreb) koji je već
nakon dvije godine izlaženja uvršten na Scientific Citation Index Expanded (SCIE) listu,
a prije par godina pokrenuo još dva časopisa Journal of Mathematical Inequalities i
Operators and Matrices. Uredništva ovih triju časopisa čini 90-ak uglednih matematičara.
Osim toga objavio je 21 publicističku knjigu – od knjige Srpski mit o Jasenovcu pa do
Za hrvatske vrednote i preko 200 članaka u kojima progovara i poučava o povijesti i
sadašnjosti Hrvatske zastupajući ideje samostalne Hrvatske.
Za svoj doprinos u znanosti dobio je Državnu nagradu za znanost za 1996. godinu,
te je odlikovan ordenom Reda Danice hrvatske s likom R. Boškovića. Od 2000. godine
profesor Pečarić redoviti je član Hrvatske akademije znanosti i umjetnosti.

Matematičko-fizički list, LIX 2 (2008. – 2009.) 119


-
49. medunarodna matematička olimpijada

-
Ovogodišnja Medunarodna matematička olimpijada (IMO) održana je u Madridu, u
srpnju 2008. Sudjelovalo je 535 natjecatelja iz 97 država. Hrvatsku su predstavljali Nina
Kamčev (XV. gimnazija, Zagreb), Adrian Satja Kurdija, (V. gimnazija, Zagreb), Ines
Marušić (V. gimnazija, Zagreb), Sara Muhvić (III. gimnazija, Osijek), Melkior Ornik
(XV. gimnazija, Zagreb) i Goran Žužić (V. gimnazija, Zagreb). Voditelji ekipe bili su
Željko Hanjš i Mea Bombardelli s Matematičkog odjela PMF-a u Zagrebu.
Učenici su krenuli u Madrid 14. srpnja zajedno s voditeljicom Meom Bombardelli,
dok je voditelj ekipe Željko Hanjš otputovao ranije u Španjolsku i u gradu La Granja
sudjelovao u izboru zadataka za natjecanje. Tamo nas je dočekalo jako španjolsko sunce
-
što nas nije spriječilo da već prvi dan s vodičem podemo u šetnju gradom.
Natjecatelji su bili smješteni u nekoliko studentskih domova. Na istom katu s nama
su bile smještene i ekipe iz Makedonije i Vijetnama. S Makedoncima smo otpočetka
ostvarili veoma dobru komunikaciju, dok je s Vijetnamcima bilo malo teže jer njihov
engleski nije bio ni upola tako dobar kao njihovo znanje matematike. Sljedeći dan, na
svečanom otvaranju, upoznali smo se i s ostalim ekipama iz naše regije – Bosancima,
Crnogorcima, Slovencima i Srbima, s kojima smo se i kasnije družili.
Na otvaranju smo, osim tradicionalnih govora o važnosti matematike, imali prilike
vidjeti i nekoliko cirkuskih nastupa. Sutradan je bio prvi dan natjecanja, na kojem su
zadaci, po mišljenju mnogih sudionika, bili lakši nego inače, ali je ipak bilo teško
nakon prvog dana osigurati bilo kakvu medalju. Nakon drugog dana natjecanja počelo
je spekuliranje o bodovima koje smo postepeno saznavali tijekom sljedeća dva dana,
dok smo išli na izlete promatrajući razne ljepote Španjolske, kao što su Segovia, San
Lorenzo de Escorial ili Toledo.
Treći dan nakon natjecanja na Internetu su bili objavljeni cjelokupni rezultati: Adrian
Satja Kurdija, Goran Žužić i Melkior Ornik osvojili su brončane medalje (prag je bio 15
bodova, a oni su imali redom 20, 19 i 17), dok su Nina Kamčev i Sara Muhvić dobile
pohvalu za točno riješen jedan zadatak (i to na različitim zadacima!). Po ukupnom
broju osvojenih bodova Hrvatska je bila 38., što je najbolji uspjeh u zadnje tri godine.
Kao i obično, (po broju osvojenih bodova) pobijedili su Kinezi s pet zlatnih medalja
i jednom srebrnom, dok je druga bila ekipa Rusije, čiji je svaki predstavnik osvojilo
zlatnu medalju.
Uvijek se može bolje – Nini (koja je nakon drugog dana rješavanja zadataka
-
otputovala u Vijetnam na Medunarodnu olimpijadu iz fizike, gdje je osvojila brončanu
medalju) je nedostajao tek bod za broncu, a Adrianu dva boda za srebro. Ipak, sve to
može se nadomjestiti sljedeće godine, kada će se održavati jubilarna 50. IMO u Bremenu
u Njemačkoj.
Trojica učenika ostvarila su maksimalan broj bodova – dvojica iz Kine i jedan iz
SAD-a (ali kineskog porijekla!). Oni su na svečanosti zatvaranja dobili pljesak veći čak
i od princa od Asturije, koji je došao da bi dodjeljivao medalje i svojom prisutnošću
uveličao svečanost.
Nakon zatvaranja išli smo na zajednički banket i druženje na Univerzitetu Carlos III.
Od raznih ekipa dobili smo sitne poklone za uspomenu, a i mi smo drugima dijelili
olovke. Mnogima je bilo teško oprostiti se od prijatelja i simpatija koje su stekli na ovoj
olimpijadi, ali sve je prošlo u prijateljskoj atmosferi. Bilo je lijepo vratiti se u Hrvatsku
i nadati se još boljem uspjehu na sljedećoj Olimpijadi.
Adrijan Satja Kurdija i Melkior Ornik

120 Matematičko-fizički list, LIX 2 (2008. – 2009.)


Zadaci

Prvi dan

Španjolska, srijeda, 16. srpnja 2008.


1. Točka H je ortocentar šiljastokutnog trokuta ABC . Kružnica sa središtem u
polovištu tranice BC prolazi kroz H i siječe pravac BC u A1 i A2 . Slično, kružnica sa
središtem u polovištu od CA prolazi kroz H i siječe pravac CA u B1 i B2 ; kružnica sa
središtem u polovištu od AB prolazi kroz H i siječe pravac AB u C1 i C2 . Dokaži da
sve točke A1 , A2 , B1 , B2 , C1 , C2 leže na kružnici.
(Rusija)
2. a) Dokaži da vrijedi:
x2 y2 z2
+ + ≥1
(x − 1)2 (y − 1)2 (z − 1)2
za sve realne brojeve x , y, z, od kojih je svaki različit od 1, i zadovoljavaju uvjet
xyz = 1 .
b) Dokaži da jednakost vrijedi za beskonačno mnogo trojki racionalnih rojeva x , y,
z, od kojih je svaki različit od 1, i zadovoljavaju uvjet xyz = 1 .
(Austrija)
3. Dokaži da postoji beskonačno mnogo√prirodnih brojeva n takvih da je n2 + 1
djeljivo prostim brojem koji je veći od 2n + 2n.
(Litva)

Drugi dan

Španjolska, srijeda, 17. srpnja 2008.


4. Nadi- sve funkcije f : (0, ∞) → (0, ∞) ( f je funkcija sa skupa pozitivnh ealnih
brojeva u skup pozitivnh realnih brojeva) takvih da vrijedi
(f (w))2 + (f (x))2 w2 + x2
= ,
f (y2 ) + f (z2 ) y2 + z2
za sve pozitivne realne brojeve w, x , y, z, koji zadovljavaju uvjet wx = yz.
(Republika Koreja)
5. Neka su n i k prirodni brojevi takvi da je k ≥ n i k − n paran broj. Dano je 2n
žarulja označenih s 1 , 2 , . . . , 2n i svaka od njih može biti ili upaljena ili ugašena. Na
početku su sve žarulje ugašene. Promatraj nizove koraka: u svakom koraku točno jedna
žarulja promijeni svoje stanje (ako je bila upaljena, ugasi se, a ako je bila ugašena, upali
se). Neka je N broj takvih nizova od kojih svaki ima k koraka i na kraju su sve žarulje
od 1 do n upaljene,dok su sve žarulje od n + 1 do 2n ugašene. Neka je M broj nizova
od kojih svaki ima k koraka, i na kraju su sve žarulje od 1 do n upaljene,dok su sve
žarulje od n + 1 do 2n ugašene, i nijedna od žarulja n + 1 do 2n nije se niti palila niti
N
gasila. Odredi omjer .
M
(Francuska)

Matematičko-fizički list, LIX 2 (2008. – 2009.) 121


6. Neka je ABCD konveksan četverokut takav da je |BA| = |BC|. Označimo kružnice
upisane rokutima ABC i ADC s ω1 i ω2 , tim redom. Pretpostavi da postoji kružnica
ω koja dodiruje polupravac BA u točki iza A i polupravac BC u točki iza C , a isto
tako dodiruje pravce AD i CD. Dokaži da se zajedničke vanjske tangente kružnica ω1
i ω2 sijeku na kružnici ω .
(Rusija)
Vrijeme za rad svakog dana: 4 sata i 30 minuta
Svaki zadatak vrijedi 7 bodova

Rang–lista
nagrade broj nagrade broj

I II III poh. bod. I II III poh. bod.


Kina 5 1 217 Bosna i Hercegovina 3 1 68
Rusija 6 199 Slovenija 2 1 68
SAD 4 2 190 Švicarska 1 1 2 68
Južna Koreja 4 2 188 Švedska 1 3 67
Iran 1 5 181 Danska 2 1 66
Tajland 2 3 1 175 Kostarika 2 3 65
Sjeverna Koreja 2 4 173 Malezija 1 4 65
Turska 3 1 2 170 Austrija 1 4 63
Tajvan 2 4 168 Norveška 1 1 62
-
Madarska 2 3 1 165 Belgija 1 1 1 61
Japan 2 3 1 163 Makedonija 2 61
Vijetnam 2 2 2 159 Luksemburg (5) 2 2 60
Poljska 2 3 1 157 -
Tadikistan 1 3 60
Bugarska 2 1 3 154 Litva 1 2 58
Ukrajina 2 2 2 153 Makau 2 1 58
Brazil 5 1 152 Maroko 1 2 58
Peru 1 3 2 141 Armenija 4 56
Rumunjska 4 2 141 Portugal 2 1 55
Australija 5 1 140 Albanija 1 1 53
Njemačka 1 2 3 139 Čile (3) 1 1 1 49
Srbija 1 3 2 139 Irska 2 45
Kanada 2 4 135 Cipar 1 1 42
Ujedinjeno Kraljevstvo 4 2 134 Novi Zeland 3 42
Italija 3 3 132 Estonija 1 1 41
Kazahstan 1 2 3 128 Finska 1 1 40
Bjelorusija 3 2 125 Bangladeš (4) 1 33
Izrael 1 1 2 2 120 Island (5) 1 31
Hong Kong 3 1 1 107 Salvador (4) 3 31
Mongolija 2 1 2 105 Šri Lanka 1 29
Francuska 1 4 1 104 Trinidad i Tobago 1 28
Indija 5 1 103 Kirgistan 5 1 28
Singapur 1 3 1 98 Kuba (1) 1 27
Nizozemska 2 2 94 Ekvador 1 26
Uzbekistan 4 94 Kambodža 1 25
Litva 1 2 3 92 Crna Gora (3) 2 24
Indonezija 1 2 2 88 Paragvaj (4) 1 24
Meksiko 1 1 4 87 Filipini 1 23
Hrvatska 3 2 86 Urugvaj (5) 1 22
Argentina 1 3 85 Tunis (4) 1 20
Češka 1 1 3 85 Honduras (2) 2 17
Grčka 2 4 85 Gvatemala (4) 1 16
Gruzija 5 1 84 Lihtenštajn (2) 1 16
Španjolska 3 3 82 Venecuela (2) 1 16
Južnoafrička Republika 1 4 79 Portoriko (3) 9
Kolumbija 2 1 77 Saudijska Arabija 8
Slovačka 3 1 76 Bolivija (5) 5
Turkmenistan 4 1 76 Ujedinjeni Arapski Emirati (4) 5
Azerbajdžan 3 74 Kuvajt (5) 3
Moldavija 1 2 74

122 Matematičko-fizički list, LIX 2 (2008. – 2009.)


Druga srednjoeuropska matematička olimpijada,
Olomouc, Češka, 4. – 10. rujna 2008.

Ilko Brnetić, Zagreb

Prošle smo godine s velikim zadovoljstvom dočekali radanje - novog regionalnog


natjecanja u kojem sudjelujemo – Srednjoeuropske matematičke olimpijade. Ove smo
godine imali priliku potvrditi uspješan nastavak jedne lijepe priče.
Ekipu za natjecanje smo birali u dvije faze i uz dosta zajedničkih priprema. Na
Državnom natjecanju smo uz odabir šestero učenika za Medunarodnu - matematičku
olimpijadu, odabrali i širi krug od dvanaest potencijalnih kandidata za nastup na
Srednjoeuropskoj matematičkoj olimpijadi u Olomoucu. Nakon dijela dopisnih i kratkih
zajedničkih priprema u dva navrata održanih u Zagrebu, održano je i izborno dvodnevno
natjecanje. U ekipu su se konačno plasirali: Ivana Antoliš (II. razred), Ivo Božić (II.
razred), Matko Ljulj (I. razred) i Irma Telarović (III. razred) iz XV. gimnazije u Zagrebu,
Grgur Valentić (I. razred) iz V. gimnazije u Zagrebu i Borna Cicvarić (II. razred)
iz Gimnazije Andrije Mohorovičića u Rijeci. Moramo naglasiti da ostali kandidati
nisu mnogo bodovno zaostali u tom zanimljivom natjecanju, kao i to da sudionici
-
medunarodne matematičke olimpijade i maturanti, po pravilu, nisu imali pravo nastupa
na ovom natjecanju. Nakon izbora pripreme su nastavljene do početnog tjedna mjeseca
srpnja, a nakon ljetnog odmora, članovi ekipe okupili su se u Zagrebu nekoliko dana
prije polaska na put.
Put u Olomouc, jedno od vodećih središta pokrajine Moravske, vodio nas je preko
Slovenije i Austrije. Zahvaljujući nenadanim popravcima na pruzi izmedju Maribora i
Graza, umjesto predvidena - dva presjedanja, imali smo ih čak pet, no, nakon 11 sati
vožnje stigli smo na odredište. Nakon toga slijedilo je pet i pol dana intenzivnog rada,
natjecanja, druženja,. . .
Natjecanje i smještaj bili su organizirani u Sveučilištu Jan Palacky. Ove godine
natjecalo se svih devet država koje su dogovorom i inicirale ovo natjecanje. Upravo je
-
sudjelovanje Madarske i Njemačke, koje prošle godine nisu sudjelovale u natjecanju,
pridonijelo značajnom jačanju kvalitete natjecanja koje se sastoji od dva dijela:
pojedinačnog i ekipnog.
U pojedinačnom dijelu sudjelovalo je 52 natjecatelja, po 6 iz svake države sudionice,
samo iz Slovenije ih je sudjelovalo četvero. Naši su učenici osvojili tri brončane medalje
(Ivo Božić, Matko Ljulj i Grgur Valentić) i jednu pohvalu (Borna Cicvarić). Ivo Božić
bio je nadomak srebrne medalje, nedostajao mu je tek bod, a Borna Cicvarić je jedini
od naših učenika riješio najteži, geometrijski zadatak. U ekipnom dijelu natjecanja naši
su učenici bili tek osmi, no uz neveliki zaostatak za bolje plasiranim ekipama.
Rezultatski gledano, najuspješnije su bile tri objektivno najjače ekipe, Madarska, -
Poljska i Njemačka, dok su sve ostale, izuzev Slovenije, imale vrlo slične rezultate. Mi
smo s time vrlo zadovoljni, jer smo imali najmladu - ekipu na natjecanju.
Proveli smo vrlo ugodne dane u Češkoj; domaćini su priredili bogat program izleta,
tako da smo uz upoznavanja vrlo lijepog Olomouca, grada bogate povijesti i kulture,
upoznali Kromeriž, grad sa spomenicima pod zaštitom UNESCO-a, poznat ponajprije
po dvorcu i lijepim vrtovima, lijepu špilju Javoričko, te dvorac Bouzov.
Sljedeće godine, natjecanje će se održati u Poljskoj.

Matematičko-fizički list, LIX 2 (2008. – 2009.) 123


Na kraju navodimo i zadatke s natjecanja (uz napomenu da je vrijeme rješavanja
svakog dana bilo 5 sati), kao i rezultate.

Zadaci s pojedinačnog natjecanja

1. Neka je (an )∞n=1 niz prirodnih brojeva takav da je an < an+1 , ∀n  1 . Za


svaku četvorku indeksa (i, j, k, m), gdje je 1 ≤ i < j ≤ k < m i i + m = j + k , vrijedi
nejednakost ai + am > aj + ak . Odredi najmanju moguću vrijednost broja a2008 .

2. Promatramo šahovsku ploču dimenzija n × n , gdje je n > 1 prirodan broj. Na


koliko načina se može postaviti 2n − 2 jednakih kamenčića na tu šahovsku ploču (na
različita polja) tako da nikoja dva od njih ne leže na istoj dijagonali šahovske ploče?
(Dva su kamenčića na istoj dijagonali šahovske ploče ako su središta tih polja paralelna
jednoj od dijagonala n × n kvadrata.)

3. Neka je ABC jednakokračan trokut u kojem je |AC| = |BC|. Njegova upisana


kružnica dodiruje AB i BC u točkama D i E redom. Pravac (različit od AE ) prolazi
kroz A i siječe upisanu kružnicu u F i G. Pravac AB siječe pravce EF i EG redom u
K i L . Dokaži da je |DK| = |DL|.

4. Odredi sve cijele brojeve k takve da su za sve cjelobrojne n , brojevi 4n + 1 i


kn + 1 relativno prosti.

Zadaci s ekipnog natjecanja

- sve funkcije f : R → R takve da vrijedi


1. Nadite
xf (x + xy) = xf (x) + f (x2 )f (y)
za sve realne brojeve x i y.

2. Na ploči je napisano n ≥ 2 prirodnih brojeva. U svakom koraku biramo par


brojeva s ploče i zamijenimo svaki od njih njihovom sumom. Odredite sve vrijednosti
n za koje možemo, nakon konačnog broja koraka, dobiti n -torku s jednakim brojevima.

3. Dan je šiljastokutni trokut ABC . Neka je E točka sa suprotne strane pravca AC


u odnosu na točku B, i neka je D točka unutar dužine AE . Pretpostavimo da vrijedi
<)ADB = < )CDE , < )BAD = < )ECD i < )ACB = <)EBA. Dokažite da točke B, C i E
leže na istom pravcu.

4. Neka je n prirodan broj. Ako je suma svih pozitivnih djelitelja broja n potencija
- potencija broja 2 . Dokažite!
broja 2 , onda je i broj tih djelitelja takoder

124 Matematičko-fizički list, LIX 2 (2008. – 2009.)


Rezultati pojedinačnog i ekipnog natjecanja

zlatna srebrna brončana broj


država poredak država
medalja medalja medalja bodova
-
Madarska 3 3 0 1. Poljska 32
Poljska 1 4 1 1. -
Madarska 32
Njemačka 1 3 1 1. Njemačka 32
Češka 0 1 1 4. Austrija 26
Hrvatska 0 0 3 5. Slovačka 25
Švicarska 0 0 3 6. Švicarska 24
Austrija 0 0 3 7. Češka 22
Slovačka 0 0 2 8. Hrvatska 21
Slovenija 0 0 0 9. Slovenija 9



Matematičko-fizički list, LIX 2 (2008. – 2009.) 125


-
Medunarodno matematičko natjecanje
“Klokan bez granica” 2008. g.

-
Medunarodno matematičko natjecanje “Klokan bez granica” odr-
žalo se deseti put pod pokroviteljstvom Ministarstva prosvjete i
športa i Hrvatskog matematičkog društva 20. travnja ove godi-
ne. S približno istim zadacima natjecalo se 5 000 000 učenika
u 42 zemlje svijeta, što ovo natjecanje čini najvećim školskim
natjecanjem u svijetu. Prema odjecima koji su stigli do nas,
vjerujemo da je natjecanje postiglo svoju svrhu i zainteresiralo
učenike za rješavanje zadataka iz matematike. U Hrvatskoj su se
natjecali učenici u 320 osnovnih i 120 srednjih škola u svim županija u šest kategorija:
Leptirići, Ecolier, Benjamin, Cadet, Junior, Student. Ukupno se natjecalo 29 806
učenika.
Sudjelovalo je 6743 učenika II. i III. razreda osnovne škole (L), 8365 učenika IV. i
V. razreda osnovne škole (E), 6166 učenika VI. i VII. razreda osnovne škole (B), 4526
učenika VIII. razreda osnovne i I. razreda srednje škole (C). 2935 učenika II. i III.
razreda srednje škole (J) i 1071 učenik IV. razreda srednje škole (S).
Najveći broj bodova koji se mogao ostvariti bio je 72 u kategoriji Leptirići, te 120
bodova za sve ostale kategorije. Prilikom dolaska na natjecanje svaki je učenik dobio
poklon, a 10% najboljih natjecatelja dobili su nagrade. Bodovni prag najboljih 10%
natjecatelja bio je: za “Leptiriće” – 50 bodova; “Ecolier” – 87.50 bodova; “Benjamin”
– 77.50 bod; “Cadet” – 55.00 bodova; “Junior” – 60.25 bodova; “Junior” – matematički
program 75.00 bodova; “Student” 48.75 bodova; “Student” – matematički program 66.25
bodova.
Sljedeći zadaci mogu vas upoznati s prošlogodišnjem natjecanjem i korisno poslužiti
za pripreme za novo natjecanje koje će se održati 19. ožujka 2009. godine.
Koordinator natjecanja “Klokan bez granica”, Neda Lukač, prof.

Zadaci za učenike 8. razreda osnovne škole i 1. razreda srednje škole (Cadet)

Pitanja za 3 boda:
1. U gusarskoj školi, svaki učenik mora sašiti svoju crno-bijelu zastavu. Pri tome
mora biti ispunjen uvjet da crni dio čini tri petine zastave. Koliko od prikazanih zastava
ispunjava taj uvjet?

A. nijedna B. jedna C. dvije D. tri E. četiri

2. U razrednom odjeljenju ima 9 dječaka i 13 djevojčica. Polovina djece u odjeljenju


ima prehladu. Koliko djevojčica najmanje ima prehladu?
A. 0 B. 1 C. 2 D. 3 E. 4

126 Matematičko-fizički list, LIX 2 (2008. – 2009.)


3. 6 klokana pojedu 6 vreća sijena u 6 minuta. Koliko klokana će pojesti 100 vreća
sijena za 100 minuta?

A. 100 B. 60 C. 6 D. 10 E. 600
4. Brojeve 2, 3, 4 i još jedan nepoznati prirodni broj treba upisati
u kvadratiće na slici. Zbroj brojeva u prvom retku mora biti 9, a zbroj
brojeva u drugom retku 6. Nepoznati broj je

A. 5 B. 6 C. 7 D. 8 E. 4
5. Trokut i kvadrat na slici imaju jednake opsege. Koliki je
opseg cijelog lika (peterokuta)?

A. 12 cm B. 24 cm C. 28 cm D. 32 cm
E. ovisno o duljinama stranica trokuta 4cm

6. Cvjećarka Rina ima 24 bijele, 42 crvene i 36 žutih ruža. Koliko najviše jednakih
kitica ona može složiti ako želi upotrijebiti sve ruže?

A. 4 B. 6 C. 8 D. 10 E. 12
7. Koliko se kvadrata može nacrtati spajanjem točaka na slici?

A. 2 B. 3 C. 4 D. 5 E. 6

8. Tri pravca prolaze istom točkom. Veličine dvaju


kutova su 108◦ i 124◦ , kao što se vidi na slici. Koliko
stupnjeva ima kut osjenčan sivom bojom? 108q
124q
◦ ◦ ◦ ◦ ◦
A. 52 B. 53 C. 54 D. 55 E. 56

Pitanja za 4 boda:

9. Na kocki su odsiječeni vrhovi, kao na slici. Koliko bridova


ima “nova” figura na slici?

A. 26 B. 30 C. 36 D. 40 E. neki drugi odgovor


10. Danijel ima 9 novčića (svaki vrijednosti 2 lipe), a njegova sestra Ana 8 novčića
-
vrijednosti 5 lipa svaki. Koliko najmanje novčića trebaju razmijeniti medusobno Danijel
i Ana da bi imali jednake novčane iznose?

A. 4 B. 5 C. 8 D. 12 E. nije moguća takva razmjena


11. Tom i Jerry razrezali su 2 sukladna pravokutnika. Tom je dobio 2 pravokutnika,
svaki opsega 40 cm, a Jerry 2 pravokutnika, svaki opsega 50 cm. Koliki je bio opseg
početnih pravokutnika?

A. 40 cm B. 50 cm C. 60 cm D. 80 cm E. 90 cm

Matematičko-fizički list, LIX 2 (2008. – 2009.) 127


12. Jedna strana kocke razrezana je po njenim dijagonalama (kao na
slici). Koja od sljedećih mreža nije mreža kocke na slici?

1 2 3 4 5

A. 1 i 3 B. 1 i 5 C. 3 i 4 D. 3 i 5 E. 2 i 4
13. Na pravcu su istaknute točke A, B, C i D. Poznate su sljedeće duljine:
|AB| = 13 , |BC| = 11 , |CD| = 14 i |DA| = 12 . Kolika je udaljenost dviju najudaljeni-
jih točaka?

A. 14 B. 38 C. 50 D. 25 E. drugi odgovor
14. U pravokutnik su upisane 4 kružnice Q P
duljine polumjera 6 cm, kao na slici. Točka P je
vrh pravokutnika, a točke Q i R dirališta kružnica
i stranica pravokutnika. Kolika je površina trokuta
PQR? R
A. 27 cm2 B. 45 cm2 C. 54 cm2 D. 108 cm2 E. 180 cm2
15. U kutiji se nalazi 7 karata. Na kartama su napisani brojevi od 1 do 7 (na svakoj
karti točno jedan broj). Matija uzima iz kutije nasumce 3 karte, a Luka 2 karte (2 su
karte ostale u kutiji ). Tada Matija kaže Luki: “Znam da je zbroj brojeva na tvojim
kartama paran”. Zbroj brojeva Matijinih karata iznosi:

A. 10 B. 12 C. 6 D. 9 E. 15
16. Francuski matematičar August de Morgan imao je x godina u godini x2 . Umro
je 1899. godine. Kada se rodio de Morgan?

A. 1806. B. 1848. C. 1849. D. 1899. E. drugi odgovor

Pitanja za 5 bodova:
17. Simetrala kuta <) C uz osnovicu |BC| jednakokračnog trokuta ABC siječe krak
AB u točki D. Ako je |BC| = |CD|, kolika je veličina kuta <
)CDA?

A. 90◦ B. 100◦ C. 108◦ D. 120◦ E. nemoguće je odrediti


3
18. Drvena kocka 11 × 11 × 11 nastala je ljepljenjem 11 jediničnih kocaka. Koliko
se najviše jediničnih kocaka može vidjeti gledajući iz iste točke gledanja?

A. 328 B. 329 C. 330 D. 331 E. 332


19. U “Malim astronomima” djevojke čine više od 45%, a manje od 50%. Koji je
najmanji mogući broj djevojaka u toj grupi?

A. 3 B. 4 C. 5 D. 6 E. 7

128 Matematičko-fizički list, LIX 2 (2008. – 2009.)


20. Dječak uvijek govori istinu četvrtkom i petkom, uvijek laže utorkom, a ostale
dane u tjednu govori istinu ili laže bez pravila. Pitali su ga, sedam dana uzastopno, za
njegovo ime i odgovori prvih 6 dana bili su: Ivan, Branko, Ivan, Branko, Petar, Branko.
Što je dječak odgovorio sedmog dana?

A. Ivan B. Branko C. Petar D. Katarina E. drugi odgovor


21. Martina i Ivica su krenuli planinariti. U selu, u podnožju planine, pročitali su
oznaku na kojoj piše da do vrha ima 2 sata i 55 minuta (pješačenja). Napustili su
selo u 12 sati. U 13 sati stali su radi kratkog odmora i pročitali novu oznaku na kojoj
piše da do vrha ima samo 1 sat i 15 minuta. Nakon 15 minuta nastavili su put is-
tom brzinom kao i prije i nisu se više zaustavljali do vrha. U koje su vrijeme stigli na vrh?

A. 14 : 30 B. 14 : 00 C. 14 : 55 D. 15 : 10 E. 15 : 20
22. Nazovimo tri prosta broja “specijalnim” ako je njihov umnožak 5 puta veći od
njihovog zbroja. Koliko “specijalnih” trojki prostih brojeva postoji?

A. 0 B. 1 C. 2 D. 4 E. 6
23. Zadana su dva skupa A i B peteroznamenkastih prirodnih brojeva. U skupu A
su brojevi čiji je umnožak svih znamenki 25, a u skupu B brojevi čiji je umnožak svih
znamenki 15. Koji skup ima više brojeva? Koliko puta više?
5 5
A. skup A, puta B. skup A, 2 puta C. skup B, puta
3 3
D. skup B, 2 puta E. oba skupa imaju jednaki broj članova
24. Najveći zajednički djelitelj dvaju prirodnih brojeva m i n je 12, a njihov najmanji
zajednički višekratnik je kvadrat. Izmedu - 5 brojeva: n , m , n , m , m · n koliko njih
3 3 4 4
su kvadrati?

A. 1 B. 2 C. 3 D. 4 E. nemoguće je odrediti

Zadaci za učenike 2. i 3. razreda srednje škole (Junior)

Pitanja za 3 boda:
1. Dano je 5 kutija, a u svakoj od njih nalaze se kartice sa slovima A, E , I , O, U
kao što je prikazano na slici. Petar je izbacio neke kartice tako da je na kraju svaka
kutija sadržavala jednu karticu, a različite kutije sadržavale su različita slova. Koja je
kartica ostala u kutiji broj 2?

A. A B. E C. I D. O E. U

Matematičko-fizički list, LIX 2 (2008. – 2009.) 129


2. Franjo i Gabrijela natjecali su se u trčanju na 200 metara. Gabrijela je stazu
pretrčala za pola minute, a Franjo za jednu stotinu sata. Tko je i za koliko sekundi bio
brži?

A. Gabrijela za 36 sekundi B. Franjo za 24 sekunde C. Gabrijela za 6 sekundi


D. Franjo za 4 sekunde E. Imaju ista vremena
3. Na proslavi ovogodišnje Nove godine, Borna je obukao majicu s brojem godine i
pred ogledalom izveo je stoj na rukama. Koju je sliku u ogledalu vidio njegov prijatelj
Vinko stojeći iza Borne, normalno na nogama?

A. B. C. D. E.

4. Zadani su brojevi a = 2 − (−4), b = (−2) · (−3), c = 2 − 8 , d = 0 − (−6) i


e = (−12) : (−2). Koliko od njih nije jednako broju 6?

A. 0 B. 1 C. 2 D. 4 E. 5

5. Kolika je duljina dužine AB, ako je duljina B


stranice svakog od kvadrata na slici 1 metar?
√ √ √
A. 5 B. 13 C. 5 + 2

D. 5 E. nijedna od danih

A
6. Koliko se najmanje slova može maknuti iz riječi KANGOUROU tako da su
preostala slova abecedno poslagana?

A. 1 B. 2 C. 3 D. 4 E. 5
7. U kutiji je 7 karata. Brojevi od 1 do 7 napisani su na njima i to točno jedan broj
na jednoj karti. Prvi igrač iz kutije na slučajan način odabere 3 karte, a drugi igrač uzme
2 karte. Tada prvi igrač kaže drugome: “Znam da je zbroj brojeva na tvojim kartama
paran.” Koliki je zbroj brojeva na kartama prvog igrača? (Igrači ne vide preostale karte
u kutiji.)

A. 10 B. 12 C. 6 D. 9 E. 15
8. Tom i Jerry dobili su jednake papire u obliku pravokutnika. Svaki od njih je svoj pa-
pir razrezao na dva jednaka pravokutnika. Tom je izrezao pravokutnike s opsegom 40 cm,
a Jerry dva pravokutnika s opsegom 50 cm. Koliki je bio opseg početnog komada papira?

A. 40 cm B. 50 cm C. 60 cm D. 80 cm E. 100 cm
Pitanja za 4 boda:
9. Na prvom testu Ana je osvojila 1 bod od mogućih 5. Na sljedećim testovima
osvojila je sve bodove od mogućih 5. Koliko je još testova trebala pisati da joj prosječan
broj bodova bude 4 po testu?

A. 2 B. 3 C. 4 D. 5 E. 6

130 Matematičko-fizički list, LIX 2 (2008. – 2009.)


10. Marko ima 10 kartica, a na svakoj od njih se nalazi točno jedan od ovih brojeva:
3, 8, 13, 18, 23, 28, 33, 48, 53, 68. Koliko najmanje kartica Marko može izabrati tako
da zbroj brojeva na njima bude jednak 100?

A. 2 B. 3 C. 4 D. 5 E. to je nemoguće učiniti
-
11. Sedam je patuljaka rodeno istog dana u 7 uzastopnih godina. Tri najmlada -
zajedno imaju 42 godine. Koliko godina zajedno imaju tri najstarija patuljka?

A. 51 B. 54 C. 57 D. 60 E. 63
12. Dva pravilna šesterokuta na slici su jednaka. Ko-
lika je površina osjenčanog dijela paralelograma?
1 1 1 1 1
A. B. C. D. E.
2 3 4 5 6
13. Šest prirodnih brojeva označeno je slovima na brojevnom pravcu. Poznato je da su
barem dva od njih djeljivi s 3 i barem dva od njih djeljivi s 5. Koji su brojevi djeljivi s 15?

A B C E D F

A. A i F B. B i D C. C i E D. svi E. samo jedan od njih


14. Koliko najviše znamenaka treba obrisati iz 1000-znamenkastog broja
20082008. . . 2008 tako da zbroj preostalih znamenaka bude 2008?

A. 746 B. 510 C. 524 D. 1020 E. 130


15. Na slici je dan jednakokračan trokut u kojem je
|AB| = |AC|. Ako je PQ okomito na AB, kut BPC je 120◦ i
kut ABP je 50◦ , koliki je kut PBC ?

A. 5◦ B. 10◦ C. 15◦ D. 20◦ E. 25◦


16. Koliko postoji parova realnih brojeva čiji su zbroj, produkt
-
i kvocijent medusobno jednaki?

A. nema takvih parova B. 1 par C. 2 para D. 4 para E. 8 parova


Pitanja za 5 bodova:
17. Svaka znamenka počevši od treće (brojano slijeva) u šesteroznamenkastom
broju jednaka je zbroju prethodne dvije znamenke (koje se nalaze slijeva). Koliko
šesteroznamenkastih brojeva ima to svojstvo?

A. nijedan B. 1 C. 2 D. 4 E. 6
18. Imam drvenu kocku čije tri strane su plave, a tri crvene. Kad je prerežem u
3 × 3 × 3 = 27 jednakih malih kocaka, koliko od njih ima barem dvije različitim bojama
obojane strane

A. 6 B. 12 C. 14 D. 16
E. ovisi o tome koje su strane velike kocke bile obojane plavo i crveno

Matematičko-fizički list, LIX 2 (2008. – 2009.) 131


19. Označimo s n! = 1 · 2 · 3 · . . . · (n − 1) · n . Ako je n! = 215 · 36 · 53 · 72 · 11 · 13 ,
tada je n jednako

A. 13 B. 14 C. 15 D. 16 E. 17
20. Kolika je duljina istaknutog (većeg) luka na kružnici
polumjera 1?
5π 5π π 3π 2π
A. B. C. D. E.
4 3 2 2 3
21. Kvadrat 4 × 4 podijeljen je na 16 jediničnih kvad-
- najveći broj dijagonala koje se mogu povući u
ratića. Nadi
tim kvadratićima tako da nikoje dvije nemaju zajedničke točke.

A. 8 B. 9 C. 10 D. 11 E. 12
22. Kanga uvijek skače po 1 m ili po 3 m. Želi skočiti točno 10 metara. Na koliko
načina može Kanga izvesti to skakanje? (Skokove 1 + 3 + 3 + 3 i 3 + 3 + 3 + 1 smatramo
različitim načinima.)

A. 28 B. 34 C. 35 D. 55 E. 56
23. Kvadrat ABCD na slici ima stranicu duljine 1, i lu-
kovima su spojeni odgovarajući nasuprotni vrhovi kvadrata.
Kolika je udaljenost točaka P i Q?

√ 3 √ √ 3 √
A. 2 − 2 B. C. 5 − 2 D. E. 3 − 1
4 2

24. Koliko postoji 2007-teroznamenkastih brojeva s ovim svojstvom: svaki dvozna-


menkasti broj sačinjen od dvije uzastopne znamenke početnog broja je djeljiv ili sa 17
ili s 23?

A. 5 B. 6 C. 7 D. 9 E. više od 9

Zadaci za za učenike 4. razreda srednje škole (Student)

Pitanja za 3 boda:
1. Brojevi 3, 4 i još dva nepoznata broja upisani su u polja tablice 2× 2. Poznato je
da je zbroj brojeva po retcima jednak 5 i 10, a zbroj po jednom stupcu je 9. Veći od ta
dva nepoznata broja je

A. 5 B. 6 C. 7 D. 8 E. 3
2008
x
2. Ako je x + y = 0 i x = 0 , tada je =
y2008
x
A. −1 B. 0 C. 1 D. 22008 E.
y

132 Matematičko-fizički list, LIX 2 (2008. – 2009.)


3. Matrica se sastoji od 21 stupca označenih brojevima 1,2,. . . ,21 i 33 retka nume-
riranih brojevima 1,2,. . . ,33. Obrišemo one retke čiji redni broj nije višekratnik od 3 i
sve one stupce čiji redni broj je paran. Koliko je u matrici ostalo polja?

A. 110 B. 121 C. 115.5 D. 119 E. 242


4
4. Koliko prostih brojeva p ima svojstvo da je i broj p + 1 prost broj?

A. nijedan B. 1 C. 2 D. 3 E. beskonačno
5. Promatramo rijeku od točke 2
A. U svom toku račva se na dva
1
3 8
2 A
rukavca. Prvi rukavac preuzima
3
količine vode, a drugi ostatak. Nakon 5
8
toga se prvi rukavac opet račva u tri, 1
pri čemu jedan novi rukavac preuzima vode,
8 B
5
drugi , a treći ostatak. Nakon nekog vremena, taj treći se
8
rukavac spaja s onim rukavcem rijeke nastalim pri prvom račvanju. Mapa rijeke
prikazana je na slici. Kolika količina vode dolazi u mjesto B?
1 5 2 1 1
A. B. C. D. E.
3 4 9 2 4
6. U kutiji je 7 karata. Brojevi od 1 do 7 napisani su na njima i to točno jedan broj
na jednoj karti. Prvi igrač iz kutije na slučajan način odabere 3 karte, a drugi igrač uzme
2 karte. Tada prvi igrač kaže drugome: “Znam da je zbroj brojeva na tvojim kartama
paran.” Koliki je zbroj brojeva na kartama prvog igrača?

A. 10 B. 12 C. 6 D. 9 E. 15
7. Dan je jednakokračan trokut ABC (|CA| = |CB|).
Na osnovici je označena točka D tako da je |AD| = |AC|
i |DB| = |DC|. Koliki je kut ACB?

A. 98◦ B. 100◦ C. 104◦ D. 108◦ E. 110◦

8. Najveća vrijednost funkcije f (x) = |5 sin x − 3| u skupu R iznosi:

A. 2 B. 3 C. π D. 5π E. 8
Pitanja za 4 boda:
9. Svaka od kocaka na slici ima bridove duljine 1. Kolika je
udaljenost od točke A do točke B?
√ √ √ √
A. 17 B. 7 C. 13 D. 7 E. 14

10. Matilda je nacrtala 36 klokana koristeći tri različite boje. 25 klokana ima nešto
- a za 20 klokana je koristila crnu boju. Samo 5
obojano žutom, 28 ih ima nešto smede,
od njih ima na sebi sve tri boje. Koliko klokana imaju na sebi samo jednu boju?

A. 0 B. 4 C. 12 D. 31 E. nemoguće je odrediti

Matematičko-fizički list, LIX 2 (2008. – 2009.) 133


11. Tri se kruga diraju u parovima kao na slici.
Polumjer svakoga je r . Kolika je površina područja A
-
omedenog s ta tri kruga?
√ √
π 2 π 3 2 1
A. 3− r B. − r C. π r2
2 2 2 √ 8
√ 3  π
3 2
D. 3− π r2 E. − r § ·
2 3 2
12. Dva pravilna šesterokuta na slici su jednaka. Ko-
lika je površina osjenčanog dijela paralelograma?
1 1 1 1 1
A. B. C. D. E.
2 3 4 5 6
13. Na natjecanju Matematički Kup zadano je 5 problema koji svi nose različiti broj
bodova. Damjan je riješio svih pet i pri tome je osvojio svih 10 bodova koje je bilo
moguće osvojiti na dva problema s najmanjim brojem bodova. Osim toga, za dva proble-
ma s najvećim brojem bodova dobio je svih 18 bodova. Koliko je bodova osvojio Damjan?

A. 30 B. 32 C. 34 D. 35 E. 40
14. Brojnik i nazivnik razlomka su negativni cijeli brojevi. Brojnik je veći od
nazivnika. Koja od sljedećih tvrdnji o tom razlomu je istinita?

A. Razlomak je manji od −1 . - −1 i 0.
B. Razlomak je izmedu
C. Razlomak je pozitivan broj manji od 1. D. Razlomak je veći od 1.
E. Ne može mu se odrediti predznak.
15. Tri su točke izabrane na slučajan način iz skupa točaka na
slici. Kolika je vjerojatnost da su te točke kolinearne?
1 1 1 1 3
A. B. C. D. E.
12 11 16 8 12
16. Ako je x2 yz3 = 73 i xy2 = 79 , tada je xyz =

A. 74 B. 76 C. 78 D. 79 E. 710
Pitanja za 5 bodova:
17. Duljine stranica kvadra u centrimetrima su prirodni brojevi koji čine geometrijski
niz s kvocijentom 2. Koji od brojeva mogu biti obujam tog kvadra?

A. 120 cm2 B. 188 cm2 C. 216 cm2 D. 350 cm2 E. 500 cm2
18. U ovom računu svaka zvjezdica označava jednu znamenku.
Koliki je zbroj svih znamenaka u rezultatu ovog množenja? ×* * *
1**
A. 16 B. 20 C. 26 D. 30 E. neki drugi odgovor
22**
- vrijednost izraza x2 + y2 + z2 , ako je x + y + z = 1 i
19. Nadi + 90*
1 1 1
+ + = 0. **2
x y z
56***
A. 0 B. 1 C. 2 D. 3 E. nemoguće je odrediti

134 Matematičko-fizički list, LIX 2 (2008. – 2009.)


20. Kružnica je upisana trokutu ABC , pri čemu je |AC| = 5 ,
|AB| = 6 i |BC| = 3 . Dužina ED dodiruje kružnicu. Koliki je
opseg trokuta ADE ?

A. 7 B. 4 C. 9 D. 6 E. 8
21. Niz je zadan ovako: an+1 = an + (−1)n · n , za n  1 , i
a1 = 0 . Ako je ak = 2008 , koliki je k ?

A. 2008 B. 2009 C. 4017 D. 4018 E. neki drugi


22. Broj 332 − 1 ima točno dva djelitelja koji su veći od 75 i manji od 85. Koliki je
produkt tih djelitelja?

A. 5852 B. 6560 C. 6804 D. 6888 E. 6972


23. Kvadrat ABCD ima stranice duljine 1. Ako je točka M
polovište stranice AB, kolika je površina osjenčanog područja?
1 1 1 1 2
A. B. C. D. E.
24 16 8 12 13
24. Ako je sin x + cos x = m, tada je sin4 x + cos4 x =
(1 − m2 )2 (1 − m2 )2 1 − (1 − m2 )2
A. 1 − B. 1 + C. D. m4 E. m4 + 1
2 2 2

Rješenja

Cadet
1. C 2. C 3. C 4. D 5. D 6. B 7. C 8. A
9. C 10. B 11. C 12. D 13. D 14. D 15. B 16. A
17. C 18. D 19. C 20. A 21. B 22. B 23. B 24. B

Junior
1. D 2. C 3. B 4. B 5. B 6. D 7. B 8. C
9. B 10. D 11. B 12. A 13. A 14. A 15. A 16. B
17. D 18. E 19. D 20. D 21. A 22. A 23. E 24. A

Student
1. B 2. C 3. B 4. B 5. D 6. B 7. D 8. E
9. A 10. B 11. A 12. A 13. D 14. C 15. B 16. A
17. C 18. A 19. B 20. E 21. C 22. B 23. D 24. A


Obavijesti o ovom natjecanju mogu se dobiti na internetskoj stranici
http://www.math.hr/hmd.

Matematičko-fizički list, LIX 2 (2008. – 2009.) 135


Žarko Dadić, Egzaktne znanosti u Hrvata u poslijeprosvjetiteljskom razdoblju
(1789.–1835.), edicija: Posebna izdanja, urednik: Milan Mirić, Naklada Ljevak, Zagreb,
2007., 326 str.
Najnovijom knjigom, o kojoj je ovdje riječ, akademik Žarko
Dadić obradovao je sve one koji su očekivali i napokon dočekali da
se objelodane rezultati njegovog dugotrajnog ustrajnog i predanog
znanstveno-istraživačkog rada. U knjizi se tako objavljuju izvorna
autorova istraživanja koja je on obavljao tijekom više od dvadeset
godina. Ova knjiga se nadovezuje na knjigu Egzaktne znanosti
u Hrvata u doba prosvjetiteljstva koju je godine 2004. objavila
Matica hrvatska a ujedno je jedna od niza knjiga u kojima autor
- cjelokupnu povijest egzaktnih znanosti u Hrvata. Razdoblje
obraduje
od Francuske revolucije do hrvatskog narodnog preporoda, koje je
-
obradeno u ovoj knjizi, u pogledu egzaktnih znanosti u Hrvata bilo
je dosad najmanje poznato. Da bi užitak pri čitanju bio što veći
pobrinuli su se, uz autora i urednika, recenzenti dr. sc. Mijo Korade i dr. sc. Trpimir
Macan. Uz skraćeni upadljivi naslov na omotu korica knjige otisnut je fotografski
prikaz u boji armilarnih sfera iz zbirke obitelji Garanjanin u Trogiru. Sfere se čuvaju u
Gradskom muzeju u Trogiru.
U predgovoru knjige autor je opisao dogadanje - u filozofiji, znanosti, književnosti i
kulturi nakon Francuske revolucije 1789. pa do 1830.–1840., u Francuskoj, Njemačkoj
i Habsburškoj monarhiji. To razdoblje moglo se s jednog motrišta označiti kao
postjozefinsko, a drugoga kao pretpreporodno, uz napomenu da hrvatski narodni
preporod počinje 1835. godine. Autor se stoga odlučio da to razdoblje označi kao
poslijeprosvjetiteljsko budući da je najveći dio knjige posvetio prirodnofilozofskim i
prirodoznanstvenim zbivanjima s poslijeprosvjetiteljskog motrišta. Još jedan razlog za to
je i činjenica da se ta knjiga nadovezuje na spomenutu knjigu iz 2004., i u odredenom -
smislu s njom čini cjelinu.
Autor je dao detaljan uvod u tematiku, naime prikazao je osobe važne za razvoj
prirodnofilozofskih i prirodnoznanstvenih shvaćanja u poslijeprosvjetiteljskom dobu, a
koje su djelovale u Francuskoj i Njemačkoj. Isto tako naglašena je njihova uloga i
obrazovnom sustavu (škole!). Škole u kojima su te osobe djelovale utemeljile su novi
model razvitka znanstvenih istraživanja. Autor je naglasio i utjecaj tih škola na visoke
škole u Habsburškoj monarhiji što je značajno za hrvatske znanstvenike koji su djelovali
u zemljama monarhije, a izvan Hrvatske.
U većem dijelu knjige su analizirana gotovo sva djela hrvatskih znanstvenika koji
su djelovali na sveučilištu u Budimu, odnosno Pešti, te na akademijama u Madarskoj - i
Slovačkoj u tom razdoblju. Knjiga pokazuje da je na Filozofskom fakultetu u Budimu,
odnosno u Pešti mnogo Hrvata u tom razdoblju predavalo matematiku i fiziku, a da ih
je mnogo bilo i na Zvjezdarnici u Budimu. Tako su, izmedu - ostalih, na Filozofskom
fakultetu predavali Ivan Paskvić, Franjo Josip Domin i Franjo Bruna, a u jednom
razdoblju su na zvjezdarnici djelovali Mirko Danijel Bogdanić, Franjo Bruna i Ivan
Paskvić. Osim toga je autor vrlo iscrpno prikazao rad Ignjata Martinovića u matematici
i fizici, koji je bio najprije u franjevačkom redu, a kad je izašao iz njega bio je profesor
u Lavovu. Autor temeljito ispravlja neispravne tvrdnje o etničkoj pripadnosti Ignjata

136 Matematičko-fizički list, LIX 2 (2008. – 2009.)


Martinovića za kojega dokazuje da je hrvatskog podrijetla. Autor je pronašao mnoge
dosad nepoznate rukopise spomenutih znanstvenika, posebno u Sveučilišnoj knjižnici
u Budimpešti i u knjižnici zvjezdarnice Madarske- akademije. Na temelju tiskane i
rukopisne grade - prikazan je cjelokupni rad spomenutih hrvatskih znanstvenika u tom
razdoblju te je pokazana geneza njihova doprinosa. Na temelju njihove korespondencije
su prikazani njihovi kontakti sa znanstvenicima u velikim znanstvenim središtima u
Europi. Osobito je na temelju izvora po prvi put opsežno razjašnjem sukob hrvatskog
astronoma Paskvića koji je početkom 19. stoljeća bio ravnatelj zvjezdarnice u Budimu s
njegovim asistentom Danielom Kmethom, pri čemu su najveći astronomi toga vremena,
primjerice Gauss, dokazali da je Paskvića neosnovano napao Kmeth i da ga je htio
znanstveno diskreditirati. Korišteni su gotovo svi astronomski časopisi u kojima su se
objavljivali rezultati motrenja hrvatskih astronoma na zvjezdarnici u Budimu u to doba.
-
U knjizi je obradena i znanstvena situacija u Hrvatskoj u to doba, posebno na Kraljevskoj
akademiji u Zagrebu. Obradeni - su rukopisi sačuvani u franjevačkim knjižnicama te je
analizirano školstvo u hrvatskim krajevima. Posebna je pozornost posvećena promjeni
programa nastave u školama u Dalmaciji nakon što su je okupirali Francuzi, a onda i
nakon Austrijske okupacije kad je školstvo u Dalmaciji ujednačeno s programima koje
je vrijedilo za sjevernu Hrvatsku.
U pogovoru naglašava se značenje 1835. godine na svjetskoj pozornici znanstvenih
zbivanja kada prirodne znanosti odbacuju metafiziku, a unutar njih pojavljuju se
specijalnosti. Autor je, uz to, istaknuo da i u matematici od tog vremena počinje jedno
sasvim novo razdoblje. Godina 1835. značajna je i za Hrvatsku, jer se usporedno
s političkim djelovanjem pojavljuju i nastojanja za uspostavu nacionalnih kulturnih
i znanstvenih ustanova. Od tada hrvatski znanstvenici uglavnom djeluju u domovini,
osnivaju domaće znanstvene ustanove i društva, i započinju sustavni i moderni znanstveni
rad u Hrvatskoj – zaključio je autor.
Na kraju valja istaći da u pogledu sustavnosti, detaljnosti i cjelokupnosti ova
knjiga nadilazi povijesno-znanstveni i povijesno-filozofski pristup. To je vrhunski prikaz
nastanka egzaktne novovjeke znanosti u Hrvata u spomenutom razdoblju. Zbog toga je
knjiga ne samo nezaobilazna literatura u proučavanju te tematike, nego je jedinstvena u
pogledu sintetičkog pristupa. Nakladniku pripada zasluga što je izdavanjem ove knjige
uvelike obogatio istraživanje prave povijesne istine našeg naroda.
Branko Hanžek, Zagreb



PAŽNJA! — STARI BROJEVI — U našem skladištu ima starih brojeva, i to: god.
XVI, br. 4; god. XXXII, br. 3; god. XXXIII, br. 4; god. XXXIV, br. 3, 4; god. XXXV,
br. 3; god. XXXVI, br. 1, 2, 3, 4; god. XXXVII, br. 1, 4; god. XXXIX, br. 1, 2, 3, 4;
god. XL, br. 2, 3, 4; god. XLI, br. 1, 2, 3, 4; god. XLII, br. 3-4; god. XLIV, br. 1, 2, 3,
4; god. XLV, br. 1, 2, 3, 4; god. XLVI, br. 1, 2, 3, 4; god. XLVII, br. 1, 2, 3, 4; god.
XLVIII, br. 1, 2, 3, 4; god. XLIX, br. 1, 2, 3, 4; god. L, br. 1, 2, 3, 4; god. LI, br. 1, 2,
3, 4; god. LII, br. 1, 2, 3, 4; god. LIII, br. 1, 2, 3, 4; god. LIV, br. 1, 2, 3, 4; god. LV,
br. 1, 2, 3, 4; god. LVI, br. 1, 2, 3, 4; god. LVII, br. 1, 2, 3, 4; god. LVIII, br. 1, 2, 3, 4.
Cijena pojedinog broja je 20 kuna.
Izvanredni broj (E) – zadaci iz matematike (cijena 20 kn); Izvanredni broj (F) –
Rječnik matematičkih naziva – hrvatski, engleski, njemački (cijena 30 kn); Izvanredni
broj (H) – zadaci iz matematike (cijena 20 kn).

Matematičko-fizički list, LIX 2 (2008. – 2009.) 137


Razredbeni ispit na FOI-u iz matematike 2008. g.

Donosimo zadatke s razredbenog ispita na Sveučilištu u Zagrebu, Fakultet organizacije


i informatike, Varaždin, 2008. g. Do 2008. godine bilo je po 20 zadataka iz matematike,
informatike i hrvatskog jezika, a od 2009. godine će se sastojati od 30 zadataka iz
matematike i 30 zadataka iz informatike. Svaki točno riješen zadatak nosi 10 bodova
i nema negativnih bodova za netočno riješene zadatke. U svakom zadatku ima 4
-
ponudena odgovora od kojih je jedan točan. Za prijelaz praga treba skupiti najmanje
200 bodova, ali mora biti ispunjen i uvjet da je 100 bodova minimalno iz matematike
i 100 iz informatike. Dakle, ako bi pristupnik imao samo 200 bodova i to samo iz
informatike, on ne bi prešao prag. Više informacija o upisima na FOI, možete naći na
http://www.foi.hr/upisi/. Ovdje donosimo 60 zadataka iz matematike, koji su bili
na razredbenom ispitu 2008. godine.

Grupa A

   
xy + y2 1 + a3 1 − a + a2
1. Algebarski razlomak x+ · : za |x| = |y| je
x+y x2 − y2 x−y
jednak
1−a a
A. 1 + a B. x + y C. D.
x+y xy
2. Ostatak pri dijeljenju polinoma p(x) = x10 + 2x5 + 1 s polinomom q(x) = x + 1
jednak je
A. x B. 2 C. 0 D. −1
2−i
3. Apsolutna vrijednost kompleksnog broja jednaka je
√ 3 − 2i √
65 √ 13
A. B. 4 C. 13 D.
13 4
4. Rješenje nejednadžbe log (x + 2)  1 je
A. [−2, 8] B.  −∞, −2] C.  −∞, 8] D.  −2, 8]
5. Ako je g(x) = 2x − 8 i h(x) = x2 − 2 , tada je g(h(2)) jednako
A. −4 B. 4 C. x D. 2
6. Neka je f (x) = |x + 3| − 4 . Minimalna vrijednost funkcije f je
A. 2 B. −4 C. 3 D. −3
7. Koliko realnih rješenja ima jednadžba x3 + 3x2 − x − 3 = 0 ?
A. 1 B. 2 C. 3 D. 4
8. Rješenje kvadratne nejednadžbe −x2 + 5x + 6  0 je
A. [−1, 6] B.  −∞, 6] C. [−1, +∞ D.  0, +∞
9. Vrhovi četverokuta su redom točke A(3, 2), B(1, −2), C(2, −5), D(4, −1). Taj
četverokut je
A. kvadrat B. pravokutnik C. paralelogram D. romb

138 Matematičko-fizički list, LIX 2 (2008. – 2009.)


10. Ako se radijus sfere udvostruči, što će se dogoditi s oplošjem sfere?
A. porast će za faktor 2 B. porast će za faktor 4
C. porast će za faktor 6 D. porast će za faktor 8
11. Volumen uspravnog stošca je 12π , a polumjer baze r = 3 . Visina stošca je
A. 1 B. 2 C. 3 D. 4
12. Mate i Sanja su udaljeni 2000 m i oboje gledaju balon. U isto vrijeme, Sanja vidi
balon pod kutem od 42◦ , a Mate pod kutem od 70◦ . Sanja je udaljena od balona
A. 2026.98 m B. 1984.37 m C. 1443.36 m D. 2086.95 m
13. Opseg paralelograma na slici je 80 cm . Površina tog paralelograma je

A. 276 cm2 B. 144 cm2 C. 138 cm2 D. 84 cm2


14. Zbroj rješenja jednadžbe 22x + 2 · 2x − 8 = 0 je
2
A. −2 B. 1 C. 4 D.
3
15. Krivulje y = x2 + 2 i y = 4−x se sijeku u
A. I. kvadrantu B. II. kvadrantu C. III. kvadrantu D. IV. kvadrantu
3
16. Središte i radijus kružnice x2 + x + y2 − 2y = je
1  1 4 √
A. S , 1 , r = 2 B. S − , 1 , r = 2
2 2
 1 3  1
C. S − , −1 , r = D. S − , 1 , r = 2
2 4 2

17. Domena funkcije f (x) = ax − 1 za 0 < a < 1 je
A.  −∞, 0] B. [0, +∞ C.  1, +∞ D.  −∞, −1]
 π
n · cos n ·
18. Četvrti član niza an = 2 jednak je
n!
1 1 1 1
A. B. C. D.
2 4 6 8
19. Odredite realni parametar a tako da su zadani pravci ax + (a − 1)y = 5 i
x + ay = 0 okomiti.
A. 0 B. −2 C. −3 D. 1
20. Omjer površina baza krnje piramide jednak je
A. omjeru visine krnje piramide i visine cijele piramide
B. volumenu krnje piramide
C. omjeru kvadrata njihovih udaljenosti od vrha potpune piramide
D. omjeru njihovih udaljenosti od vrha potpune piramide

Matematičko-fizički list, LIX 2 (2008. – 2009.) 139


Grupa B

a3 − 8b3 a2 + 2ab + 4b2


1. Algebarski izraz : je jednak
a2 − 4b2 a2 + 2ab
A. 1 B. a C. a + 2b D. 1 − ab

6
3

2. Izraz 3x2 y : 9xy2 · 27x4 y3 je jednak

A. 3x2 y B. 6 3x2 y3 C. 3 xy D. 9xy2
1
3. Imaginarni dio kompleksnog broja je jednak
−1 + 2i
1 2 5
A. 0 B. − C. − D.
2 5 2
2
4. Kakav treba biti realni parametar a tako da kvadratna jednadžba −3x − ax− 4 = 0
nema realnih√rješenja?
√ √ √
A. a ∈  −4 3, 4 3 B. a ∈  −∞, 4 3 C. a ∈ Re D. a ∈ [− 48, 0]
5. Luka je ispucao loptu. Visina u metrima h(t) koju dostiže lopta nakon t sekundi
je h(t) = −16t2 + 48t . Lopta će udariti u tlo nakon
25
A. 1.5 s B. 2 s C. s D. 3 s
8
6. Ako je f (x) = 3x−2 i g(x) = x2 − x , tada je g(f (3)) jednako
A. 6 B. 3 C. 2 D. −4
2
x − 7x + 10
7. Rješenje nejednadžbe > 0 je
x−4
A. x ∈  2, 4 ∪  5, ∞ B. x ∈  −∞, 5 C. x ∈  4, 5 D. x ∈ [5, +∞
8. Rješenje jednadžbe log5 (x − 4) + log5 (x + 2) = log5 16 pripada intervalu
A. [0, 4] B.  4, 10] C.  −∞, 0 D.  10, +∞
9. Cijena iznajmljivanja bicikla najprije je povećana 25%, a zatim snižena 22%. Što
treba napraviti s trenutnom cijenom da opet bude jednaka početnoj?
A. Povećati je 3% B. Sniziti je 3% C. Povećati je 2.56% D. Sniziti je 2.56%
x2
10. Domena funkcije f (x) = log2 je
x+1
A.  −∞, −1 ∪  1, +∞ B.  −1, 0 C.  −∞, −1] D.  −1, +∞ \ {0}
-
11. Nadite površinu osjenčanog kružnog odsječka.

A. 17.01 cm2 B. 18.27 cm2 C. 18.53 cm2 D. 19.34 cm2


12. Odredite koordinate središta S i duljinu polumjera r kružnice x + y2 + 8x +
2
2y + 13 = 0 .
A. S(−4, −1), r = 2 B. S(4, 1), r = 2 C. S(−4, 1), r = 4 D. S(8, 2), r = 13

140 Matematičko-fizički list, LIX 2 (2008. – 2009.)


13. Polinom drugog stupnja koji prolazi točkama A(−1, −6), B(0, −3) i C(1, −4)
je polinom
A. f (x) = −6x2 − 3x − 4 B. f (x) = 5x2 + 3x + 5
C. f (x) = −2x2 + x − 3 D. f (x) = −3x2 + x − 2
-
14. Kakav trokut odreduju pravci y = 3x − 1 , x − 7y − 7 = 0 i x + y − 7 = 0 ?
A. pravokutan B. tupokutan
C. jednakokračan D. trokut sa sve tri stranice različitih duljina
15. Ako je oplošje valjka 8π cm2 , a polumjer baze jednak visini valjka, tada je
volumen valjka jednak √ √
A. 2 cm3 B. 2π cm3 C. 8π cm3 D. 2π 2 cm3
16. Formula za volumen V = B · v, gdje je B površina baze, a v visina tijela, vrijedi
za
A. prizmu i valjak B. stožac i kvadar
C. piramidu i kuglu D. paralelepiped i krnju piramidu
17. Na koliko se načina 5 ljudi može posjesti oko okruglog stola?
A. 120 B. 24 C. 20 D. 48
cos x
18. Izraz + tg x je jednak
1 + sin x
1
A. tg x B. cos x C. D. ctg x
cos x
19. U kakvom su odnosu pravac y = −2x + 9 i parabola y2 = 3x ?
A. pravac je normala parabole B. pravac je tangenta parabole
C. pravac ne siječe parabolu D. pravac siječe parabolu u ishodištu

20. Treći član geometrijskog√ niza kojemu su prva dva člana 2 i 4 je jednak √
A. 8 B. 4 2 C. 6 D. 8 2

Grupa C

a(a + 2)
1. Izraz je jednak
(a + 1)4 − 1
a+2 1 1
A. B. 2 C. 1 D.
a−2 a +2 a2 + 2a + 2

3+2 3
2. Racionalizacijom nazivnika u izrazu √ dobije se
√ √ 3+2 √ √
A. 3 B. − 3 C. 2 3 D. 3 + 2
3. Broj rješenja nejednadžbe x2 − 5x + 6  0 u skupu Z je
A. 1 B. 2 C. 3 D. 4
4. Otopina soli A miješa se sa 16% otopinom soli B u omjeru 3 : 4 i dobije se 22%
otopina. Postotak soli u otopini A iznosi
A. 20% B. 24% C. 28% D. 30%
3 3 7 2 5 9
5. Nakon što se podijele potencije (−21x a b) : (7xa b ) dobije se
37 x12 a3 214x6 a 21 73 x12 a
A. − 2 38 B. − 16
C. abx D. − 7 28
7 b 9b 7 3b

Matematičko-fizički list, LIX 2 (2008. – 2009.) 141


6. Polinom p(x) = x4 + 3x3 + 4x2 + x + a je djeljiv s q(x) = x2 + x − 1 ako je a
jednako
A. 3 B. −3 C. 4 D. −4
 1 + i 4
7. Imaginarni dio kompleksnog broja je jednak
1−i
A. −1 B. 1 C. 2 D. 0
8. Ordinata točke na osi y koja je jednako udaljena od točaka A(1, −5) i B(7, 3) je
jednaka
A. 2 B. 3 C. 0 D. −2
x+1
9. Skup svih rješenja nejednadžbe  2 je
x−1
A.  1, 3] B. [1, +∞ C.  −∞, 1 ∪ [3, +∞ D. R
10. Broj realnih rješenja jednadžbe log x4 + log (x + 4)4 = 4 log 3 je
A. 1 B. 2 C. 3 D. 4
11. Napišite preslikavanje u obliku F(x, y) = (?, ?) tako da opisuje transformaciju
koja točku (x, y) osno simetrično preslikava s obzirom na os y.
A. F(x, y) = (−x, y) B. F(x, y) = (y, x) C. F(x, y) = (x, −y) D. F(x, y) = (−x, −y)
12. Na slici je graf funkcije f (x) = logb x . Odredite b .

A. b = 2 B. b = 4 C. b = 8 D. b = 16
2 2
13. Radijus kružnice x + 2x √ + y − 6y = 0 je √
A. 10 B. 10 C. 6 D. 2
14. Opseg baze pravilne √ uspravne četverostrane piramide je 24 cm, a površina
dijagonalnog presjeka je 3 14 cm2 . Volumen piramide √ je √
A. 72 cm3 B. 36 cm3 C. 12 7 cm3 D. 4 14 cm3
15. Hipotenuza pravokutnog trokuta iznosi c = 13 cm , a kut pri vrhu A je α = 30◦ .
Površina√trokuta je
169 3 √ √
A. cm2 B. 169 cm2 C. 13 3 cm2 D. 144 3 cm2
8
16. Točka presjeka pravaca 2x + 4y = 3 i 3x − y = 0 se nalazi na
A. osi x B. pravcu 7x + 7y − 6 = 0 C. kružnici x2 + y2 = 1 D. osi y
2 2 2
17. Zajednička tetiva krivulja
√ y = 8x i x − y = 9 ima duljinu √
A. 4 B. 3 6 C. 12 D. 12 2
18. Rješenje jednadžbe 3 · 2x − 2x−1 = 20 zadovoljava uvjet
5 5 9 9
A. x < 0 B. 0 < x  C. < x  D. x >
2 2 2 2
19. Odredite istinitu tvrdnju.
A. Funkcija f (x) = log x poprima samo pozitivne realne vrijednosti.
-
B. Funkcije g(x) = log2 x i h(x) = 2x su medusobno inverzne.
x
C. Jednadžba a = 0 (a > 0) ima uvijek dva realna rješenja.
D. Os x je asimptota funkcije f (x) = log x .

142 Matematičko-fizički list, LIX 2 (2008. – 2009.)



2
20. Ako je cos x = , cos 2x jednako je
4
3 4 4 4
A. − B. C. D. −
4 5 3 5

Rješenja zadataka

Grupa A
1. A 2. C 3. A 4. D 5. A 6. B 7. C 8. A 9. C 10. B
11. D 12. A 13. A 14. B 15. B 16. B 17. A 18. C 19. A 20. C
Grupa B
1. B 2. A 3. C 4. A 5. D 6. A 7. A 8. B 9. C 10. D
11. B 12. A 13. C 14. C 15. D 16. A 17. B 18. C 19. A 20. D
Grupa C
1. D 2. A 3. B 4. D 5. A 6. B 7. D 8. A 9. C 10. D
11. A 12. A 13. B 14. C 15. A 16. B 17. D 18. C 19. B 20. A

Priredili Blaženka Divjak i Damir Horvat

SVIM SURADNICIMA

U Matematičko–fizičkom listu objavljuju se članci iz matematike, fizike i informatike, s


malim prilogom iz astronomije, zadaci i rješenja, prikazi natjecanja i ljetnih škola iz matematike
i fizike, zanimljivosti u obliku članaka i zadataka od učenika, profesora i ostalih matematičara,
novosti iz znanosti, zadaci s razredbenih (kvalifikacijskih) ispita, zabavna matematika i nagradni
natječaj.
Prilozi trebaju biti napisani računalom (Word, Tex, Latex) ili pisaćim strojem sa širokim
proredom na formatu A-4. Uz kopiju pošaljite i disketu.
Slike trebaju biti jasno nacrtane na posebnom papiru i pogodne za presnimavanje. Slike crtane
računalom (eps, tif, gif, jpg i sl.) pošaljite i na disketi.
Članci neka ne budu dulji od osam stranica, a ako je to potrebno neka budu napisani u
nastavcima.
Pozivaju se učenici da pošalju članak o nekoj od spomenutih tema, originalne zadatke s
rješenjima ili prikaze nekih manifestacija (ljetne škole, susreti učenika, rad školske grupe).
Kako se rukopisi ne vraćaju, sačuvajte original a pošaljite kopiju na papiru formata A-4.
-
Svi rukopisi podliježu recenziji redakcije ili neke stručne osobe za odredeno područje.
Prilozi se šalju na adresu ovog časopisa koja je na prvoj stranici lista.

RJEŠAVATELJIMA ZADATAKA

Svako rješenje neka bude napisano na posebnom papiru (formata A-4 ili A-5) i to samo
na jednoj strani papira. Uz svako rješenje na vrhu papira treba potpuno ispisati tekst zadatka.
Svako rješenje treba čitljivo potpisati (ime i prezime), naznačiti razred, školu i mjesto.

Matematičko-fizički list, LIX 2 (2008. – 2009.) 143


Rješenje nagradnog natječaja br. 183

Rješenje. Koristeći binomnu formulu dobivamo


! ! ! !
100 100 100 2 100 100 100
(1 + 2) = +2 +2 + ... + 2 ,
0 1 2 100
tj.
xy = 3100 = (32 )50 = (34 )25 = (35 )20 = (325 )4 = (310 )10 = (320 )5 = (350 )2 ,
odakle je (x, y) ∈ {(3, 100), (32 , 50), (34 , 25), (35 , 20), (325 , 4), (310 , 10), (320 , 5), (350 , 2)} .
-
Knjigom su nagradeni rješavatelji:
1. Edin Ajanović (3), Prva bošnjačka gimnazija, Sarajevo, BiH;
2. Kristijan Kvaternik (1), V. gimnazija, Zagreb.

Riješili zadatke iz br. 4/232

(Broj u zagradi označava razred–godište srednje–osnovne škole.)


a) Iz matematike: Edin Ajanović (3), I. bošnjačka gimnazija, Sarajevo, BiH, 3105–3108,
3110–3113, 3115, 3117, 3118; Davor Devald (1), I. prirodoslovno-matematička gimnazija, SŠ
Izidora Kršnjavoga, Našice, 3105–3108, 3110, 3116; Kristijan Kvaternik (1), V. gimnazija,
Zagreb, 3105–3108; Vanja Ubović (2), Gimnazija Petra Preradovića, Virovitica, 3106–3108,
3112, 3115.
b) Iz fizike: Ana Lucija Alviž (7), OŠ Fausta Vrančića, Šibenik, 278; Filip Dunaj (8),
OŠ Augusta Cesarca, Krapina 278, 280; Arijan Golub (8), OŠ Augusta Cesarca, Krapina 278,
280; Juraj Krsnik (8), OŠ Augusta Cesarca, Krapina 278, 280; Dorian Lacko (8), OŠ Augusta
Cesarca, Krapina 278, 280; Ana Paić (7), OŠ Fausta Vrančića, Šibenik 278–281; Karlo Radečić
(7), OŠ Fausta Vrančića, Šibenik, 278, 280; Kristijan Kvaternik (1), V. gimnazija, Zagreb, 1392,
1395, 1396.

Nagradni natječaj br. 185

Dokaži da je broj
11 . 
. . 111 22 . 
. . 222 5
2008 2009
potpun kvadrat.

144 Matematičko-fizički list, LIX 2 (2008. – 2009.)

You might also like